বাস্তব সংখ্যা
Real Numbers
barcode
এ অধ্যায়ের পাঠ্যসূচী।
ঐতিহাসিক পটভূমি
Historical Background
straight3
জর্জ কান্টর (Georg Cantor)
(১৮৪৫ খ্রিস্টাব্দ-১৯১৮ খ্রিস্টাব্দ)
জার্মান গণিতবিদ
সৃষ্টির শুরু থেকেই মানুষের চারপাশে যা কিছু বর্তমান তার হিসাব রাখা এবং গণনার জন্যই মূলত সংখ্যার সৃষ্টি। মানব সমাজের ক্রমবর্ধমান উন্নতির সঙ্গে সঙ্গে সংখ্যার ব্যবহারেরও ক্রমবিকাশ ঘটেছে। আধুনিক বিশ্বের সর্বাধুনিক আবিষ্কার কম্পিউটার এর কর্মপদ্ধতিও তৈরি করা হয় সংখ্যাকে কাজে লাগিয়ে।
সংখ্যার ধারণা অতি প্রাচীন। সংখ্যার উৎপত্তি কখন হয়েছিল তা সঠিকভাবে জানা সম্ভব হয়নি। স্বাভাবিক সংখ্যা দিয়ে গণনা শুরু হলেও সময়ের ব্যবধানে নতুন নতুন সংখ্যার লিখন পদ্ধতি পরিপূর্ণরূপে প্রকাশ পেয়েছে। পূর্ণ সংখ্যা ও ভগ্নাংশ নিয়ে মূলদ সংখ্যা গঠিত হয়। মূলদ সংখ্যা ও অমূলদ সংখ্যা নিয়েই বাস্তব সংখ্যা।
যীশুখৃষ্টের জন্মের প্রায় দুই হাজার বছর পূর্বে সংখ্যাভিত্তিক গণিতের সৃষ্টি হয়েছিল। ইতিহাসবিদদের ধারণা, ভারতীয় ও চীনা দার্শনিকরা পূর্ণসংখ্যার, মিশরীয়রা ভগ্নাংশের ও গ্রীক দার্শনিকরা জ্যামিতিক চিত্র অঙ্কনের সূচনা করেছিল।
খ্রিষ্টপূর্ব ১০০০ এর মধ্যে মিশরের গণিতবিদগণ সামান্য ভগ্নাংশ (Vulgar fraction) ব্যবহার করেন। খ্রিষ্টপূর্ব (৭৫০-৬৯০) এর মধ্যে ভারতীয় এবং খ্রিষ্টপূর্ব ৫০০ এর মধ্যে গ্রিসের গণিতবিদগণ অমূলদ সংখ্যার ধারণা দেন। ইংরেজ গণিতবিদ জন ওয়ালিস (John Wallies)straight3 জন ওয়ালিস (John Wallies)
(১৬১৬খ্রিস্টাব্দ-১৭০৩খ্রিস্টাব্দ)
জন ওয়ালিস ছিলেন একজন ইংরেজ পাদ্রি এবং গণিতবিদ যিনি অসীম ক্যালকুলাসের বিকাশের জন্য আংশিক কৃতিত্ব প্রদান করেন। ১৬৪৩ থেকে ১৬৮৯ এর মধ্যে তিনি পার্লামেন্ট এবং পরে, রাজদরবারের প্রধান ক্রিপ্টোগ্রাফার হিসাবে কাজ করেছিলেন। অনন্তের ধারণার প্রতিনিধিত্ব করার জন্য the প্রতীকটি প্রবর্তনের কৃতিত্ব তার।
(১৬১৬খ্রিস্টাব্দ-১৭০৩খ্রিস্টাব্দ) এবং ফ্রেন্স গণিতবিদ পিয়ারে বগার (Pierre Bouguer)straight3পিয়ারে বগার (Pierre Bouguer)
(১৬৯৮খ্রিস্টাব্দ-১৭৫৮খ্রিস্টাব্দ)
পিয়ের বাউগার (Pierre Bouguer) ছিলেন একজন ফরাসি গণিতবিদ, ভূ -পদার্থবিদ, ভূতাত্ত্বিক এবং জ্যোতির্বিজ্ঞানী। তিনি "নৌ স্থাপত্যের জনক" নামেও পরিচিত।
(১৬৯৮খ্রিস্টাব্দ-১৭৫৮খ্রিস্টাব্দ) যথাক্রমে ১৬৭০ এবং ১৭৩৪ খ্রিষ্টাব্দে সর্বপ্রথম অসমতার চিহ্ন (\(\le\) এবং \(\ge\) ) ব্যবহার করেন। এছাড়া "The Analytical Arts Applied to Solving Algebraic Equations" বইটিতে বৃটিশ গণিতবিদ ও দার্শনিক টমাস হ্যারিয়ট (Thomas Harriot) straight3টমাস হ্যারিয়ট (Thomas Harriot)
(১৫৬০খ্রিস্টাব্দ-১৬২১খ্রিস্টাব্দ)
টমাস হ্যারিয়ট (Thomas Harriot) তিনি ছিলেন একজন ইংরেজ জ্যোতির্বিজ্ঞানী, গণিতবিদ, নৃতাত্ত্বিক এবং অনুবাদক, যার প্রতিফলন তত্ত্বকে দায়ী করা হয়।
(১৫৬০খ্রিস্টাব্দ-১৬২১খ্রিস্টাব্দ) বৃহত্তম ও ক্ষুদ্রতম চিহ্ন (\(\gt\) এবং \(\lt\) ) ব্যবহার করেন যা ১৬৩১ খ্রিষ্টাব্দে প্রকাশিত হয়।
কসি-সোয়াজ অসমতা (Cauchy-Schwarz inequality) লিনিয়ার অ্যালজ্যাবরায় ও পরিসংখ্যানে খুবই গুরুত্বপূর্ণ অসমতা হিসেবে বিবেচিত হয়।
বাস্তব সংখ্যার বিষদ বিবরণ
Details of Real Numbers
বাস্তব সংখ্যা সম্পর্কে স্পষ্ট ধারণার জন্য পর্যায়ক্রমে স্বাভাবিক সংখ্যা, পূর্ণসংখ্যা, অ-ঋনাত্মক পূর্ণসংখ্যা, ঋনাত্মক পূর্ণসংখ্যা, ঋনাত্মক সংখ্যা, মৌলিক সংখ্যা, কৃত্রিম সংখ্যা এবং মূলদ ও অমূলদ সংখ্যা সম্পর্কে ধারণা থাকা প্রয়োজন।
স্বাভাবিক সংখ্যাঃ সকল ধনাত্মক পূর্ণসংখ্যাকে স্বাভাবিক সংখ্যা (Natural Number) বলা হয়। সকল স্বাভাবিক সংখ্যার সেটকে \(\mathbb{N}\) দ্বারা প্রকাশ করা হয়।
অর্থাৎ \(\mathbb{N}=\left\{1, \ 2, \ 3, ....\right\}\)।
গণনার প্রয়োজনেই স্বাভাবিক সংখ্যা আবিষ্কৃত হয়, এ কারণে স্বাভাবিক সংখ্যাকে গণনাকারী সংখ্যাও বলা হয়। সকল ধনাত্মক পূর্ণসংখ্যার সেটকে \(\mathbb{Z_{\gt{0}}}\) দ্বারা প্রকাশ করা হয়।
অর্থাৎ \(\mathbb{Z_{\gt{0}}}=\mathbb{N}=\left\{1, \ 2, \ 3, ....\right\}\)।
পূর্ণসংখ্যাঃ সকল ধনাত্মক পূর্ণসংখ্যা, ঋনাত্মক পূর্ণসংখ্যা ও শূন্য নিয়ে পূর্ণসংখ্যার সেট (Set of integer) গঠিত। পূর্ণসংখ্যার সেটকে \(\mathbb{Z}\) দ্বারা প্রকাশ করা হয়।
অর্থাৎ \(\mathbb{Z}=\left\{...., \ -3, \ -2, \ -1, \ 0, \ 1, \ 2, \ 3, ....\right\}\)।
অ-ঋনাত্মক পূর্ণসংখ্যাঃ শূন্য \((0)\) সহ সকল স্বাভাবিক সংখ্যাকে স্বাভাবিক সংখ্যাঃ সকল ধনাত্মক পূর্ণসংখ্যাকে স্বাভাবিক সংখ্যা (Natural Number) বলা হয়। সকল স্বাভাবিক সংখ্যার সেটকে \(\mathbb{N}\) দ্বারা প্রকাশ করা হয়।
অর্থাৎ \(\mathbb{N}=\left\{1, \ 2, \ 3, ....\right\}\)।
অ-ঋনাত্মক পূর্ণসংখ্যা (Non negative integer) বলা হয়। সকল অ-ঋনাত্মক পূর্ণসংখ্যার সেটকে \(\mathbb{Z_{\ge{0}}}\) দ্বারা প্রকাশ করা হয়।
অর্থাৎ \(\mathbb{Z_{\ge{0}}}=\left\{0, \ 1, \ 2, \ 3, ....\right\}\)।
\(=\left\{0\right\}\cup\mathbb{Z_{\gt{0}}}\)।
ঋনাত্মক পূর্ণসংখ্যাঃ শূন্য \((0)\) অপেক্ষা ছোট পূর্ণসংখ্যাকে ঋনাত্মক পূর্ণসংখ্যা (Negative integer) বলা হয়। সকল ঋনাত্মক পূর্ণসংখ্যার সেটকে \(\mathbb{Z_{\lt{0}}}\) দ্বারা প্রকাশ করা হয়।
অর্থাৎ \(\mathbb{Z_{\lt{0}}}=\left\{......, \ -3, \ -2, \ -1\right\}\)।
ঋনাত্মক সংখ্যাঃ শূন্য \((0)\) অপেক্ষা ছোট সংখ্যাকে ঋনাত্মক সংখ্যা (Negative number) বলা হয়। সকল ঋনাত্মক সংখ্যার সেট নিম্নরূপঃ
\(\left\{-1, \ -2, \ -\frac{1}{2}, \ -\frac{1}{3}, \ -\sqrt{2}, -0.322, \ -0.63, \ -4.12034506 \ ........\text{ইত্যাদি।} \right\}\)।
মৌলিক সংখ্যাঃ \(1\) ব্যতীত যে সকল স্বাভাবিক সংখ্যা স্বাভাবিক সংখ্যাঃ সকল ধনাত্মক পূর্ণসংখ্যাকে স্বাভাবিক সংখ্যা (Natural Number) বলা হয়। সকল স্বাভাবিক সংখ্যার সেটকে \(\mathbb{N}\) দ্বারা প্রকাশ করা হয়।
অর্থাৎ \(\mathbb{N}=\left\{1, \ 2, \ 3, ....\right\}\)।
কেবলমাত্র ঐ সংখ্যা ও \(1\) দ্বারা বিভাজ্য, ঐ সকল সংখ্যাকে মৌলিক সংখ্যা (Prime number) বলা হয়। সকল মৌলিক সংখ্যার সেটকে \(\mathbb{P}\) দ্বারা প্রকাশ করা হয়।
অর্থাৎ \(\mathbb{P}=\left\{2, \ 3, \ 5, \ 7, \ 11, \ 13, \ 17, \ 19, \ 23, \ ......\right\}\)।
বিকল্প সঙ্গাঃ
মৌলিক সংখ্যাঃ \(1\) ব্যতীত যে সকল স্বাভাবিক সংখ্যাকে স্বাভাবিক সংখ্যাঃ সকল ধনাত্মক পূর্ণসংখ্যাকে স্বাভাবিক সংখ্যা (Natural Number) বলা হয়। সকল স্বাভাবিক সংখ্যার সেটকে \(\mathbb{N}\) দ্বারা প্রকাশ করা হয়।
অর্থাৎ \(\mathbb{N}=\left\{1, \ 2, \ 3, ....\right\}\)।
দুইয়ের অধিক উৎপাদকে বিশ্লেষণ করা যায় না, ঐ সকল সংখ্যাকে মৌলিক সংখ্যা (Prime number) বলা হয়।
যেমনঃ \(2=2\times1\)।
\(3=3\times1\)।
\(5=5\times1\)।
\(7=7\times1\)।
\(11=11\times1\)।
\(13=13\times1\)।
\(17=17\times1\)।
\(17=17\times1\)।
\(23=23\times1\)।
\(... \ ... \ ...\ ...\ ...\)
\(... \ ... \ ...\ ...\ ...\)
\(... \ ... \ ...\ ...\ ...\)
অর্থাৎ \(\mathbb{P}=\left\{2, \ 3, \ 5, \ 7, \ 11, \ 13, \ 17, \ 19, \ 23, \ ......\right\}\)।
কৃত্রিম সংখ্যাঃ \(1\) ব্যতীত যে সকল স্বাভাবিক সংখ্যা স্বাভাবিক সংখ্যাঃ সকল ধনাত্মক পূর্ণসংখ্যাকে স্বাভাবিক সংখ্যা (Natural Number) বলা হয়। সকল স্বাভাবিক সংখ্যার সেটকে \(\mathbb{N}\) দ্বারা প্রকাশ করা হয়।
অর্থাৎ \(\mathbb{N}=\left\{1, \ 2, \ 3, ....\right\}\)।
ঐ সংখ্যা ও \(1\) ব্যতীত এক বা একাধিক সংখ্যা দ্বারা বিভাজ্য, ঐ সকল সংখ্যাকে কৃত্রিম সংখ্যা (Composite number) বলা হয়। কৃত্রিম সংখ্যার সেট নিম্নরূপঃ
কৃত্রিম সংখ্যার সেট \(=\left\{4, \ 6, \ 8, \ 9, \ 10, \ 12, \ 14, \ 15, \ 16, \ 18, \ 20, \ 21, \ ......\right\}\)।
বিকল্প সঙ্গাঃ
কৃত্রিম সংখ্যাঃ \(1\) ব্যতীত যে সকল স্বাভাবিক সংখ্যাকে স্বাভাবিক সংখ্যাঃ সকল ধনাত্মক পূর্ণসংখ্যাকে স্বাভাবিক সংখ্যা (Natural Number) বলা হয়। সকল স্বাভাবিক সংখ্যার সেটকে \(\mathbb{N}\) দ্বারা প্রকাশ করা হয়।
অর্থাৎ \(\mathbb{N}=\left\{1, \ 2, \ 3, ....\right\}\)।
দুইয়ের অধিক উৎপাদকে বিশ্লেষণ করা যায়, ঐ সকল সংখ্যাকে কৃত্রিম সংখ্যা (Composite number) বলা হয়।
যেমনঃ \(4=2\times2\times1\)।
\(6=3\times2\times1\)।
\(8=2\times2\times2\times1\)।
\(9=3\times3\times1\)।
\(10=5\times2\times1\)।
\(12=3\times2\times2\times1\)।
\(14=7\times2\times1\)।
\(16=2\times2\times2\times2\times1\)।
\(18=3\times3\times2\times1\)।
\(20=5\times2\times2\times1\)।
\(21=7\times3\times1\)।
\(... \ ... \ ...\ ...\ ...\)
\(... \ ... \ ...\ ...\ ...\)
\(... \ ... \ ...\ ...\ ...\)
কৃত্রিম সংখ্যার সেট \(=\left\{4, \ 6, \ 8, \ 9, \ 10, \ 12, \ 14, \ 15, \ 16, \ 18, \ 20, \ 21, \ ......\right\}\)।
সহমৌলিকঃ যদি দুইটি পূর্ণ সংখ্যার মধ্যে \(1\) ব্যতীত অন্য কোনো সাধারণ উৎপাদক না থাকে তবে তাদেরকে একে অপরের সহমৌলিক (Coprime) বলে। অন্য কোনো সাধারণ উৎপাদক না থাকায় এদের একটি দ্বারা অন্যটি কখনই নিঃশেষে বিভাজ্য নয়।
যেমনঃ \(3\) ও \(5\); \(4\) ও \(9\); \(5\) ও \(12\) ইত্যাদি।
মূলদ সংখ্যাঃ যে সকল সংখ্যাকে \(\frac{p}{q}\) আকারে প্রকাশ করা যায় (যেখানে, \(p, \ q\in{\mathbb{Z}}\) এবং \(q\ne{0}\)) ঐ সকল সংখ্যাকে মূলদ সংখ্যা (Rational number) বলা হয়। সকল মূলদ সংখ্যার সেটকে \(\mathbb{Q}\) দ্বারা প্রকাশ করা হয়।
অর্থাৎ \(\mathbb{Q}=\left\{\frac{p}{q}; \ p, \ q\in{\mathbb{Z}} \text{ এবং } \ q\ne{0}\right\}\)।
বিশেষ দ্রষ্টব্যঃ
দুইটি সংখ্যার যোগ, বিয়োগ ও গুণের ফলে অপর একটি পূর্ণসংখ্যা পাওয়া যায় কিন্তু দুইটি পূর্ণসংখ্যা ভাগ করলে ভাগফল পূর্ণ সংখ্যা নাও হতে পারে।
যেমনঃ \(\frac{9}{3}=3\)
কিন্তু \(\frac{3}{9}=\frac{1}{3}\) যা পূর্ণ সংখ্যা নয়। সুতরাং এ ধারণা থেকেই সংখ্যার একটি নতুন শ্রেণির আবির্ভাব ঘটে, যা ভগ্নাংশ (Fraction) হিসেবে পরিচিত।
যদি \(q=1\) হয় তবে, \(\frac{p}{q}\) আকারের সকল মূলদ সংখ্যাগুলি মূলদ সংখ্যাঃ যে সকল সংখ্যাকে \(\frac{p}{q}\) আকারে প্রকাশ করা যায় (যেখানে, \(p, \ q\in{\mathbb{Z}}\) এবং \(q\ne{0}\)) ঐ সকল সংখ্যাকে মূলদ সংখ্যা (Rational number) বলা হয়। সকল মূলদ সংখ্যার সেটকে \(\mathbb{Q}\) দ্বারা প্রকাশ করা হয়।
অর্থাৎ \(\mathbb{Q}=\left\{\frac{p}{q}; \ p, \ q\in{\mathbb{Z}} \text{ এবং } \ q\ne{0}\right\}\)।
পূর্ণসংখ্যা হয়।
সুতরাং মূলদ সংখ্যা মূলদ সংখ্যাঃ যে সকল সংখ্যাকে \(\frac{p}{q}\) আকারে প্রকাশ করা যায় (যেখানে, \(p, \ q\in{\mathbb{Z}}\) এবং \(q\ne{0}\)) ঐ সকল সংখ্যাকে মূলদ সংখ্যা (Rational number) বলা হয়। সকল মূলদ সংখ্যার সেটকে \(\mathbb{Q}\) দ্বারা প্রকাশ করা হয়।
অর্থাৎ \(\mathbb{Q}=\left\{\frac{p}{q}; \ p, \ q\in{\mathbb{Z}} \text{ এবং } \ q\ne{0}\right\}\)।
হয় ভগ্নাংশ অথবা পূর্ণসংখ্যা।
অমূলদ সংখ্যাঃ যে সকল সংখ্যাকে \(\frac{p}{q}\) আকারে প্রকাশ করা যায় না (যেখানে, \(p, \ q\in{\mathbb{Z}}\) ও সহমৌলিক সহমৌলিকঃ যদি দুইটি পূর্ণ সংখ্যার মধ্যে \(1\) ব্যতীত অন্য কোনো সাধারণ উৎপাদক না থাকে তবে তাদেরকে একে অপরের সহমৌলিক (Coprime) বলে। অন্য কোনো সাধারণ উৎপাদক না থাকায় এদের একটি দ্বারা অন্যটি কখনই নিঃশেষে বিভাজ্য নয়।
যেমনঃ \(3\) ও \(5\); \(4\) ও \(9\); \(5\) ও \(12\) ইত্যাদি।
এবং \(q\ne{0}\)) ঐ সকল সংখ্যাকে অমূলদ সংখ্যা (Irrational number) বলা হয়। সকল অমূলদ সংখ্যার সেটকে \(\mathbb{Q^{\prime}}\) দ্বারা প্রকাশ করা হয়।
অর্থাৎ \(\mathbb{Q^{\prime}}=\mathbb{R}-\{\mathbb{Q}\}\) যেখানে, \(\mathbb{R}\) বাস্তব সংখ্যার সেট।
যেমনঃ \(\sqrt{2}, \ \sqrt{3}, \ \sqrt{5}, \ \sqrt{6}, \ \sqrt{7}, \ \sqrt{11}, \) তুরীয় সংখ্যা (\(e, \ \pi \)) প্রভৃতি অমূলদ সংখ্যা ।
বাস্তব সংখ্যাঃ সকল মূলদ মূলদ সংখ্যাঃ যে সকল সংখ্যাকে \(\frac{p}{q}\) আকারে প্রকাশ করা যায় (যেখানে, \(p, \ q\in{\mathbb{Z}}\) এবং \(q\ne{0}\)) ঐ সকল সংখ্যাকে মূলদ সংখ্যা (Rational number) বলা হয়। সকল মূলদ সংখ্যার সেটকে \(\mathbb{Q}\) দ্বারা প্রকাশ করা হয়।
অর্থাৎ \(\mathbb{Q}=\left\{\frac{p}{q}; \ p, \ q\in{\mathbb{Z}} \text{ এবং } \ q\ne{0}\right\}\)।
এবং অমূলদ অমূলদ সংখ্যাঃ যে সকল সংখ্যাকে \(\frac{p}{q}\) আকারে প্রকাশ করা যায় না (যেখানে, \(p, \ q\in{\mathbb{Z}}\) ও সহমৌলিক এবং \(q\ne{0}\)) ঐ সকল সংখ্যাকে অমূলদ সংখ্যা (Irrational number) বলা হয়। সকল অমূলদ সংখ্যার সেটকে \(\mathbb{Q^{\prime}}\) দ্বারা প্রকাশ করা হয়।
অর্থাৎ \(\mathbb{Q^{\prime}}=\mathbb{R}-\{\mathbb{Q}\}\) যেখানে, \(\mathbb{R}\) বাস্তব সংখ্যার সেট।
যেমনঃ \(\sqrt{2}, \ \sqrt{3}, \ \sqrt{5}, \ \sqrt{6}, \ \sqrt{7}, \ \sqrt{11}, \) তুরীয় সংখ্যা (\(e, \ \pi \)) প্রভৃতি অমূলদ সংখ্যা ।
সংখ্যাগুলিকে একত্রে বাস্তব সংখ্যা (Real Number) বলা হয়। অর্থাৎ প্রত্যেক মূলদ মূলদ সংখ্যাঃ যে সকল সংখ্যাকে \(\frac{p}{q}\) আকারে প্রকাশ করা যায় (যেখানে, \(p, \ q\in{\mathbb{Z}}\) এবং \(q\ne{0}\)) ঐ সকল সংখ্যাকে মূলদ সংখ্যা (Rational number) বলা হয়। সকল মূলদ সংখ্যার সেটকে \(\mathbb{Q}\) দ্বারা প্রকাশ করা হয়।
অর্থাৎ \(\mathbb{Q}=\left\{\frac{p}{q}; \ p, \ q\in{\mathbb{Z}} \text{ এবং } \ q\ne{0}\right\}\)।
বা অমূলদ অমূলদ সংখ্যাঃ যে সকল সংখ্যাকে \(\frac{p}{q}\) আকারে প্রকাশ করা যায় না (যেখানে, \(p, \ q\in{\mathbb{Z}}\) ও সহমৌলিক এবং \(q\ne{0}\)) ঐ সকল সংখ্যাকে অমূলদ সংখ্যা (Irrational number) বলা হয়। সকল অমূলদ সংখ্যার সেটকে \(\mathbb{Q^{\prime}}\) দ্বারা প্রকাশ করা হয়।
অর্থাৎ \(\mathbb{Q^{\prime}}=\mathbb{R}-\{\mathbb{Q}\}\) যেখানে, \(\mathbb{R}\) বাস্তব সংখ্যার সেট।
যেমনঃ \(\sqrt{2}, \ \sqrt{3}, \ \sqrt{5}, \ \sqrt{6}, \ \sqrt{7}, \ \sqrt{11}, \) তুরীয় সংখ্যা (\(e, \ \pi \)) প্রভৃতি অমূলদ সংখ্যা ।
সংখ্যাই এক একটি বাস্তব সংখ্যা (Real Number)। বাস্তব সংখ্যার (Real Number) সেটকে \(\mathbb{R}\) দ্বারা প্রকাশ করা হয়।
অর্থাৎ \(\mathbb{R}=\mathbb{Q}\cup{\mathbb{Q^{\prime}}}\)
চিত্রের সাহায্যে বাস্তব সংখ্যাঃ
realNumber
বাস্তব সংখ্যার জ্যামিতিক ব্যাখ্যাঃ যেকোনো বাস্তব সংখ্যাকে তার মান অনুসারে যে সরলরেখার উপর বিন্দুর সাহায্যে চিত্রের মাধ্যমে প্রকাশ করা হয় তাকে সংখ্যারেখা (The number line) বলা হয়। এই রেখাকে বাস্তব রেখাও (Real line) বলা হয়ে থাকে। সুতরাং সকল বাস্তব সংখ্যা এবং বাস্তব রেখাস্থ সকল বিন্দুর মধ্যে একটি এক-এক মিল (One-One correspondences) দেখানো যায়।
নিম্নে \(X^{\prime}X\) একটি অসীম দৈর্ঘ্যবিশিষ্ট সরলরেখা। \(O\) সরলরেখাটির উপর যেকোনো একটি বিন্দু। \(O\) বিন্দুকে \(0\) (শূন্য) ধরে, \(O\) এর ডানে প্রতি একক দূরত্বের বিন্দুসমূহকে \(1, \ 2, \ 3, \ ...... \) ইত্যাদি এবং বামের বিন্দুসমূহকে \(-1, \ -2, \ -3, \ ...... \) ইত্যাদি দ্বারা সূচিত করা হয়।
এভাবে \(\frac{1}{2}, \ -\frac{3}{5}, \ 2\frac{1}{4}, \ 3.5 \) ইত্যাদি যেকোনো বাস্তব সংখ্যা ও মূলদ সংখ্যা \(X^{\prime}X\) রেখার উপরিস্থিত বিভিন্ন বিন্দু দ্বারা সূচিত করা হয়।
realNumber
মনে করি \(AB\perp{X^{\prime}X}\) এবং \(OA=AB=1;\) তাহলে \(OAB\) একটি সমদ্বিবাহু সমকোণী ত্রিভুজ যার অতিভুজ \(OB=\sqrt{OA^2+AB^2}\)।
\(=\sqrt{1^2+1^2}\)।
\(=\sqrt{1+1}\)।
\(=\sqrt{2}\)।
এখন, \(O\) কে কেন্দ্র করে \(OB\) এর সমান ব্যাসার্ধ নিয়ে অঙ্কিত বৃত্তচাপ \(X^{\prime}X\) কে \(P\) বিন্দুতে ছেদ করে। অতএব, \(OP=\sqrt{2}\) এবং \(\sqrt{2}\) অমূলদ সংখ্যাটি \(X^{\prime}X\) রেখার উপরিস্থিত \(P\) বিন্দু দ্বারা সূচিত করা যায়। সুতরাং যেকোনো মূলদ অথবা অমূলদ সংখ্যা \(X^{\prime}X\) রেখার উপরিস্থিত যেকোনো বিন্দু দ্বারা সূচিত করা যায়।
বাস্তব সংখ্যার উপসেট
Subsets of real numbers
স্বাভাবিক সংখ্যার সেটঃ \(\mathbb{N}=\left\{1, \ 2, \ 3, ....\right\}\)।
পূর্ণসংখ্যার সেটঃ \(\mathbb{Z}=\left\{...., \ -3, \ -2, \ -1, \ 0, \ 1, \ 2, \ 3, ....\right\}\)।
অ-ঋনাত্মক পূর্ণসংখ্যার সেটঃ \(\mathbb{Z_{\ge{0}}}=\left\{0, \ 1, \ 2, \ 3, ....\right\}\)।
ঋনাত্মক পূর্ণসংখ্যার সেটঃ \(\mathbb{Z_{\lt{0}}}=\left\{......, \ -3, \ -2, \ -1\right\}\)।
ঋনাত্মক সংখ্যার সেটঃ \(\left\{-1, \ -2, \ -\frac{1}{2}, \ -\frac{1}{3}, \ -\sqrt{2}, -0.322, \ -0.63, \ -4.12034506 \ ........\text{ইত্যাদি।} \right\}\)।
মৌলিক সংখ্যার সেটঃ \(\mathbb{P}=\left\{2, \ 3, \ 5, \ 7, \ 11, \ 13, \ 17, \ 19, \ 23, \ ......\right\}\)।
কৃত্রিম সংখ্যার সেটঃ \(=\left\{4, \ 6, \ 8, \ 9, \ 10, \ 12, \ 14, \ 15, \ 16, \ 18, \ 20, \ 21, \ ......\right\}\)।
মূলদ সংখ্যার সেটঃ \(\mathbb{Q}=\left\{\frac{p}{q}; \ p, \ q\in{\mathbb{Z}} \text{ এবং } \ q\ne{0}\right\}\)।
অমূলদ সংখ্যার সেটঃ \(\mathbb{Q^{\prime}}=\mathbb{R}-\{\mathbb{Q}\}\) যেখানে, \(\mathbb{R}\) বাস্তব সংখ্যার সেট।
বাস্তব সংখ্যার সেটঃ \(\mathbb{R}=\mathbb{Q}\cup{\mathbb{Q^{\prime}}}\)
বাস্তব সংখ্যার শ্রেণীবিন্যাস
Classification of real numbers
realNumber
বাস্তব সংখ্যার স্বীকার্য ভিত্তিক বর্ণনা
Axioms of real numbers
বাস্তব সংখ্যার কয়েক প্রকার স্বীকার্য রয়েছে, এর মধ্যে বীজগণিতীয় গুণাবলি ভিত্তিক বা ফিল্ড স্বীকার্য এবং ক্রম ভিত্তিক স্বীকার্য অন্যতম।
বীজগণিতীয় গুণাবলি ভিত্তিক বা ফিল্ড স্বীকার্যঃ বাস্তব সংখ্যার সেট \(\mathbb{R}\) এর বীজগণিতীয় গুণাবলী ভিত্তিক স্বীকার্য মূলত যোগ \((+)\) এবং গুণন \((.)\) এর উপর নির্ভরশীল।
বাস্তব সংখ্যার স্বীকার্যসমূহ
Axioms of the real numbers
আবদ্ধতাঃ বাস্তব সংখ্যা \(\mathbb{R}\) যোগ \((+)\) এবং গুণন \((.)\) প্রক্রিয়ায় আবদ্ধ (Closure law)।
যদি \(a, \ b\in{\mathbb{R}}\) হয় তবে,
যোগে আবদ্ধঃ \(a+b\in{\mathbb{R}}\)
গুণনে আবদ্ধঃ \(ab\in{\mathbb{R}}\)
যেমনঃ \(2, \ 3\in{\mathbb{R}}\) হয় তবে,
\(2+3=5\in{\mathbb{R}}\) এবং \(2.3=6\in{\mathbb{R}}\)
বিনিময় যোগ্যতাঃ বাস্তব সংখ্যা \(\mathbb{R}\) যোগ \((+)\) এবং গুণন \((.)\) প্রক্রিয়ার জন্য বিনিময় যোগ্য (Commutative law)।
যদি \(a, \ b\in{\mathbb{R}}\) হয় তবে,
যোগের বিনিময় বিধিঃ \(a+b=b+a\)
গুণনের বিনিময় বিধিঃ \(ab=ba\)
যেমনঃ \(3, \ 4\in{\mathbb{R}}\) হয় তবে,
\(3+4=4+3\) এবং \(3.4=4.3\)
সংযোজন যোগ্যতাঃ বাস্তব সংখ্যা \(\mathbb{R}\) যোগ \((+)\) এবং গুণন \((.)\) প্রক্রিয়ার জন্য সংযোজন যোগ্য (Associative law)।
যদি \(a, \ b, \ c\in{\mathbb{R}}\) হয় তবে,
যোগের সংযোজন বিধিঃ \((a+b)+c=a+(b+c)\)
গুণনের সংযোজন বিধিঃ \((a.b).c=a.(b.c)\)
যেমনঃ \(3, \ 4, \ 5\in{\mathbb{R}}\) হয় তবে,
\((3+4)+5=3+(4+5)\) এবং \((3.4).5=3.(4.5)\)
বন্টন যোগ্যতা (Distributive law) বন্টন যোগ্যতাঃ বাস্তব সংখ্যা \(\mathbb{R}\) যোগ \((+)\) এবং গুণন \((.)\) প্রক্রিয়ার জন্য বন্টন যোগ্য (Distributive law)।
যদি \(a, \ b, \ c\in{\mathbb{R}}\) হয় তবে,
বাম বন্টন বিধিঃ \(a(b+c)=ab+ac\)
ডান বন্টন বিধিঃ \((b+c)a=ba+ca\)
যেমনঃ \(3, \ 4, \ 5\in{\mathbb{R}}\) হয় তবে,
\(3.(4+5)=3.4+3.5\) এবং \((4+5).3=4.3+5.3\)
অনন্যতাঃ বাস্তব সংখ্যা \(\mathbb{R}\) যোগ \((+)\) এবং গুণন \((.)\) প্রক্রিয়ার জন্য অনন্য (Uniqueness law)।
যদি \(a, \ b, \ c, \ d\in{\mathbb{R}}\) এবং \(a=c, \ b=d\) হয় তবে,
যোগের অনন্যতাঃ \(a+b=c+d\)
গুণনের অনন্যতাঃ \(a.b=c.d\)
যেমনঃ \(x, \ y, \ p, \ q\in{\mathbb{R}}\) হয় তবে,
\(x+y=p+y\) হলে, \(x=p\) এবং \(xy=xq\) হলে, \(y=q\)
অভেদকের অস্তিত্বঃ বাস্তব সংখ্যা \(\mathbb{R}\) যোগ \((+)\) এবং গুণন \((.)\) প্রক্রিয়ার জন্য অভেদকের অস্তিত্ব (law of existance of identity)।
যেকোনো \(a\in{\mathbb{R}}\) হয় তবে,
যোগের অভেদকঃ \(a+0=0+a\)
গুণনের অভেদকঃ \(a.1=1.a\)
যেমনঃ \(0\) এবং \(1\) কে যথাক্রমে যোগ \((+)\) এবং গুণন \((.)\) এর অভেদক বলে।
বিপরীতকের অস্তিত্বঃ বাস্তব সংখ্যা \(\mathbb{R}\) যোগ \((+)\) এবং গুণন \((.)\) প্রক্রিয়ার জন্য বিপরীতকের অস্তিত্ব (law of existance of inverse)।
যেকোনো \(a\in{\mathbb{R}}\) এর জন্য \(-a\in{\mathbb{R}}\) হয় তবে,
যোগের বিপরীতকঃ \(a+(-a)=(-a)+a=0\)
যেকোনো \(a\in{\mathbb{R}}\) এবং \(a\ne{0}\) এর জন্য \(a^{-1}\in{\mathbb{R}}\) হয় তবে,
গুণনের বিপরীতকঃ \(a.a^{-1}=a^{-1}a=1\)
যেমনঃ \(5, \ -5\in{\mathbb{R}}\) এবং \(5, \ 5^{-1}=\frac{1}{5}\in{\mathbb{R}}\)
\(5+(-5)=(-5)+5=0\) এবং \(5.5^{-1}=5^{-1}.5=1\)
অসমতা
Inequalities
অসমতাঃ অসমতা (Inequalities) এমন এক প্রকার গাণিতিক বাক্যের প্রকাশ (Mathematical Expression) যা সংখ্যা, পরিমাণ বা গাণিতিক বাক্যের ক্রমের সম্পর্ক (Order Relation) নির্দেশ করে।
গাণিতিকভাবে অসমতাকে \(\lt{}(less \ than), \ \gt{}(greater \ than), \ \le{}(less \ than \ or \ equal), \ \ge{}(greater \ than \ or \ equal)\) ইত্যাদি সম্পর্ক প্রতীক দ্বারা প্রকাশ করা হয়। \(2\gt{1}\) অথবা \(1\lt{2}\) এর অর্থ হচ্ছে \(2, \ 1\) থেকে বড় অথবা \(1, \ 2\) থেকে ছোট। আবার, \(x\gt{0}\) অসমতাটি \(x\) এর সকল ধনাত্মক মানের জন্য সত্য হলেও \(x^2\gt{0}\) অসমতাটি \(x=0\) ব্যতীত সকল বাস্তব মানের জন্য সত্য। অসমতা ও সমীকরণের মধ্যে অনেক বৈশিষ্ট্যের মিল বিদ্যমান থাকলেও অসমতার সমাধান নির্দিষ্ট কোনো সংখ্যা বা মানের জন্য স্থির না থেকে সমাধানের ব্যপ্তি নির্দেশ করে। অর্থাৎ নির্দিষ্ট সেটে বা অঞ্চলে বিদ্যমান সকল মানের জন্য অসমতা সিদ্ধ হয়। অসমতা গণিতে বিশেষ স্থান দখল করে আছে। যোগাশ্রয়ী প্রোগ্রাম গঠন, কোণের সম্পর্ক নির্ণয়, ত্রিভুজ ও চতুর্ভুজ সম্পর্কিত উপপাদ্য তথা গণিতের অনেক মৌলিক তথ্যাবলি অসমতার সাহায্যে ব্যাখ্যা করা হয়।
বাস্তব সংখ্যার অসমতা সম্পর্কিত স্বীকার্যসমূহ
Axioms of the real numbers related to inequality
স্বীকার্যঃ
সকল \(a, \ b\in{\mathbb{R}}\) এর জন্য
\(a\gt{b}\) বা \(a=b\) বা \(a\lt{b}\)
স্বীকার্যঃ
সকল \(a, \ b, \ c\in{\mathbb{R}}\) এবং \(a\gt{b}\) ও \(b\gt{c}\) এর জন্য
\(a\gt{c}\)

প্রমাণঃ
\(a, \ b\in{\mathbb{R}}\) এবং পরস্পর অসমান হলে,
\(a\lt{b}\) বা \(a\gt{b}\)
\(\Rightarrow a-b\gt{0}\)
\(\therefore a-b\gt{0} ....(1)\)
\(b, \ c\in{\mathbb{R}}\) এবং পরস্পর অসমান হলে,
\(b\lt{c}\) বা \(b\gt{c}\)
\(\Rightarrow b-c\gt{0}\)
\(\therefore b-c\gt{0} ....(2)\)
এখন, \(a-b+b-c\gt{0}\) ➜ \((1)\) ও \((2)\) যোগ করে,

\(\Rightarrow a-c\gt{0}\)
\(\Rightarrow a-c+c\gt{0+c}\) ➜ উভয় পার্শে \(c\) যোগ করে,

\(\Rightarrow a+(c-c)\gt{c}\)
\(\Rightarrow a+0\gt{c}\)
\(\therefore a\gt{c}\)
সকল \(a, \ b, \ c\in{\mathbb{R}}\) এবং \(a\lt{b}\) ও \(b\lt{c}\) এর জন্য
\(a\lt{c}\)

প্রমাণঃ
\(a, \ b\in{\mathbb{R}}\) এবং পরস্পর অসমান হলে,
\(a\gt{b}\) বা \(a\lt{b}\)
\(\Rightarrow a-b\lt{0}\)
\(\therefore a-b\lt{0} ....(1)\)
\(b, \ c\in{\mathbb{R}}\) এবং পরস্পর অসমান হলে,
\(b\gt{c}\) বা \(b\lt{c}\)
\(\Rightarrow b-c\lt{0}\)
\(\therefore b-c\lt{0} ....(2)\)
এখন, \(a-b+b-c\lt{0}\) ➜ \((1)\) ও \((2)\) যোগ করে,

\(\Rightarrow a-c\lt{0}\)
\(\Rightarrow a-c+c\lt{0+c}\) ➜ উভয় পার্শে \(c\) যোগ করে,

\(\Rightarrow a+(c-c)\lt{c}\)
\(\Rightarrow a+0\lt{c}\)
\(\therefore a\lt{c}\)
স্বীকার্যঃ
সকল \(a, \ b, \ c\in{\mathbb{R}}\) এবং \(a\gt{b}\) এর জন্য
\(a+c\gt{b+c}\) এবং \(a-c\gt{b-c}\)

প্রমাণঃ
\(a, \ b, \ c\in{\mathbb{R}}\) এবং \(a\gt{b}\) হলে,
\(\Rightarrow a\gt{b}\) হলে,
\(\Rightarrow a+c\gt{b+c}\) ➜ উভয় পার্শে \(c\) যোগ করে,

\(\therefore a+c\gt{b+c}\)
আবার, \(a, \ b, \ c\in{\mathbb{R}}\) এবং \(a\gt{b}\) হলে,
\(\Rightarrow a\gt{b}\)
\(\Rightarrow a+(-c)\gt{b+(-c)}\) ➜ উভয় পার্শে \((-c)\) যোগ করে,

\(\therefore a-c\gt{b-c}\)
স্বীকার্যঃ
সকল \(a, \ b, \ c\in{\mathbb{R}}\) এবং \(a\lt{b}\) এর জন্য
\(a+c\lt{b+c}\) এবং \(a-c\lt{b-c}\)

প্রমাণঃ
\(a, \ b, \ c\in{\mathbb{R}}\) এবং \(a\lt{b}\) হলে,
\(\Rightarrow a\lt{b}\) হলে,
\(\Rightarrow a+c\lt{b+c}\) ➜ উভয় পার্শে \(c\) যোগ করে,

\(\therefore a+c\lt{b+c}\)
আবার, \(a, \ b, \ c\in{\mathbb{R}}\) এবং \(a\lt{b}\) হলে,
\(\Rightarrow a\lt{b}\) হলে,
\(\Rightarrow a+(-c)\lt{b+(-c)}\) ➜ উভয় পার্শে \((-c)\) যোগ করে,

\(\therefore a-c\lt{b-c}\)
স্বীকার্যঃ
সকল \(a, \ b, \ c, \ d\in{\mathbb{R}};\) \(a\gt{b}\) এবং \(c\gt{d}\) এর জন্য
\(a+c\gt{b+d}\)

প্রমাণঃ
\(a, \ b, \ c, \ d\in{\mathbb{R}};\) \(a\gt{b}\) এবং \(c\gt{d}\) হলে,
ধরি, \(a\gt{b} ......(1)\)
এবং \(c\gt{d} ......(2)\)
\(\Rightarrow a+c\gt{b+d}\) ➜ \((1)\) ও \((2)\) যোগ করে,

\(\therefore a+c\gt{b+d}\)
স্বীকার্যঃ
সকল \(a, \ b, \ c\in{\mathbb{R}};\) \(a\gt{b}\) এবং \(c\gt{0}\) এর জন্য
\(ac\gt{bc}\) এবং \(\frac{a}{c}\gt{\frac{b}{c}}\)

প্রমাণঃ
\(a, \ b, \ c\in{\mathbb{R}};\) \(a\gt{b}\) এবং \(c\gt{0}\) হলে,
\(\Rightarrow a-b\gt{0}\) এবং \(c\gt{0}\)
ধরি, \(a-b\gt{0} ......(1)\)
এবং \(c\gt{0} ........(2)\)
\(\Rightarrow (a-b).c\gt{0}\) ➜ \((1)\) ও \((2)\) গুণ করে,

\(\Rightarrow a.c-b.c\gt{0}\)
\(\Rightarrow ac-bc\gt{0}\)
\(\Rightarrow ac-bc+bc\gt{0+bc}\) ➜ উভয় পার্শে \(bc\) যোগ করে,

\(\Rightarrow ac+(bc-bc)\gt{bc}\)
\(\Rightarrow ac+0\gt{bc}\)
\(\therefore ac\gt{bc}\)
আবার, \(a, \ b, \ c\in{\mathbb{R}};\) \(a\gt{b}\) এবং \(c\gt{0}\) হলে,
\(\Rightarrow a-b\gt{0}\) এবং \(\frac{1}{c}\gt{0}\)
ধরি, \(a-b\gt{0} ......(1)\)
এবং \(\frac{1}{c}\gt{0} ........(2)\)
\(\Rightarrow (a-b).\frac{1}{c}\gt{0}\) ➜ \((1)\) ও \((2)\) গুণ করে,

\(\Rightarrow a.\frac{1}{c}-b.\frac{1}{c}\gt{0}\)
\(\Rightarrow \frac{a}{c}-\frac{b}{c}\gt{0}\)
\(\Rightarrow \frac{a}{c}-\frac{b}{c}+\frac{b}{c}\gt{0+\frac{b}{c}}\) ➜ উভয় পার্শে \(\frac{b}{c}\) যোগ করে,

\(\Rightarrow \frac{a}{c}+\left(\frac{b}{c}-\frac{b}{c}\right)\gt{\frac{b}{c}}\)
\(\Rightarrow \frac{a}{c}+0\gt{\frac{b}{c}}\)
\(\therefore \frac{a}{c}\gt{\frac{b}{c}}\)
স্বীকার্যঃ
সকল \(a, \ b, \ c\in{\mathbb{R}};\) \(a\gt{b}\) এবং \(c\lt{0}\) এর জন্য
\(ac\lt{bc}\) এবং \(\frac{a}{c}\lt{\frac{b}{c}}\)

প্রমাণঃ
\(a, \ b, \ c\in{\mathbb{R}};\) \(a\gt{b}\) এবং \(c\lt{0}\) হলে,
\(\Rightarrow a-b\gt{0}\) এবং \(c\lt{0}\)
ধরি, \(a-b\gt{0} ......(1)\)
এবং \(c\lt{0} ........(2)\)
\(\Rightarrow (a-b).c\lt{0}\) ➜ \((1)\) ও \((2)\) গুণ করে,

\(\Rightarrow a.c-b.c\lt{0}\)
\(\Rightarrow ac-bc\lt{0}\)
\(\Rightarrow ac-bc+bc\lt{0+bc}\) ➜ উভয় পার্শে \(bc\) যোগ করে,

\(\Rightarrow ac+(bc-bc)\lt{bc}\)
\(\Rightarrow ac+0\lt{bc}\)
\(\therefore ac\lt{bc}\)
আবার, \(a, \ b, \ c\in{\mathbb{R}};\) \(a\gt{b}\) এবং \(c\lt{0}\) হলে,
\(\Rightarrow a-b\gt{0}\) এবং \(\frac{1}{c}\lt{0}\)
ধরি, \(a-b\gt{0} ......(1)\)
এবং \(\frac{1}{c}\lt{0} ........(2)\)
\(\Rightarrow (a-b).\frac{1}{c}\lt{0}\) ➜ \((1)\) ও \((2)\) গুণ করে,

\(\Rightarrow a.\frac{1}{c}-b.\frac{1}{c}\lt{0}\)
\(\Rightarrow \frac{a}{c}-\frac{b}{c}\lt{0}\)
\(\Rightarrow \frac{a}{c}-\frac{b}{c}+\frac{b}{c}\lt{0+\frac{b}{c}}\) ➜ উভয় পার্শে \(\frac{b}{c}\) যোগ করে,

\(\Rightarrow \frac{a}{c}+\left(\frac{b}{c}-\frac{b}{c}\right)\lt{\frac{b}{c}}\)
\(\Rightarrow \frac{a}{c}+0\lt{\frac{b}{c}}\)
\(\therefore \frac{a}{c}\lt{\frac{b}{c}}\)
স্বীকার্যঃ
সকল \(a, \ b\in{\mathbb{R}};\) \(a\gt{0},\) \(b\gt{0}\) এবং \(a\gt{b}\) এর জন্য
\(ab\gt{0}\) এবং \(\frac{1}{a}\lt{\frac{1}{b}}\)

প্রমাণঃ
\(a, \ b\in{\mathbb{R}};\) \(a\gt{0},\) \(b\gt{0}\) এবং \(a\gt{b}\) হলে,
ধরি, \(a\gt{0} ......(1)\)
এবং \(b\gt{0} ........(2)\)
\(\Rightarrow a.b\gt{0}\) ➜ \((1)\) ও \((2)\) গুণ করে,

\(\therefore ab\gt{0}\)
আবার, \(a, \ b\in{\mathbb{R}};\) \(a\gt{0},\) \(b\gt{0}\) এবং \(a\gt{b}\) হলে,
\(\Rightarrow ab\gt{0}\) এবং \(a\gt{b}\)
\(\therefore \frac{1}{ab}\gt{0}\) এবং \(a\gt{b}\)
এখন, \(a\gt{b}\)
\(\Rightarrow a.\frac{1}{ab}\gt{b.\frac{1}{ab}}\) ➜ উভয় পার্শে \(\frac{1}{ab}\) গুণ করে,

\(\Rightarrow \frac{1}{b}\gt{\frac{1}{a}}\)
\(\therefore \frac{1}{a}\lt{\frac{1}{b}}\)
স্বীকার্যঃ
সকল \(a, \ b\in{\mathbb{R}};\) \(a\gt{0},\) \(b\gt{0}\) এবং \(a\lt{b}\) এর জন্য
\(\frac{1}{a}\gt{\frac{1}{b}}\)

প্রমাণঃ
\(a, \ b\in{\mathbb{R}};\) \(a\gt{0},\) \(b\gt{0}\) এবং \(a\lt{b}\) হলে,
\(\Rightarrow ab\gt{0}\) এবং \(a\lt{b}\)
\(\therefore \frac{1}{ab}\gt{0}\) এবং \(a\lt{b}\)
এখন, \(a\lt{b}\)
\(\Rightarrow \frac{a}{ab}\lt{\frac{b}{ab}}\) ➜ উভয় পার্শে \(\frac{1}{ab}\) গুণ করে,

\(\Rightarrow \frac{1}{b}\lt{\frac{1}{a}}\)
\(\therefore \frac{1}{a}\gt{\frac{1}{b}}\)
বিশেষ দ্রষ্টব্যঃ \(a, \ b\in{\mathbb{R}}\) এর জন্য
\(a\ge{b}\) এবং \(a\le{b}\) কে দুর্বল অসমতা বলে। দুর্বল অসমতাও মৌলিক স্বীকার্য মেনে চলে।
স্বীকার্যঃ
সকল \(a\in{\mathbb{R}}\) এর জন্য
\(a^2\ge{0}\)
ব্যবধি
Interval
ব্যবধিঃ বাস্তব সংখ্যার বিশেষ ধরনের সেটকে ব্যবধি (Interval) বলা হয়। ব্যবধি দুই প্রকার।
যেমনঃ
সসীম ব্যবধি (Finite Interval)
অসীম ব্যবধি (Infinite Interval)
সসীম ব্যবধি
Finite Interval
সসীম ব্যবধিঃ \(a\) ও \(b\) বাস্তব সংখ্যা এবং \(a\lt{b}\) হলে, \(a\) ও \(b\) এর মধ্যবর্তী সকল বাস্তব সংখ্যার সেটকে সসীম ব্যবধি (Finite Interval) বলে।
খোলা ব্যবধিঃ কোনো বাস্তব চলক \(x\) এর মান \(a\) ও \(b\) ব্যতীত এদের মধ্যবর্তী সকল বাস্তব সংখ্যা হলে, ঐ ব্যবধিকে খোলা ব্যবধি (Open Interval) বলা হয়।
গাণিতিকভাবেঃ \((a, b)=\left\{x\in{\mathbb{R}}: a\lt{x}\lt{b}\right\}\)
সংখ্যারেখাঃ
realNumber
বদ্ধ ব্যবধিঃ কোনো বাস্তব চলক \(x\) এর মান \(a\) ও \(b\) সহ এদের মধ্যবর্তী সকল বাস্তব সংখ্যা হলে, ঐ ব্যবধিকে বদ্ধ ব্যবধি (Closed Interval) বলা হয়।
গাণিতিকভাবেঃ \([a, b]=\left\{x\in{\mathbb{R}}: a\le{x}\le{b}\right\}\)
সংখ্যারেখাঃ
realNumber
বদ্ধ-খোলা ব্যবধিঃ কোনো বাস্তব চলক \(x\) এর মান \(a\) সহ এবং \(b\) ব্যতীত এদের মধ্যবর্তী সকল বাস্তব সংখ্যা হলে, ঐ ব্যবধিকে বদ্ধ-খোলা ব্যবধি (Closed-Open Interval) বলা হয়।
গাণিতিকভাবেঃ \([a, b)=\left\{x\in{\mathbb{R}}: a\le{x}\lt{b}\right\}\)
সংখ্যারেখাঃ
realNumber
খোলা-বদ্ধ ব্যবধিঃ কোনো বাস্তব চলক \(x\) এর মান \(a\) ব্যতীত এবং \(b\) সহ এদের মধ্যবর্তী সকল বাস্তব সংখ্যা হলে, ঐ ব্যবধিকে খোলা-বদ্ধ ব্যবধি (Open-closed Interval) বলা হয়।
গাণিতিকভাবেঃ \((a, b]=\left\{x\in{\mathbb{R}}: a\lt{x}\le{b}\right\}\)
সংখ্যারেখাঃ
realNumber
অসীম ব্যবধি
Infinite Interval
অসীম ব্যবধিঃ যেকোনো বাস্তব সংখ্যা \(a\) হলে, \(a\) এর চেয়ে বড় ; কিংবা \(a\) এর চেয়ে ছোট সকল বাস্তব সংখ্যার সেটকে অসীম ব্যবধি (Infinite Interval) বলা হয়। সুতরাং বিন্দু \(a\) বিশিষ্ট চারটি অসীম ব্যবধি রয়েছে। যেমনঃ
বামে খোলা ডানে অসীম ব্যবধিঃ \((a, \infty)=\left\{x\in{\mathbb{R}}: x\gt{a}\right\}\)
সংখ্যারেখাঃ
realNumber
বামে বদ্ধ ডানে অসীম ব্যবধিঃ \([a, \infty)=\left\{x\in{\mathbb{R}}: x\ge{a}\right\}\)
সংখ্যারেখাঃ
realNumber
বামে অসীম ডানে খোলা ব্যবধিঃ \((-\infty, a)=\left\{x\in{\mathbb{R}}: x\lt{a}\right\}\)
সংখ্যারেখাঃ
realNumber
বামে অসীম ডানে বদ্ধ ব্যবধিঃ \((-\infty, a]=\left\{x\in{\mathbb{R}}: x\le{a}\right\}\)
সংখ্যারেখাঃ
realNumber
বাস্তব সংখ্যার সম্পূর্ণতা ধর্ম
Property of completeness of \(\mathbb{R}\)
ঊর্ধেব সীমিত সেটঃ বাস্তব সংখ্যার কোনো সেট \(S\) কে ঊর্ধেব সীমিত (Bounded above) বলা হয়; যদি একটি বাস্তব সংখ্যা \(M\) থাকে যেন তা একটি বাস্তব সংখ্যার অশূন্য (Non empty) উপসেট \(S\) এর যেকোনো উপাদানের সমান অথবা \(S\) এর যেকোনো উপাদান অপেক্ষা বৃহত্তর হয় (অর্থাৎ সকল \(s\in{S}\) এর জন্য \(M\ge{s}\)) তাহলে \(M\) হলো \(S\) উপসেটের একটি ঊর্ধবসীমা। \(M\) এর চেয়ে বড় যেকোনো সংখ্যা \(S\) এর একটি ঊর্ধবসীমা।
যেমনঃ \(S=\left\{... ...., -3, \ -2, \ -1, \ 0, \ 1, \ 2, \ 3\right\}\) একটি ঊর্ধেব সীমিত সেট। \(3, \ 4, \ 5, \ ...\) ইত্যাদি সেটটির ঊর্ধবসীমা।
ক্ষুদ্রতম ঊর্ধবসীমাঃ ঊর্ধেব সীমিত সেটের ঊর্ধবসীমাগুলোর মধ্যে ক্ষুদ্রতমটিকে ক্ষুদ্রতম ঊর্ধবসীমা (Supremum or Least Supper Bound) বা লঘিষ্ঠ ঊর্ধবসীমা বলে। \(S\) এর ক্ষুদ্রতম ঊর্ধবসীমাকে \(\left(Sup(S)\right)\) দ্বারা প্রকাশ করা হয়।
যেমনঃ \(S=\left\{... ...., -3, \ -2, \ -1, \ 0, \ 1, \ 2, \ 3\right\}\) একটি ঊর্ধেব সীমিত সেট। \(3, \ 4, \ 5, \ ...\) ইত্যাদি সেটটির ঊর্ধবসীমা।
এখানে ক্ষুদ্রতম ঊর্ধবসীমা \(3\) ।
সুতরাং \(Sup(S)=3\)।
নিম্নে সীমিত সেটঃ বাস্তব সংখ্যার কোনো সেট \(S\) কে নিম্নে সীমিত (Bounded below) বলা হয়; যদি একটি বাস্তব সংখ্যা \(m\) থাকে যেন তা একটি বাস্তব সংখ্যার উপসেট \(S\) এর যেকোনো উপাদানের সমান অথবা \(S\) এর যেকোনো উপাদান অপেক্ষা ক্ষুদ্রতর হয় (অর্থাৎ সকল \(p\in{S}\) এর জন্য \(m\ge{p}\)) তাহলে \(m\) হলো \(S\) উপসেটের একটি নিম্নসীমা। \(m\) এর চেয়ে ছোট যেকোনো সংখ্যা \(S\) এর একটি নিম্নসীমা।
যেমনঃ \(S=\left\{2, \ 3, \ 4, \ 5, \ .......\right\}\) একটি নিম্নে সীমিত সেট। \(2, \ 1, \ 0, \ -1, \ -2, \ -3, \ ...\) ইত্যাদি সেটটির নিম্নসীমা।
বৃহত্তম নিম্নসীমাঃ নিম্নে সীমিত সেটের নিম্নসীমাগুলোর মধ্যে বৃহত্তমটিকে বৃহত্তম নিম্নসীমা (Infimum or Greatest Lower Bound) বা গরিষ্ঠ নিম্নসীমা বলে। \(S\) এর বৃহত্তম নিম্নসীমাকে \(Inf(S)\) দ্বারা প্রকাশ করা হয়।
যেমনঃ \(S=\left\{2, \ 3, \ 4, \ 5, \ .......\right\}\) একটি নিম্নে সীমিত সেট। \(2, \ 1, \ 0, \ -1, \ -2, \ -3, \ ...\) ইত্যাদি সেটটির নিম্নসীমা।
এখানে বৃহত্তম নিম্নসীমা \(2\) ।
সুতরাং \(Inf(S)=2\)।
সীমিত সেটঃ যদি বাস্তব সংখ্যার একটি উপসেট \(S\) ঊর্ধেবসীমিত এবং নিম্নেসীমিত উভয় ধরনের হয়, তবে \(S\) কে সীমিত সেট (Bounded set) বলা হয়।
যেমনঃ \(S=\left\{2, \ 3, \ 4, \ 5, \ 6\right\}\) একটি সীমিত সেট।
অসীমিত সেটঃ যে সেট সীমিত নয় তাকে অসীমিত সেট (Unbounded set) বলা হয়।
যেমনঃ \(S=\left\{2, \ 3, \ 4, \ 5, \ 6, \ .......\right\}\) একটি অসীমিত সেট।
বাস্তব সংখ্যার সম্পূর্ণতা স্বীকার্য
Axioms of completeness of \(\mathbb{R}\)
বাস্তব সংখ্যার প্রত্যেক অশূন্য ঊর্ধেব সীমিত (Bounded above) উপসেট একটি (অনন্য) লঘিষ্ঠ ঊর্ধবসীমা বিদ্যমান যা একটি বাস্তব সংখ্যা।
বাস্তব সংখ্যার প্রত্যেক অশূন্য নিম্নে সীমিত (Bounded below) উপসেট একটি (অনন্য) গরিষ্ঠ নিম্নসীমা বিদ্যমান যা একটি বাস্তব সংখ্যা।
দ্রষ্টব্যঃ মূলদ সংখ্যার সেটে সম্পূর্ণতার ধর্ম খাটে না।
যেমনঃ ধরি, মূলদ সংখ্যার একটি উপসেট \(S=\left\{x\in{\mathbb{Q}}: x\lt{0} \text{ এবং} \ x^2\lt{2}\right\}\)।
যেহেতু \(1\in{S}\), সুতরাং \(S\) ফাঁকা সেট নয়।
যেহেতু \(2^2\gt{2}\)।
\(\therefore S\) একটি ঊর্ধেবসীমিত সেট।
অর্থাৎ \(S\) একটি অশূন্য ঊর্ধেবসীমিত সেট।
\(\therefore S\) এর লঘিষ্ঠ ঊর্ধবসীমা \(\sqrt{2},\) যা মূলদ সংখ্যা নয়।
অর্থাৎ মূলদ সংখ্যার সেটে সম্পূর্ণতার ধর্ম খাটে না।
পরম মান
Absolute value
পরম মানঃ সংখ্যারেখায় মূলবিন্দু (\(0\) নির্দেশক বিন্দু) এবং সংখ্যা নির্দেশক বিন্দুর মধ্যবর্তী দূরত্বকে সংখ্যাটির পরমমান (Absolute value) বলা হয়।
যেমনঃ সংখ্যারেখায় মূলবিন্দু থেকে \(-4\) এর দূরত্ব \(4\) এবং \(4\) এর দূরত্ব \(4\) একক। অর্থাৎ \(-4\) এর পরমমান \(4\) এবং \(4\) এর পরমমান \(4\) ।
সংখ্যারেখাঃ
realNumber
সুতরাং সকল ধনাত্মক সংখ্যার পরমমান সংখ্যাগুলির সমান, সকল ঋনাত্মক সংখ্যার পরমমান সংখ্যাগুলির বিপরীত চিহ্নবিশিষ্ট এবং \(0\) এর পরমমান \(0\)।
যেকোনো বাস্তব সংখ্যা \(x\) এর পরমমান \(|x|\) দ্বারা সূচিত হয় এবং
\(|x|=\begin{cases} \ \ \ x, & \text{যখন} \ x\gt{0} \\ \ \ \ 0, & \text{যখন} \ x =0 \\-x, & \text{যখন} \ x \lt{0}\end{cases}\) \(\Rightarrow\) \(|x|=\begin{cases} \ \ \ x, & \text{যখন} \ x\ge{0} \\-x, & \text{যখন} \ x \lt{0}\end{cases}\)
অর্থাৎ শূন্য \((0)\) ব্যতীত সকল বাস্তব সংখ্যার পরমমান ধনাত্মক এবং শূন্য \((0)\) এর পরমমান শূন্য \((0)\) হবে।
সকল \(x\in{\mathbb{R}}\) এর জন্য,
\(|x|=\sqrt{x}\)

প্রমাণঃ
আমরা জানি,
\(|x|^2=x^2\) যা \(x\) এর সকল ধ্নাত্মক, ঋনাত্মক ও শূন্যের জন্য সত্য।
\(\Rightarrow |x|=\pm\sqrt{x^2}\)
যেহেতু \(|x|\ge{0}\) কাজেই ঋনাত্মক মান বর্জন করে,
\(\therefore |x|=\sqrt{x^2}\)
বিশেষ দ্রষ্টব্যঃ যেকোনো বাস্তব সংখ্যার পরমমান শূন্য অপেক্ষা বৃহত্তর বা শূন্যের সমান।
পরম মানের বৈশিষ্ট্যসমূহ এবং এদের প্রমাণ
Properties of absolute value and its proof
সকল \(a\in{\mathbb{R}}\) এর জন্য,
\(|a|\ge{0}\)

প্রমাণঃ
সকল \(a\in{\mathbb{R}}\) এর জন্য,
\(a=0\) হলে,
\(\Rightarrow |a|=|0|\)
\(\therefore |a|=0 ......(1)\) ➜ \(\because |0|=0\)

আবার, সকল \(a\in{\mathbb{R}}\) এর জন্য,
\(a\gt{0}\) হলে,
\(\Rightarrow |a|=a\)
\(\therefore |a|\gt{0} ......(2)\)
আবার, সকল \(a\in{\mathbb{R}}\) এর জন্য,
\(a\lt{0}\) হলে,
\(\Rightarrow |a|=-a\)
আবার, \(a\lt{0}\) হলে,
\(\Rightarrow -a\gt{0}\)
\(\therefore |a|\gt{0}\) যা \((2)\) এর অনুরূপ ➜ \(\because |a|=-a\)

\((1)\) ও \((2)\) হতে,
\(|a|\ge{0}\)
সকল \(a\in{\mathbb{R}}\) এর জন্য,
\(|a|\ge{a}\)

প্রমাণঃ
সকল \(a\in{\mathbb{R}}\) এর জন্য,
\(a\ge{0}\) হলে,
\(|a|=a ......(1)\) ➜ পরমমানের সংজ্ঞানুসারে,

সকল \(a\in{\mathbb{R}}\) এর জন্য,
\(a\lt{0}\) হলে,
\(\Rightarrow |a|=-a\)
আবার, সকল \(a\in{\mathbb{R}}\) এর জন্য,
\(a\lt{0}\) হলে,
\(\Rightarrow -a\gt{0}\)
\(\Rightarrow -a\gt{0}\gt{a}\)
\(\Rightarrow -a\gt{a}\)
\(\therefore |a|\gt{a} ......(2)\) ➜ \(\because |a|=-a\)

\((1)\) ও \((2)\) হতে,
\(|a|\ge{a}\)
সকল \(a\in{\mathbb{R}}\) এর জন্য,
\(|a|\ge{-a}\)

প্রমাণঃ
সকল \(a\in{\mathbb{R}}\) এর জন্য,
\(a\ge{0}\) হলে,
\(|a|=a ......(1)\) ➜ পরমমানের সংজ্ঞানুসারে,

আবার, সকল \(a\in{\mathbb{R}}\) এর জন্য,
\(a\ge{0}\) হলে,
\(\Rightarrow -a\le{0}\)
\(\Rightarrow 0\ge{-a}\)
\(\Rightarrow a\ge{0}\ge{-a}\)
\(\therefore a\ge{-a} ......(2)\)
\((1)\) ও \((2)\) হতে,
\(|a|\ge{-a} ......(3)\)
আবার, সকল \(a\in{\mathbb{R}}\) এর জন্য,
\(a\lt{0}\) হলে,
\(\therefore |a|=-a .....(4)\)
\((3)\) ও \((4)\) হতে,
\(|a|\ge{-a}\)
সকল \(a\in{\mathbb{R}}\) এর জন্য,
\(-|a|\le{a}\le{|a|}\)
কুঃ ২০১১ ।

প্রমাণঃ
সকল \(a\in{\mathbb{R}}\) এর জন্য,
\(|a|\ge{a}\) হলে,
\(\therefore a\le{|a|} ......(1)\)
আবার, সকল \(a\in{\mathbb{R}}\) এর জন্য,
\(|a|\ge{-a}\)
\(\therefore -|a|\le{a} ......(2)\) ➜ উভয় পার্শে \(-1\) গুণ করে,

\((1)\) ও \((2)\) হতে,
\(-|a|\le{a}\le{|a|}\)
সকল \(a\in{\mathbb{R}}\) এর জন্য,
\(|a|^2=a^2\) বা \(|a|=\sqrt{a^2}\)

প্রমাণঃ
সকল \(a\in{\mathbb{R}}\) এর জন্য,
\(a\gt{0}\) হলে,
\(\Rightarrow |a|=a\)
\(\therefore |a|^2=a^2\) ➜ উভয় পার্শে বর্গ করে,

আবার, সকল \(a\in{\mathbb{R}}\) এর জন্য,
\(a\lt{0}\) হলে,
\(\Rightarrow |a|=-a\)
\(\therefore |a|^2=a^2\) ➜ উভয় পার্শে বর্গ করে,

\(\therefore |a|^2=a^2\)
\(\Rightarrow |a|=\pm\sqrt{a^2}\)
\(\therefore |a|=\sqrt{a^2}\) ➜ পরমমানের বর্গমূল ঋনাত্মক হতে পারে না,

সকল \(a, \ b\in{\mathbb{R}}\) এর জন্য,
\(|ab|=|a||b|\)
সিঃ ২০১০; রাঃ ২০০৯ ।

প্রমাণঃ
আমরা জানি,
সকল \(x\in{\mathbb{R}}\) এর জন্য,
\(|x|^2=x^2\)
\(\Rightarrow |ab|^2=(ab)^2\) ➜ \(x\) এর পরিবর্তে \(ab\) বসিয়ে,

\(\Rightarrow |ab|^2=a^2b^2\)
\(\Rightarrow |ab|^2=|a|^2|b|^2\) ➜ \(\because |x|^2=x^2\)

\(\Rightarrow |ab|^2=\left(|a||b|\right)^2\)
\(\therefore |ab|=|a||b|\) ➜ \(\because |x|^2=x^2\)
\(\Rightarrow |x|=x\)

সকল \(a, \ b\in{\mathbb{R}}\) এর জন্য,
\(\left|\frac{a}{b}\right|=\frac{|a|}{|b|}\)

প্রমাণঃ
আমরা জানি,
সকল \(x\in{\mathbb{R}}\) এর জন্য,
\(|x|^2=x^2\)
\(\Rightarrow \left|\frac{a}{b}\right|^2=\left(\frac{a}{b}\right)^2\) ➜ \(x\) এর পরিবর্তে \(\frac{a}{b}\) বসিয়ে,

\(\Rightarrow \left|\frac{a}{b}\right|^2=\frac{a^2}{b^2}\)
\(\Rightarrow \left|\frac{a}{b}\right|^2=\frac{|a|^2}{|b|^2}\) ➜ \(\because |x|^2=x^2\)

\(\Rightarrow \left|\frac{a}{b}\right|^2=\left(\frac{|a|}{|b|}\right)^2\)
\(\therefore \left|\frac{a}{b}\right|=\frac{|a|}{|b|}\) ➜ \(\because |x|^2=x^2\)
\(\Rightarrow |x|=x\)

সকল \(a, \ b\in{\mathbb{R}}\) এর জন্য,
\(|a+b|\le{|a|+|b|}\)
বঃ ২০১২,২০১০,২০০৬,২০০৩; কুঃ ২০১৪,২০১২,২০১০,২০০৮,২০০৫; ঢাঃ ২০১৩,২০০৮,২০০৬,২০০৪; রাঃ ২০১৩,২০১১,২০০৭,২০০৫,২০০৩; দিঃ ২০১২,২০১০; সিঃ ২০০৮,২০০২; যঃ ২০১৩,২০০৭,২০০৪,২০০১; মাঃ ২০০৬,২০০৪; চঃ ২০১৩,২০০৭,২০০৩ ।

প্রমাণঃ
আমরা জানি,
সকল \(a, \ b\in{\mathbb{R}}\) এর জন্য,
\(ab\le{|ab|}\)
\(\Rightarrow ab\le{|a||b|}\) ➜ \(\because |ab|=|a||b|\)

\(\Rightarrow 2ab\le{2|a||b|}\) ➜ উভয় পার্শে \(2\) গুণ করে,

\(\Rightarrow |a|^2+2ab+|b|^2\le{|a|^2+2|a||b|+|b|^2}\) ➜ উভয় পার্শে \(|a|^2+|b|^2\) যোগ করে,

\(\Rightarrow a^2+2ab+b^2\le{|a|^2+2|a||b|+|b|^2}\) ➜ \(\because |x|^2=x^2\)

\(\Rightarrow (a+b)^2\le{(|a|+|b|)^2}\) ➜ \(\because x^2+2xy+y^2=(x+y)^2\)

\(\Rightarrow |a+b|^2\le{(|a|+|b|)^2}\) ➜ \(\because x^2=|x|^2\)

\(\therefore |a+b|\le{|a|+|b|}\) ➜ \(\because |x|^2=x^2\)
\(\Rightarrow |x|=x\)

সকল \(a, \ b\in{\mathbb{R}}\) এর জন্য,
\(|a-b|\le{|a|+|b|}\)

প্রমাণঃ
আমরা জানি,
\(|a-b|=|a+(-b)|\)
\(\Rightarrow |a-b|\le{|a|+|(-b)|}\) ➜ \(\because |a+b|\le{|a|+|b|}\)

\(\therefore |a-b|\le{|a|+|b|}\) ➜ \(\because |(-x)|=|x|\)

সকল \(a, \ b\in{\mathbb{R}}\) এর জন্য,
\(|a-b|\ge{|a|-|b|}\)

প্রমাণঃ
আমরা জানি,
সকল \(a, \ b\in{\mathbb{R}}\) এর জন্য,
\(ab\le{|ab|}\)
\(\Rightarrow ab\le{|a||b|}\) ➜ \(\because |ab|=|a||b|\)

\(\Rightarrow -2ab\ge{-2|a||b|}\) ➜ উভয় পার্শে \(-2\) গুণ করে,

\(\Rightarrow |a|^2-2ab+|b|^2\ge{|a|^2-2|a||b|+|b|^2}\) ➜ উভয় পার্শে \(|a|^2+|b|^2\) যোগ করে,

\(\Rightarrow a^2-2ab+b^2\ge{|a|^2-2|a||b|+|b|^2}\) ➜ \(\because |x|^2=x^2\)

\(\Rightarrow (a-b)^2\ge{(|a|-|b|)^2}\) ➜ \(\because x^2-2xy+y^2=(x-y)^2\)

\(\Rightarrow |a-b|^2\ge{\left||a|-|b|\right|^2}\) ➜ \(\because x^2=|x|^2\)

\(\therefore |a-b|\ge{|a|-|b|}\) ➜ \(\because |x|^2=x^2\)
\(\Rightarrow |x|=x\)

\(|a-b|\ge{||a|-|b||}\)

প্রমাণঃ
আমরা জানি,
সকল \(a, \ b\in{\mathbb{R}}\) এর জন্য,
\(|a+b|\le{|a|+|b|} .........(1)\)
আবার, \(|a|\le{|a-b+b|}\)
\(\Rightarrow |a|\le{|a-b|+|b|}\) ➜ \((1)\) হতে,
\(\because |a+b|\le{|a|+|b|}\)

\(\therefore |a|-|b|\le{|a-b|} ........(2)\)
আবার, \(|b|-|a|\le{|b-a|}\)
\(\Rightarrow |b|-|a|\le{|a-b|}\) ➜ \(\because |b-a|=|a-b|\)

\(\Rightarrow -(|b|-|a|)\ge{-|a-b|}\) ➜ উভয় পার্শে \(-1\) গুণ করে,

\(\Rightarrow -|b|+|a|\ge{-|a-b|}\)
\(\Rightarrow |a|-|b|\ge{-|a-b|}\)
\(\therefore -|a-b|\le{|a|-|b|} .....(3)\)
\((2)\) ও \((3)\) হতে,
\(\Rightarrow -|a-b|\le{|a|-|b|}\le{|a-b|}\)
\(\Rightarrow \left||a|-|b|\right|\le{|a-b|}\) ➜ \(\because -p\le{x}\le{p}\)
\(\Rightarrow |x|\le{p}\)

\(\therefore |a-b|\ge{\left||a|-|b|\right|}\)
সকল \(a, \ b\in{\mathbb{R}}\) এর জন্য,
\(|a+b|\ge{|a|-|b|}\)

প্রমাণঃ
আমরা জানি,
সকল \(a, \ b\in{\mathbb{R}}\) এর জন্য,
\(|ab|\ge{-ab}\)
\(\Rightarrow -2|ab|\le{2ab}\) ➜ উভয় পার্শে \(-2\) গুণ করে,

\(\Rightarrow |a|^2-2|ab|+|b|^2\le{|a|^2+2ab+|b|^2}\) ➜ উভয় পার্শে \(|a|^2+|b|^2\) যোগ করে,

\(\Rightarrow |a|^2-2|a|.|b|+|b|^2\le{a^2+2ab+b^2}\) ➜ \(\because |x|^2=x^2\)

\(\Rightarrow a^2+2ab+b^2\ge{|a|^2-2|a|.|b|+|b|^2}\)
\(\Rightarrow (a+b)^2\ge{(|a|-|b|)^2}\) ➜ \(\because x^2+2xy+y^2=(x-y)^2\)
এবং \(x^2-2xy+y^2=(x-y)^2\)

\(\Rightarrow |a+b|^2\ge{\left||a|-|b|\right|^2}\) ➜ \(\because x^2=|x|^2\)

\(\therefore |a+b|\ge{|a|-|b|}\) ➜ \(\because |x|^2=x^2\)
\(\Rightarrow |x|=x\)

সকল \(a, \ x\in{\mathbb{R}}\) এর জন্য, \(a\ge{0}\) এবং \(|x|\le{a}\) হলে,
\(-a\le{x}\le{a}\)

প্রমাণঃ
\(x\ge{0}\) এবং \(|x|\le{a}\) হলে,
\(x\le{a} .......(1)\)
আবার, \(x\lt{0}\) এবং \(|x|\le{a}\)
\(\Rightarrow -x\le{a}\)
\(\Rightarrow x\ge{-a}\) ➜ উভয় পার্শে \(-1\) গুণ করে,

\(\therefore -a\le{x} .....(2)\)
\((1)\) ও \((2)\) হতে,
\(-a\le{x}\le{a}\)
সকল \(a, \ x\in{\mathbb{R}}\) এর জন্য, \(a\gt{0}\) এবং \(|x|\lt{a}\) হলে,
\(-a\lt{x}\lt{a}\)

প্রমাণঃ
\(x\gt{0}\) এবং \(|x|\lt{a}\) হলে,
\(x\lt{a} .......(1)\)
আবার, \(x\lt{0}\) এবং \(|x|\lt{a}\)
\(\Rightarrow -x\lt{a}\)
\(\Rightarrow x\gt{-a}\) ➜ উভয় পার্শে \(-1\) গুণ করে,

\(\therefore -a\lt{x} .....(2)\)
\((1)\) ও \((2)\) হতে,
\(-a\lt{x}\lt{a}\)
সকল \(a, \ x\in{\mathbb{R}}\) এর জন্য, \(|x|\ge{a}\) হলে,
\(x\le{-a}\) অথবা \(x\ge{a}\)
যেখানে, \(a\ge{0}\)

প্রমাণঃ
\(|x|\ge{a} .......(1)\)
এখন, \(x\ge{0}\) হলে,
\((1)\) নং হতে পাই,
\(x\ge{a}\)
আবার, \(x\lt{0}\) হলে,
\((1)\) নং হতে পাই,
\(-x\ge{a}\)
\(\Rightarrow x\le{-a}\) ➜ উভয় পার্শে \(-1\) গুণ করে,

সুতরাং \(|x|\ge{a}\) হলে, \(x\le{-a}\) অথবা \(x\ge{a}\)
সকল \(a, \ x\in{\mathbb{R}}\) এর জন্য, \(|x|\gt{a}\) হলে,
\(x\lt{-a}\) অথবা \(x\gt{a}\)
যেখানে, \(a\gt{0}\)

প্রমাণঃ
\(|x|\gt{a} .......(1)\)
এখন, \(x\gt{0}\) হলে,
\((1)\) নং হতে পাই,
\(x\gt{a}\)
আবার, \(x\lt{0}\) হলে,
\((1)\) নং হতে পাই,
\(-x\gt{a}\)
\(\Rightarrow x\lt{-a}\) ➜ উভয় পার্শে \(-1\) গুণ করে,

সুতরাং \(|x|\gt{a}\) হলে, \(x\lt{-a}\) অথবা \(x\gt{a}\)
এক চলক সম্বলিত অসমতা
Inequalities of one variable
এক চলক সম্বলিত অসমতারঃ এক চলক সম্বলিত বাক্য যার একটি রাশি অপর একটি রাশির চেয়ে ছোট অথবা বড়, ছোট বা সমান, বড় বা সমান অথবা কোনোটিই নয় এরূপ বাক্যকে এক চলক সম্বলিত অসমতা (Inequalities of one variable) বলে।
যেমনঃ \(x\gt{5}, \ x\lt{5}, \ x\ngtr{5}, \ x\nless{5}, \ x\ge{5}, \ x\le{5}, \ x\ngeq{5}, \ x\nleq{5}\) ইত্যাদি।
যৌগিক অসমতাঃ একাধিক বাক্য সমন্বিত অসমতাকে যৌগিক অসমতা (Compound inequalities) বলা হয়।
যেমনঃ \(a\gt{x}\gt{b}\) একটি যৌগিক অসমতা।
কারণ এখানে একটি অসমতা \(a\gt{x}\) এবং অপরটি \(x\gt{b}\)।
এক চলক সম্বলিত অসমতার সমাধান
Solution of inequalities with one variable
যে অসমতার মধ্যে কেবল একটি চলক বিধ্যমান তাকে এক চলক সম্বলিত অসমতা বলে। এক চলক সম্বলিত অসমতাকে দুই ভাগে বিভক্ত করা যায়।
শর্তাধীন অসমতাঃ যে সমস্ত অসমতা সম্পর্কযুক্ত চলকের নির্দিষ্ট কিছু মানের জন্য সত্য তাকে শর্তাধীন অসমতা (Conditional inequalities) বলা হয়।
যেমনঃ \(x+5\gt{7}\) একটি শর্তাধীন অসমতা।
কারণ এটি কেবল \(x\gt{2}\) এর জন্য সত্য।
শর্তহীন অসমতাঃ যে সমস্ত অসমতা সম্পর্কযুক্ত চলকের প্রত্যেক মানের জন্য সত্য তাকে শর্তহীন অসমতা (Unconditional inequalities) বলা হয়।
যেমনঃ \(x+5\gt{x}\) একটি শর্তহীন অসমতা।
কারণ এটি \(x\) এর প্রত্যেক মানের জন্য সত্য।
এক চলক সম্বলিত দ্বিঘাত অসমতার সমাধান
Solution of quadratic inequality with one variable
\(a\gt{b}\) হলে, \((x-a)(x-b)\lt{0}, \ \frac{x-a}{x-b}\lt{0}, \ \frac{x-b}{x-a}\lt{0}\) এবং \(\frac{1}{(x-a)(x-b)}\lt{0}\) এর সমাধানঃ
\(b\lt{x}\lt{a}\)

প্রমাণঃ
দেওয়া আছে,
\(a\gt{b}\) এবং \((x-a)(x-b)\lt{0}\)
এখন, \((x-a)(x-b)\lt{0}\)
\(\Rightarrow x-a\gt{0}, \ x-b\lt{0}\) অথবা, \(x-a\lt{0}, \ x-b\gt{0}\)
\(\Rightarrow x\gt{a}, \ x\lt{b}\) অথবা, \(x\lt{a}, \ x\gt{b}\)
\(\Rightarrow x\lt{a}, \ x\gt{b}\) \(\because a\gt{b}\)
\(\Rightarrow x\lt{a}, \ b\lt{x}\)
\(\therefore b\lt{x}\lt{a}\)
\(a\gt{b}\) হলে, \((x-a)(x-b)\gt{0}, \ \frac{x-a}{x-b}\gt{0}, \ \frac{x-b}{x-a}\gt{0}\) এবং \(\frac{1}{(x-a)(x-b)}\gt{0}\) এর সমাধানঃ
\(x\lt{b}\) অথবা \(x\gt{a}\)

প্রমাণঃ
দেওয়া আছে,
\(a\gt{b}\) এবং \((x-a)(x-b)\gt{0}\)
এখন, \((x-a)(x-b)\gt{0}\)
\(\Rightarrow x-a\gt{0}, \ x-b\gt{0}\) অথবা, \(x-a\lt{0}, \ x-b\lt{0}\)
\(\Rightarrow x\gt{a}, \ x\gt{b}\) অথবা, \(x\lt{a}, \ x\lt{b}\)
\(\Rightarrow x\gt{a}\) অথবা, \(x\lt{b}\) \(\because a\gt{b}\)
\(\therefore x\lt{b}\) অথবা, \(x\gt{a}\)
লেখচিত্রের সাহায্যে এক চলক সম্বলিত দ্বিঘাত অসমতার সমাধান
Solution of quadratic inequality with one variable with the help of graph
লেখচিত্রের সাহায্যে এক চলক সম্বলিত দ্বিঘাত অসমতার সমাধানের জন্য নিচের ধাপগুলি অনুসরণীয়।
প্রথমে সংশ্লিষ্ট সমীকরণের সমাধান করতে হবে।
পরবর্তীতে দ্বিঘাত ফাংশনের লেখচিত্র অঙ্কন করতে হবে।
দ্রষ্টব্যঃ \(ax^2+bx+c=0\) এ \(a\gt{0}\) হলে, পরাবৃত্তের আকার হবে \(\cup\) এবং \(a\lt{0}\) হলে পরাবৃত্তের আকার হবে \(\cap\)
অসমতাটি ঋনাত্মক হলে, সমাধান হবে \(x\) অক্ষের সাথে পরাবৃত্তের ছেদকের মধ্যস্থ সরলরেখা এবং অসমতাটি ধনাত্মক হলে, সমাধান হবে \(x\) অক্ষের সাথে পরাবৃত্তের ছেদকের বাইরের সরলরেখাদ্বয়।
উদাহরণসমুহ
\(Ex.1.\) দেখাও যে, পূর্ণ বর্গ নয় এরূপ ধনাত্মক পূর্ণ সংখ্যার বর্গমূল অমূলদ সংখ্যা।

সমাধান করঃ
\(Ex.2.(a)\) \(3x-2\gt{2x-1}\)
উত্তরঃ \(x\gt{1}\)

সমাধান করঃ
\(Ex.2.(b)\) \(x-9\gt{3x+1}\)
উত্তরঃ \(x\lt{5}\)

\(Ex.2.(c)\) \(x^2-2x\gt{0}\)
উত্তরঃ \(S=\left\{x\in{\mathbb{R}}: x\lt{0} \text{ অথবা} \ x\gt{2}\right\}\)

লেখচিত্র অঙ্কন করঃ
\(Ex.3.(a)\) \(2x+y-4=0\)

লেখচিত্র অঙ্কন করঃ
\(Ex.3.(b)\) \(2x+3y+4\gt{0}\)

\(Ex.4.\) সমাধান করঃ \(x\le{\frac{1}{2}x+1}\) এবং সমাধান সেট সংখ্যারেখায় দেখাও।
উত্তরঃ \(S=\left\{x\in{\mathbb{R}}:x\le{2}\right\}\)
সংখ্যারেখাঃ
realNumber

\(Ex.5.\) সমাধান করঃ \(x\le{\frac{x}{3}+4}\) এবং সমাধান সেট সংখ্যারেখায় দেখাও।
উত্তরঃ \(S=\left\{x\in{\mathbb{R}}:x\le{6}\right\}\)
সংখ্যারেখাঃ
realNumber

\(Ex.6.\) সমাধান করঃ \(\frac{x(x+1)}{x-2}\gt{0}\) এবং সমাধান সেট সংখ্যারেখায় দেখাও।
উত্তরঃ \(S=\left\{x\in{\mathbb{R}}:-1\lt{x}\lt{0}\right\}\cup\left\{x\in{\mathbb{R}}:x\gt{2}\right\}\)
সংখ্যারেখাঃ
realNumber

সমাধান কর এবং সমাধান সেট সংখ্যারেখায় দেখাও।
\(Ex.7.(a)\) \(x\left(\frac{x-4}{x-5}\right)\lt{0}\)
উত্তরঃ \(S=\left\{x\in{\mathbb{R}}: x\lt{0}\right\}\cup\left\{x\in{\mathbb{R}}: 4\lt{x}\lt{5}\right\}\)
সংখ্যারেখাঃ
realNumber

সমাধান কর এবং সমাধান সেট সংখ্যারেখায় দেখাও।
\(Ex.7.(b)\) \(x^2-x-6\gt{0}\)
উত্তরঃ \(S=\left\{x\in{\mathbb{R}}: x\lt{-2} \text{ অথবা} \ x\gt{3}\right\}\)
সংখ্যারেখাঃ
realNumber

\(Ex.7.(c)\) \(x^2-4x-5\gt{0}\)
উত্তরঃ \(S=\left\{x\in{\mathbb{R}}: x\lt{-1} \text{ অথবা} \ x\gt{5}\right\}\)
সংখ্যারেখাঃ
realNumber

\(Ex.7.(d)\) \(x^2-5x+6\lt{0}\)
উত্তরঃ \(S=\left\{x\in{\mathbb{R}}: 2\lt{x}\lt{3}\right\}\)
সংখ্যারেখাঃ
realNumber

\(Ex.7.(e)\) \(x^2-4x+3\lt{0}\)
উত্তরঃ \(S=\left\{x\in{\mathbb{R}}: 1\lt{x}\lt{3}\right\}\)
সংখ্যারেখাঃ
realNumber

\(Ex.7.(f)\) \(2x+4\gt{8}\)
উত্তরঃ \(S=\left\{x\in{\mathbb{R}}: x\gt{2}\right\}\)
সংখ্যারেখাঃ
realNumber

\(Ex.7.(g)\) \(2x+3\lt{5}\)
উত্তরঃ \(S=\left\{x\in{\mathbb{R}}: x\lt{1}\right\}\)
সংখ্যারেখাঃ
realNumber

\(Ex.7.(h)\) \(2x-10\gt{5x+2}\)
উত্তরঃ \(S=\left\{x\in{\mathbb{R}}: x\lt{-4}\right\}\)
সংখ্যারেখাঃ
realNumber

\(Ex.7.(i)\) \(\frac{2x}{x-1}\gt{x}\)
উত্তরঃ \(S=\left\{x\in{\mathbb{R}}: x\lt{0} \text{ অথবা} \ 1\lt{x}\lt{3}\right\}\)
সংখ্যারেখাঃ
realNumber

\(Ex.7.(j)\) \(\frac{4x-1}{x+2}\lt{1}\)
উত্তরঃ \(S=\left\{x\in{\mathbb{R}}: -2\lt{x}\lt{1}\right\}\)
সংখ্যারেখাঃ
realNumber

\(Ex.7.(k)\) \(6x^2+x-2\lt{0}\)
উত্তরঃ \(S=\left\{x\in{\mathbb{R}}: -\frac{2}{3}\lt{x}\lt{\frac{1}{2}}\right\}\)
সংখ্যারেখাঃ
realNumber

\(Ex.7.(l)\) \(6x^2+x-2\gt{0}\)
উত্তরঃ \(S=\left\{x\in{\mathbb{R}}: x\lt{-\frac{2}{3}} \text{ অথবা} \ x\gt{\frac{1}{2}}\right\}\)
সংখ্যারেখাঃ
realNumber

\(Ex.7.(m)\) \(\frac{x}{x^2+1}\lt{\frac{1}{x+1}}\)
উত্তরঃ \(S=\left\{x\in{\mathbb{R}}: -1\lt{x}\lt{1}\right\}\)
সংখ্যারেখাঃ
realNumber

পরমমান চিহ্ন ব্যতীত প্রকাশ করঃ
\(Ex.8.(a)\) \(|x-2|\lt{5}\)
উত্তরঃ \(-3\lt{x}\lt{7}\)
বঃ ২০০২; ঢাঃ ২০০৩,২০০৯; দিঃ ২০১১ ।

পরমমান চিহ্ন ব্যতীত প্রকাশ করঃ
\(Ex.8.(b)\) \(\frac{1}{|2x-1|}\ge{7}, \ x\ne{\frac{1}{2}}\)
উত্তরঃ \(\frac{3}{7}\le{x}\lt{\frac{4}{7}}, \ x\ne{\frac{1}{2}}\)

\(Ex.8.(c)\) \(|7-3x|\ge{5}\)
উত্তরঃ \(x\le{\frac{2}{3}}\) অথবা \(x\ge{4}\)

\(Ex.8.(d)\) \(|2x-7|\lt{13}\)
উত্তরঃ \(-3\lt{x}\lt{10}\)

\(Ex.8.(e)\) \(|7x-50|\gt{81}\)
উত্তরঃ \(x\lt{-\frac{31}{7}}\) অথবা \(x\gt{\frac{131}{7}}\)

\(Ex.8.(f)\) \(|x+4|\lt{3}\)
উত্তরঃ \(-7\lt{x}\lt{-1}\)

\(Ex.8.(g)\) \(|x-4|\lt{2}\)
উত্তরঃ \(2\lt{x}\lt{6}\)

পরমমান চিহ্নের সাহায্যে প্রকাশ করঃ
\(Ex.9.(a)\) \(-2\lt{x}\lt{6}\)
উত্তরঃ \(|x-2|\lt{4}\)

পরমমান চিহ্নের সাহায্যে প্রকাশ করঃ
\(Ex.9.(b)\) \(-5\lt{x}\lt{11}\)
উত্তরঃ \(|x-3|\lt{8}\)

\(Ex.9.(c)\) \(-3\lt{x}\lt{11}\)
উত্তরঃ \(|x-4|\lt{7}\)

\(Ex.9.(d)\) \(-5\lt{x}\lt{12}\)
উত্তরঃ \(|2x-7|\lt{17}\)

সমাধান কর এবং সমাধান সেট সংখ্যারেখায় দেখাওঃ
\(Ex.10.(a)\) \(|2x+1|\lt{3}\)
উত্তরঃ \(S=\left\{x\in{\mathbb{R}}:-2\lt{x}\lt{1}\right\}\)
সংখ্যারেখাঃ
realNumber

সমাধান কর এবং সমাধান সেট সংখ্যারেখায় দেখাওঃ
\(Ex.10.(b)\) \(|x-3|\ge{2}\)
উত্তরঃ \(S=\left\{x\in{\mathbb{R}}: x\le{1}\right\}\cup\left\{x\in{\mathbb{R}}: x\ge{5}\right\}\)
সংখ্যারেখাঃ
realNumber

\(Ex.10.(c)\) \(\frac{1}{|3x-5|}\gt{2}, \ x\ne{\frac{5}{3}}\)
উত্তরঃ \(S=\left\{x\in{\mathbb{R}}: \frac{3}{2}\lt{x}\lt{\frac{11}{6}} \text{ এবং} \ x\ne{\frac{5}{3}}\right\}\)
সংখ্যারেখাঃ
realNumber

\(Ex.10.(d)\) \(|x-5|=|2x-3|\)
উত্তরঃ \(S=\left\{-2, \ \frac{8}{3}\right\}\)
সংখ্যারেখাঃ
realNumber

\(Ex.10.(e)\) \(|12x-11|\le{7}\)
উত্তরঃ \(S=\left\{x\in{\mathbb{R}}: \frac{1}{3}\le{x}\le{\frac{3}{2}}\right\}\)
সংখ্যারেখাঃ
realNumber

\(Ex.10.(f)\) \(6x^2-x-1\gt{0}\)
উত্তরঃ \(S=\left\{x\in{\mathbb{R}}: x\gt{\frac{1}{2}} \text{ অথবা} \ x\le{-\frac{1}{3}}\right\}\)
সংখ্যারেখাঃ
realNumber

\(Ex.10.(g)\) \(\frac{x}{x^2+1}\lt{\frac{1}{x+1}}\)
উত্তরঃ \(S=\left\{x\in{\mathbb{R}}: -1\lt{x}\lt{1}\right\}\)
সংখ্যারেখাঃ
realNumber

\(Ex.10.(h)\) \(|2x+3|\gt{7}\)
উত্তরঃ \(S=\left\{x\in{\mathbb{R}}: x\gt{2} \text{ অথবা} \ x\le{-5}\right\}\)
সংখ্যারেখাঃ
realNumber

\(Ex.10.(i)\) \(1\lt{|x|}\lt{2}\)
উত্তরঃ \(S=\left\{x\in{\mathbb{R}}: -2\lt{x}\lt{-1} \text{ অথবা} \ 1\lt{x}\lt{2}\right\}\)
সংখ্যারেখাঃ
realNumber

\(Ex.10.(j)\) \(|3x+2|\lt{4x}\)
উত্তরঃ \(S=\left\{x\in{\mathbb{R}}: -\frac{7}{2}\lt{x}\lt{-\frac{2}{3}} \text{ অথবা} \ x\gt{2}\right\}\)
সংখ্যারেখাঃ
realNumber

\(Ex.10.(k)\) \(|x+1|+|x-2|\le{5}\)
উত্তরঃ \(S=\left\{x\in{\mathbb{R}}: -2\le{x}\le{3}\right\}\)
সংখ্যারেখাঃ
realNumber

\(Ex.10.(l)\) \(\left|\frac{2x}{x-2}\right|\le{1}\)
উত্তরঃ \(S=\left\{x\in{\mathbb{R}}: -2\le{x}\le{\frac{2}{3}}\right\}\)
সংখ্যারেখাঃ
realNumber

\(Ex.10.(m)\) \(\left|\frac{x-1}{x+2}\right|\lt{2}\)
উত্তরঃ \(S=\left\{x\in{\mathbb{R}}: x\lt{-5}, \ -2\lt{x}\lt{-1} \text{ অথবা} \ x\ge{1}\right\}\)
সংখ্যারেখাঃ
realNumber

মান নির্ণয় করঃ
\(Ex.11.(a)\) \(\left||2-6|-10+|7-3|\right|\)
উত্তরঃ \(2\)

মান নির্ণয় করঃ
\(Ex.11.(b)\) \(|-5-7|-|-2+9|+|-3|\)
উত্তরঃ \(8\)

ভগ্নাংশ আকারে প্রকাশ করে দেখাও যে, নিম্নলিখিত সংখ্যাগুলি মূলদ সংখ্যাঃ
\(Ex.12.(a)\) \(2.090909...\)

ভগ্নাংশ আকারে প্রকাশ করে দেখাও যে, নিম্নলিখিত সংখ্যাগুলি মূলদ সংখ্যাঃ
\(Ex.12.(b)\) \(3.274274274.....\)

\(Ex.12.(c)\) \(2.309\)

\(Ex.12.(d)\) \(3.1787878 ......\)

\(Ex.12.(e)\) \(1.2345......\)

দেখাও যে, নিম্নলিখিত সংখ্যাগুলি অমূলদ।
\(Ex.13.(a)\) \(\sqrt{2}\)
বঃ ২০১৪; কুঃ ২০০৭; রাঃ ২০০৫ ; সিঃ ২০১৩,২০০৫ ।

দেখাও যে, নিম্নলিখিত সংখ্যাগুলি অমূলদ।
\(Ex.13.(b)\) \(\sqrt{3}\)
ঢাঃ ২০১১,২০০৮; চঃ ২০১৪,২০১১; বঃ ২০১২; কুঃ ২০১৩,২০১১ রাঃ ২০১২,২০১০ ; সিঃ ২০১৫,২০১১; যঃ ২০১৬,২০১৩,২০১১; মাঃ ২০১৫,২০১৪,২০১০ ।

\(Ex.13.(c)\) \(\sqrt{5}\)
বঃ ২০০৭; ঢাঃ ২০১৪; যঃ ২০০৬ ।

\(Ex.13.(d)\) \(\sqrt{7}\)

\(Ex.13.(e)\) \(\sqrt{11}\)

\(Ex.13.(f)\) \(\sqrt{3}+\sqrt{2}\)

\(Ex.14.\) যদি \(a\in{\mathbb{R}}\) হয় তবে প্রমাণ কর যে, \(a.0=0\)।
ঢাঃ ২০১৪,২০০৯; কুঃ ২০০৬; রাঃ ২০১৪ ; দিঃ ২০১৬; ।

\(Ex.15.\) যদি \(a, \ b\in{\mathbb{R}}\) হয় তবে প্রমাণ কর যে, \(-(a+b)=-a-b\)।
রাঃ ২০১১ ।

\(Ex.16.\) সংখ্যারেখায় \(\sqrt{13}\) এর অবস্থান নির্ণয় কর।

নিম্নলিখিত সেটগুলির সুপ্রিমাম \((Sup)\) এবং ইনফিমাম \((Inf)\) নির্ণয় করঃ
\(Ex.17.(a)\) যদি \(P=\{x\in{\mathbb{N}}: 4\le{x^2}\le{81}\}.\)
উত্তরঃ \(Sup(P)=9\) এবং \(Inf(P)=2\)

নিম্নলিখিত সেটগুলির সুপ্রিমাম \((Sup)\) এবং ইনফিমাম \((Inf)\) নির্ণয় করঃ
\(Ex.17.(b)\) \(S=\{1, \ 2, \ 3, \ 4\}.\)
উত্তরঃ সুপ্রিমাম \(Sup(S)=4\)

\(Ex.17.(c)\) \(S=\{1, \ 5, \ 7\}.\)
উত্তরঃ ইনফিমাম \(Inf(S)=1\)

\(Ex.17.(d)\) \(S=\{-2, \ -\frac{3}{2}, \ -\frac{4}{3}\}.\)
উত্তরঃ সুপ্রিমাম \(\left(Sup(S)\right)\) নেই এবং ইনফিমাম \(Inf(S)=-2\)

\(Ex.17.(e)\) \(S=\left\{\frac{n}{n+1}: n\in{\mathbb{N}}\right\}.\)
উত্তরঃ সুপ্রিমাম \(Sup(S)=1\) এবং ইনফিমাম \(Inf(S)=\frac{1}{2}\)
ঢাঃ ২০১৯;যঃ ২০১৯; ।

\(Ex.17.(f)\) \(S=\left\{x\in{\mathbb{R}}: 2x^2+3x-5\le{0}\right\}.\)
উত্তরঃ সুপ্রিমাম \(Sup(S)=1\) এবং ইনফিমাম \(Inf(S)=-\frac{5}{2}\)

\(Ex.17.(g)\) \(S=\left\{x\in{\mathbb{N}}: 7\lt{x^2}\lt{39}\right\}.\)
উত্তরঃ সুপ্রিমাম \(Sup(S)=6\) এবং ইনফিমাম \(Inf(S)=3\)

\(Ex.17.(h)\) \(S=\left\{x\in{\mathbb{R}}: 3x^2-7x+2\lt{0}\right\}.\)
উত্তরঃ সুপ্রিমাম \(Sup(S)=2\) এবং ইনফিমাম \(Inf(S)=\frac{1}{3}\)

\(Ex.17.(i)\) \(S=\left\{2, \ 1, \ 3, \ 2, \ \frac{3}{2}, \ 4, \ 3, \ \frac{4}{3} .....\right\}.\)
উত্তরঃ সুপ্রিমাম \(\left(Sup(S)\right)\) নেই। ইনফিমাম \(Inf(S)=1\)

\(Ex.17.(j)\) \(S=\left\{-1, \ 1, \ -2, \ \frac{1}{2}, \ -3, \ \frac{1}{3} .....\right\}.\)
উত্তরঃ সুপ্রিমাম \(Sup(S)=1\), ইনফিমাম \(Inf(S)\) নেই।

\(Ex.17.(k)\) \(S=\left\{(-1)^n\frac{n}{n+1}: n\in{\mathbb{N}}\right\}.\)
উত্তরঃ সুপ্রিমাম \(Sup(S)=1\) এবং ইনফিমাম \(Inf(S)=-1\)

\(Ex.18.\) সমাধান করঃ \(a(x+b)\lt{c}\)
উত্তরঃ \(x\lt{\frac{c}{a}-b}\) যখন, \(a\gt{0}\)
\(x\gt{\frac{c}{a}-b}\) যখন, \(a\lt{0}\)

\(Ex.19.\) \(x\in{\mathbb{R}}\) এর জন্য সমাধান সেট নির্ণয় করঃ \(|x+1|+|x-1|\le{3}\)
উত্তরঃ \(S=\left\{x\in{\mathbb{R}}: -\frac{3}{2}\le{x}\le{\frac{3}{2}}\right\}\)

অসমতাযুগলের সমাধান সেটের লেখচিত্র অঙ্কন করঃ
\(Ex.20.(a)\) \(x+2y\le{10}\) এবং \(x+y\le{6}\)।

অসমতাযুগলের সমাধান সেটের লেখচিত্র অঙ্কন করঃ
\(Ex.20.(b)\) \(2x+3y-6\gt{0}\) এবং \(2x+3y-6\lt{0}\)।

\(Ex.20.(c)\) \(3x-2y-1\ge{0}\) এবং \(3x+2y-7\le{0}\)।

\(Ex.20.(d)\) \(3x+2y-1\ge{0}\) এবং \(3x-2y-7\le{0}\)।

\(Ex.20.(e)\) \(2x+3y-7\le{0}\) এবং \(y\le{2x}\)।

\(Ex.20.(f)\) \(2x+y\le{10}\) এবং \(x+3y\le{15}\)।

\(Ex.20.(g)\) \(2x+3y\ge{18}\) এবং \(5x+y\ge{10}\)।

\(Ex.20.(h)\) \(x+2y\le{12}\) এবং \(3x-y\le{6}\)।

\(Ex.21.\) \(a\) ও \(b\) ধনাত্মক সংখ্যা হলে দেখাও যে, \(\frac{a+b}{2}\ge{\sqrt{ab}}\ge{\frac{2ab}{a+b}}\)

অসমতাটির লৈখিক পদ্ধতিতে সমাধান করঃ
\(Ex.22.(a)\) \(x^2-3x-10\lt{0}\)
উত্তরঃ \(S=\left\{x\in{\mathbb{R}}: -2\lt{x}\lt{5}\right\}\)

অসমতাটির লৈখিক পদ্ধতিতে সমাধান করঃ
\(Ex.22.(b)\) \(x^2-5x+4\gt{0}\)
উত্তরঃ \(S=\left\{x\in{\mathbb{R}}: x\lt{1} \text{ অথবা} \ x\gt{4}\right\}\)

সংখ্যারেখার সাহায্যে সমাধান করঃ
\(Ex.23.(a)\) \(4x+5\gt{21}\)
উত্তরঃ \(S=\left\{x\in{\mathbb{R}}: x\gt{4}\right\}\)

সংখ্যারেখার সাহায্যে সমাধান করঃ
\(Ex.23.(b)\) \(x^2\gt{4}\)
উত্তরঃ \(S=\left\{x\in{\mathbb{R}}: x\lt{-2} \text{ অথবা} \ x\gt{2}\right\}\)

\(Ex.23.(c)\) \(x^2\lt{9}\)
উত্তরঃ \(S=\left\{x\in{\mathbb{R}}: -3\lt{x}\lt{3}\right\}\)

\(Ex.23.(d)\) \(x^2\lt{4}\)
উত্তরঃ \(S=\left\{x\in{\mathbb{R}}: -2\lt{x}\lt{2}\right\}\)

\(Ex.23.(e)\) \(x^2-2x\gt{0}\)
উত্তরঃ \(S=\left\{x\in{\mathbb{R}}: x\lt{0} \text{ অথবা} \ x\gt{2}\right\}\)

\(Ex.23.(f)\) \(\frac{x-3}{x-4}\lt{\frac{x-2}{x-1}}\)
উত্তরঃ \(S=\left\{x\in{\mathbb{R}}: x\lt{1} \text{ অথবা} \ \frac{5}{2}\lt{x}\lt{4}\right\}\)

\(Ex.24.\) \(x^2-4x-5\lt{0}\) এবং \(6x-5\gt{1}\) অসমতা দুইটির সমাধান কর।
উত্তরঃ \(S=\left\{x\in{\mathbb{R}}: 1\lt{x}\lt{5}\right\}\)

\(Ex.25.\) \(a, \ b, \ c, \ d\in{\mathbb{R}}, \ a+b=a+c\) এবং \(d=\sqrt{3}.\)
\((a)\) \(A=\left\{x\in{\mathbb{R}}: |x-2|\le{3}\right\}\) এর ক্ষুদ্রতম ঊর্ধসীমা \(\left(Sup(A)\right)\) নির্ণয় কর।
উত্তরঃ \((a) \ Sup(A)=5\)
\((b)\) প্রমাণ কর যে, \(b=c\)
ঢাঃ ২০৪,২০১২; বঃ ২০০৭,২০০৯; কুঃ ২০০৪ রাঃ ২০১১ ; সিঃ ২০০৪,২০০৯; যঃ ২০০৩,২০১১ ।
\((c)\) প্রমাণ কর যে, \(d\) একটি অমূলদ সংখ্যা।
ঢাঃ ২০১১,২০০৮; চঃ ২০১৪,২০১১; বঃ ২০১২; কুঃ ২০১৩,২০১১ রাঃ ২০১২,২০১০ ; সিঃ ২০১৫,২০১১; যঃ ২০১৬,২০১৩,২০১১; মাঃ ২০১৫,২০১৪,২০১০ ।

\(Ex.26.\) \(f(x)=x-1\) যেখানে, \(x\in{\mathbb{R}}.\)
\((a)\) \(-2\lt{x}\lt{6}\) অসমতাকে পরম মান চিহ্ন ব্যবহার করে প্রকাশ কর।
উত্তরঃ \((a) \ |x-2|\lt{4}\)
বঃ ২০০১; রাঃ ২০০২ ; চঃ ২০০৪ ।
\((b)\) \(|f(x)|\lt{\frac{1}{10}}\) হলে, দেখাও যে, \(|f(x)\times{f(x+2)}|\lt{\frac{21}{100}}.\)
ঢাঃ ২০১৭,২০১৪,২০১২,২০১০,২০০৬; চঃ২০১৫; সিঃ ২০১৫,২০০৯; রাঃ ২০১০; দীঃ ২০১৫,২০১২; যঃ২০০৮; চঃ ২০১৬,২০১১,২০০৮; বঃ ২০১৩,২০০৮; মাঃ ২০১৪,২০১১; রুয়েটঃ ২০১২-২০১৩ ।
\((c)\) সংখ্যারেখার সাহায্যে সমাধান করঃ \(f(x)\times{f(x-1)}\gt{0}\) এবং \(f(x-3)\times{f(x+2)}\le{0}.\)
ঢাঃ ২০১১,২০০৮; চঃ ২০১৪,২০১১; বঃ ২০১২; কুঃ ২০১৩,২০১১ রাঃ ২০১২,২০১০ ; সিঃ ২০১৫,২০১১; যঃ ২০১৬,২০১৩,২০১১; মাঃ ২০১৫,২০১৪,২০১০ ।

\(Ex.27.\) নিচের অসমতা দুইটি লক্ষ করঃ
\((i) \ |3x-4|\le{2}\)
\((ii) \ \frac{1}{|2x-3|}\gt{2}; \ x\ne{\frac{3}{2}}\)
\((a)\) \(-7\lt{x}\lt{-1}\) অসমতাকে পরম মান চিহ্ন ব্যবহার করে প্রকাশ কর।
\((b) \ (i)\) এর সমাধান নির্ণয় কর।
\((c) \ (ii)\) এর সমাধান সেট নির্ণয় কর এবং সংখ্যারেখায় দেখাও।
উত্তরঃ \((a) \ |x+4|\lt{3}\)
\((b) \ \frac{2}{3}\le{x}\le{2}\)
\((c) \ \frac{5}{4}\lt{x}\lt{\frac{7}{4}}; \ x\ne{\frac{3}{2}}\)
সংখ্যারেখাঃ
realNumber

\(Ex.28.\) \(f(x)=|3x+1|\) একটি পরমমান ফাংশন এবং \(S\in{\mathbb{R}};\) যেখানে, \(\mathbb{R}\) একটি বাস্তব সংখ্যার সেট যা যোগ প্রক্রিয়ায় আবদ্ধ।
\((a)\) যদি \(S=\left\{x: 5x^2-16x+3\lt{0}\right\}\) হয়, তবে \(\left(Sup(S)\right)\) এবং \(Inf(S)\) নির্ণয় কর।
\((b) \ \frac{1}{f(x)}\ge{5}\) অসমতাটি সমাধান কর এবং সমাধান সেট সংখ্যারেখায় দেখাও। যেখানে, \(x\ne{-\frac{1}{3}}\)
\((c)\) যদি \(p, \ q, \ r\in{\mathbb{R}}\) এবং \(p+q=p+r\) হয়, তবে প্রমাণ কর যে, \(q=r.\)
উত্তরঃ \((a) \ Sup(S)=3\) এবং \(Inf(S)=\frac{1}{5}\)
\((b) \ -\frac{2}{5}\le{x}\le{-\frac{4}{15}}, \ x\ne{-\frac{1}{3}}\)
সংখ্যারেখাঃ
realNumber

\(Ex.29.\) দৃশ্যকল্প-১: \(f(x)=x-1;\) যেখানে, \(x\in{\mathbb{R}}\)
দৃশ্যকল্প-২: মূলদ সংখ্যার সেট \(\mathbb{Q}\) এবং \(r=\sqrt{10}.\)
\((a)\) \(-2\lt{x}\lt{6}\) অসমতাটিকে পরমমান চিহ্নের সাহায্যে প্রকাশ কর।
\((b)\) \(|f(x)|\lt{\frac{3}{5}}\) হলে দেখায় যে, \(|f(x)f(x+2)|\lt{\frac{39}{25}}\)
\((c)\) প্রমাণ কর যে, \(r\notin{\mathbb{Q}}\)
উত্তরঃ \((a) \ |x-2|\lt{4}\)

\(Ex.30.\) দৃশ্যকল্প-১: \(L=\left\{x\in{\mathbb{R}}: 2x^2+5x\lt{0}\right\}\) এবং
দৃশ্যকল্প-২: \(f(x)=|x-3|\)
\((a)\) সমাধান করঃ \(|2x-7|\gt{5}\)
যঃ ২০১৭ ।
\((b)\) \(L\) সমাধান সেটের অসমতাটিকে পরমমান চিহ্নের সাহায্যে প্রকাশ কর।
যঃ ২০১৭ ।
\((c)\) \(f(x)\lt{\frac{1}{5}}\) হলে দেখাও যে, \(f(x^2-6)\lt{\frac{31}{25}}\)
সিঃ ২০১৭ ।
উত্তরঃ \((a) \ x\lt{1}\) অথবা \(x\gt{6}\)
\((b) \ \left|4x+5\right|\lt{5}\)

\(Ex.31.\) জায়ান এবং জারিফ তাদের বাবার জন্য একটি উপহার কিনতে চায়। উপহার কিনতে গিয়ে জায়ান, জারিফের চেয়ে \(5\) টাকা বেশি দিল। যদি উপহারটির মূল্য \(41\) টাকার বেশি না হয় তবে জায়ানকে সর্বাধিক কত টাকা দিতে হলো?
উত্তরঃ \(23\) টাকা।

\(Ex.32.\) পাঁচটি কুইজ প্রতিযোগিতার প্রথম চারটিতে অংশ নিয়ে জায়ান স্কোর করেছে \(78, \ 72, \ 87\) এবং \(90\) পঞ্চম প্রতিযোগিতার স্কোর কত হলে জায়নের গড় স্কোর কমপক্ষে \(82\) হবে?
উত্তরঃ \(83\) টাকা।

Read Example
Q.1-এর সংক্ষিপ্ত প্রশ্নসমূহ
মান নির্ণয় করঃ
\(Q.1.i.(a)\) \(|1+3|+|1-3|\)
উত্তরঃ \(6\)

মান নির্ণয় করঃ
\(Q.1.i.(b)\) \(|-3-5|\)
উত্তরঃ \(8\)

\(Q.1.i.(c)\) \(\left||3-5|+|7-12|\right|\)
উত্তরঃ \(7\)
কুঃ ২০০৬ ।

\(Q.1.i.(d)\) \(\left||2-6|-|1-9|\right|\)
উত্তরঃ \(4\)
রাঃ ২০০৫ ।

\(Q.1.i.(e)\) \(|-1-8|+|3-1|\)
উত্তরঃ \(11\)
রাঃ ২০০৫ ।

\(Q.1.i.(f)\) \(\left||-2|-|-6|\right|\)
উত্তরঃ \(4\)

\(Q.1.i.(g)\) \(|a-3a|-|5a-7a|\)
উত্তরঃ \(0\)

\(Q.1.i.(h)\) \(\left||-9a|-|11a-2a|-7\right|\)
উত্তরঃ \(7\)

\(Q.1.i.(i)\) \(\left||-16+3|+|-1-4|-3-|-1-7|\right|\)
উত্তরঃ \(7\)
চঃ ২০০৫ ।

\(Q.1.i.(j)\) \(13+|-1-4|-3-|-8|\)
উত্তরঃ \(7\)
কুঃ ২০১০; মাঃ২০১১ ।

\(Q.1.i.(k)\) \(|-3|\)
উত্তরঃ \(3\)

\(Q.1.i.(l)\) \(|-2|-|-7|\)
উত্তরঃ \(-5\)


\(Q.1.i.(m)\) \(|-2-6|\)
উত্তরঃ \(8\)

\(Q.1.i.(n)\) \(|-|-7||\)
উত্তরঃ \(7\)

পরমমান চিহ্ন ব্যতীত প্রকাশ করঃ
\(Q.1.ii.(a)\) \(|x|\lt{3}\)
উত্তরঃ \(-3\lt{x}\lt{3}\)
ঢাঃ২০০৩ ।

পরমমান চিহ্ন ব্যতীত প্রকাশ করঃ
\(Q.1.ii.(b)\) \(|x+1|\le{2}\)
উত্তরঃ \(-3\le{x}\le{1}\)
কুঃ ২০১০; মাঃ২০১১ ।

\(Q.1.ii.(c)\) \(|x-3|\le{7}\)
উত্তরঃ \(-4\le{x}\le{10}\)
কুঃ ২০০৫ ।

\(Q.1.ii.(d)\) \(\left|x-\frac{4}{3}\right|\le{5}\)
উত্তরঃ \(-\frac{11}{3}\le{x}\le{\frac{19}{3}}\)

\(Q.1.ii.(e)\) \(\frac{1}{|1-4x|}\ge{3}, \ x\ne{\frac{1}{4}}\)
উত্তরঃ \(\frac{1}{6}\le{x}\le{\frac{1}{3}}, \ x\ne{\frac{1}{4}}\)

\(Q.1.ii.(f)\) \(1\lt{|x|}\lt{5}\)
উত্তরঃ \(1\lt{x}\lt{5} or, \ -5\lt{x}\lt{-1}\)

\(Q.1.ii.(g)\) \(|2x+3|\lt{7}\)
উত্তরঃ \(-5\lt{x}\lt{2}\)
বঃ২০০২; ঢাঃ২০০৩,২০০৯; চঃ২০১২; রাঃ, কুঃ, চঃ, বঃ ২০১৮ ।

\(Q.1.ii.(h)\) \(\frac{1}{|3x+1|}\ge{5}, \ x\ne{-\frac{1}{3}}\)
উত্তরঃ \(-\frac{2}{5}\le{x}\le{-\frac{4}{15}}, \ x\ne{-\frac{1}{3}}\)
যঃ ২০০৮; চঃ ২০১৩; সিঃ ২০০৬; রুয়েটঃ ২০১০-২০১১ ।

\(Q.1.ii.(i)\) \(|2x+1|\lt{5}\)
উত্তরঃ \(-3\lt{x}\lt{2}\)

\(Q.1.ii.(j)\) \(|x-2|\lt{7}\)
উত্তরঃ \(-5\le{x}\le{9}\)
কুঃ ২০০৫ ।

\(Q.1.ii.(k)\) \(2\le{\frac{1}{|x-1|}}\) এখানে, \(x\ne{1}\)
উত্তরঃ \(\frac{1}{2}\le{x}\le{\frac{3}{2}} \text{ এবং} \ x\ne{1}\)
রাঃ ২০১৫; রূয়েটঃ ২০০৫-২০০৬ ।

\(Q.1.ii.(l)\) \(\frac{1}{|x-1|}\lt{2}\) এখানে, \(x\ne{1}\)
উত্তরঃ \(x\lt{\frac{1}{2}} \text{ অথবা} \ x\gt{\frac{3}{2}} \text{ এবং} \ x\ne{1}\)

\(Q.1.ii.(m)\) \(3\le{|x-2|}\le{7}\)
উত্তরঃ \(-2\le{x}\le{-1}\) অথবা \(5\le{x}\le{9}\)

\(Q.1.ii.(n)\) \(|5-2x|\ge{4}\)
উত্তরঃ \(x\le{\frac{1}{2}}\) অথবা \(x\ge{\frac{9}{2}}\)

\(Q.1.ii.(o)\) \(\frac{1}{|2x+1|}\ge{5}\) \(x\ne{-\frac{1}{2}}\)
উত্তরঃ \(-\frac{3}{5}\le{x}\lt{-\frac{1}{2}}\) অথবা \(-\frac{1}{2}\lt{x}\le{-\frac{2}{5}}\)

\(Q.1.ii.(p)\) \(|2x+4|\lt{8}\)
উত্তরঃ \(-6\lt{x}\lt{2}\)\)

\(Q.1.ii.(q)\) \(\frac{1}{|3x-5|}\gt{2}, \ x\ne{\frac{5}{3}}\)
উত্তরঃ \(\frac{3}{2}\lt{x}\lt{\frac{5}{3}}\) অথবা \(\frac{5}{3}\lt{x}\lt{\frac{11}{6}}\)
কুঃ ২০১৩; সিঃ ২০১৪; বঃ ২০১৪ ।

\(Q.1.ii.(r)\) \(\frac{1}{|5x-1|}\gt{\frac{1}{9}}, \ x\ne{\frac{1}{5}}\)
উত্তরঃ \(-\frac{8}{5}\lt{x}\lt{\frac{1}{5}}\) অথবা \(\frac{1}{5}\lt{x}\lt{2}\)
কুঃ ২০১৫।

পরমমান চিহ্নের সাহায্যে প্রকাশ করঃ
\(Q.1.iii.(a)\) \(-1\lt{x}\lt{3}\)
উত্তরঃ \(|x-1|\lt{2}\)

পরমমান চিহ্নের সাহায্যে প্রকাশ করঃ
\(Q.1.iii.(b)\) \(-8\le{x}\le{2}\)
উত্তরঃ \(|x+3|\le{5}\)

\(Q.1.iii.(c)\) \(-6\le{5x}\le{0}\)
উত্তরঃ \(|5x+3|\le{3}\)

\(Q.1.iii.(d)\) \(-2\lt{3-x}\lt{8}\)
উত্তরঃ \(|x|\gt{5}\)

\(Q.1.iii.(e)\) \(1\le{3x+7}\le{5}\)
উত্তরঃ \(|3x+4|\le{2}\)

\(Q.1.iii.(f)\) \(2\le{2x+3}\le{4}\)
উত্তরঃ \(|2x|\le{1}\)

\(Q.1.iii.(g)\) \(-5\lt{x}\lt{7}\)
উত্তরঃ \(|x-1|\lt{6}\)
রাঃ ২০১৩; বঃ ২০০৬ ।

\(Q.1.iii.(h)\) \(2\le{x}\le{8}\)
উত্তরঃ \(|x-5|\lt{3}\)
যঃ ২০০৭ ।

\(Q.1.iii.(i)\) \(-7\lt{x}\lt{-1}\)
উত্তরঃ \(|x+4|\lt{3}\)
চঃ ২০০৯; ঢাঃবিঃ ২০১৫-২০১৬ ।

\(Q.1.iii.(j)\) \(2x^2+5x\lt{0}\)
উত্তরঃ \(\left|x+\frac{5}{4}\right|\lt{\frac{5}{4}}\)
যঃ ২০১৭ ।

\(Q.1.iii.(k)\) \(-1\lt{2x-3}\lt{5}\)
উত্তরঃ \(|2x-5|\lt{3}\)
দিঃ ২০১৪; চঃ ২০০৬; সিঃ ২০১৪; যঃ ২০১০; বঃ ২০১১; মাঃ২০০৯; রুয়েটঃ ২০১০-২০১১ ।

\(Q.1.iii.(l)\) \(-4\lt{2x-1}\lt{12}\)
উত্তরঃ \(|2x-5|\lt{8}\)
সিঃ ২০১৭ ।

\(Q.1.iii.(m)\) \(-2\lt{2x+1}\lt{4}\)
উত্তরঃ \(|2x|\lt{3}\)
দিঃ ২০১৯ ।

\(Q.1.iii.(n)\) \(-1\le{x-2}\le{11}\)
উত্তরঃ \(|x-7|\le{6}\)
সিঃ ২০১৯ ।

\(Q.1.iii.(o)\) \(-3\lt{2x-1}\lt{7}\)
উত্তরঃ \(|2x-3|\lt{5}\)
কুঃ ২০১৯ ।

\(Q.1.iii.(p)\) \(4\lt{x}\lt{10}\)
উত্তরঃ \(|x-7|\lt{3}\)
বঃ২০০১; রাঃ২০০২; কুঃ ২০০৪ ।

\(Q.1.iii.(q)\) \(x^2-6x-7\lt{0}\)
উত্তরঃ \(|x-3|\lt{4}\)

\(Q.1.iii.(r)\) \(24+2x-x^2\gt{0}\)
উত্তরঃ \(|x-1|\lt{5}\)

\(Q.1.iii.(s)\) \(\frac{2x+3}{x-3}\lt{\frac{x+3}{x-1}}\)
উত্তরঃ \(|x-2|\lt{1}\)

\(Q.1.iii.(t)\) \(1\lt{x}\lt{5}\)
উত্তরঃ \(|x-3|\lt{2}\)

\(Q.1.iii.(u)\) \(-5\le{x}\le{9}\)
উত্তরঃ \(|x-2|\le{7}\)

\(Q.1.iii.(v)\) \(-3\lt{5-2x}\lt{7}\)
উত্তরঃ \(|3-2x|\lt{5}\)
দিঃ ২০১৯ ।

\(Q.1.iii.(w)\) \(-8\lt{3-x}\lt{-2}\)
উত্তরঃ \(|8-x|\lt{3}\)
কুঃ ২০১৯ ।

\(Q.1.iii.(x)\) \(-3\lt{x}\lt{9}\)
উত্তরঃ \(|x-3|\lt{6}\)

\(Q.1.iii.(y)\) \(-18\lt{x}\lt{20}\)
উত্তরঃ \(|x-1|\lt{19}\)

\(Q.1.iii.(z)\) \(-4\lt{3x}\lt{0}\)
উত্তরঃ \(|3x+2|\lt{2}\)

নিচের ব্যবধিগুলিকে সেট আকারে প্রকাশ করঃ
\(Q.1.iv.(a)\) \(\left(-\frac{1}{2}, \ \frac{3}{2}\right)\)
উত্তরঃ \(\left\{x\in{\mathbb{R}}: -\frac{1}{2}\lt{x}\lt{\frac{3}{2}}\right\}\)

নিচের ব্যবধিগুলিকে সেট আকারে প্রকাশ করঃ
\(Q.1.iv.(b)\) \([-1, \ 7]\)
উত্তরঃ \(\left\{x\in{\mathbb{R}}: -1\le{x}\le{7}\right\}\)

\(Q.1.iv.(c)\) \((-\infty, \ 0)\)
উত্তরঃ \(\left\{x\in{\mathbb{R}}: x\lt{0}\right\}\)

\(Q.1.iv.(d)\) \([0, \ \infty)\)
উত্তরঃ \(\left\{x\in{\mathbb{R}}: x\ge{0}\right\}\)

\(Q.1.iv.(e)\) \([-3, \ 2)\cup{(2, \ 5]}\)
উত্তরঃ \(\left\{x\in{\mathbb{R}}: -3\le{x}\le{5}, \ x\ne{2}\right\}\)

\(Q.1.iv.(f)\) \((-\infty, \ 1]\cup{[3, \ \infty)}\)
উত্তরঃ \(\left\{x\in{\mathbb{R}}: x\le{1} \text{ অথবা} \ x\ge{3}\right\}\)

নিচের সেটগুলিকে ব্যবধিতে প্রকাশ করঃ
\(Q.1.v.(a)\) \(\left\{x\in{\mathbb{R}}: -1\lt{x}\le{3}\right\}\)
উত্তরঃ \((-1, \ 3]\)

নিচের সেটগুলিকে ব্যবধিতে প্রকাশ করঃ
\(Q.1.v.(b)\) \(\left\{x\in{\mathbb{R}}: x\le{-5} \text{ অথবা} \ x\ge{2}\right\}\)
উত্তরঃ \((-\infty, \ -5]\cup{[2, \ \infty)}\)

\(Q.1.v.(c)\) \(\left\{x\in{\mathbb{R}}: -2\le{x}\le{2}\right\}\)
উত্তরঃ \([-2, \ 2]\)

\(Q.1.v.(d)\) \(\left\{x\in{\mathbb{R}}: -\frac{1}{2}\le{x}\le{-\frac{1}{4}}, \ x\ne{-\frac{1}{3}}\right\}\)
উত্তরঃ \(\left[-\frac{1}{2}, \ -\frac{1}{3}\right)\cup{\left(-\frac{1}{3}, \ -\frac{1}{4}\right]}\)

\(Q.1.v.(e)\) \(\left\{x\in{\mathbb{R}}: x\ne{3}\right\}\)
উত্তরঃ \((-\infty, \ 3)\cup{(3, \ \infty)}\)

নিচের সংখ্যাগুলি মূলদ না অমূলদ তা কারণসহ নির্ণয় করঃ
\(Q.1.vi.(a)\) \(\pi\)

নিচের সংখ্যাগুলি মূলদ না অমূলদ তা কারণসহ নির্ণয় করঃ
\(Q.1.vi.(b)\) \(e\)

\(Q.1.vi.(c)\) \(0.027\)

\(Q.1.vi.(d)\) \(0.66666....\)

Read Short Question
Q.2-এর বর্ণনামূলক প্রশ্নসমূহ
সমাধান করঃ
\(Q.2.i.(a)\) \(|x-1|\le{3}\)
উত্তরঃ \(-2\le{x}\le{4}\)

সমাধান করঃ
\(Q.2.i.(b)\) \(|x+1|+|x-1|\le{5}\)
উত্তরঃ \(-\frac{5}{2}\le{x}\le{\frac{5}{2}}\)

\(Q.2.i.(c)\) \(|x+2|\lt{2}\) এবং \(x\in{\mathbb{Z}}\)
উত্তরঃ \(x=-3, \ -2, \ -1\)

\(Q.2.i.(d)\) \(|x-5|\gt{4}\)
উত্তরঃ \(x\lt{1}\) বা, \(x\gt{9}\)

\(Q.2.i.(e)\) \(|2x+3|\gt{9}\)
উত্তরঃ \(x\lt{-6}\) বা, \(x\gt{3}\)

\(Q.2.i.(f)\) \(|3x-4|\lt{2}\)
উত্তরঃ \(\frac{2}{3}\lt{x}\lt{2}\)
ঢাঃ ২০০৫; রাঃ২০০৮ ।

\(Q.2.i.(g)\) \(|x-1|=|3x-4|\)
উত্তরঃ \(\frac{3}{2}, \ \frac{5}{4}\)

\(Q.2.i.(h)\) \(|x-1|+|2x-3|=5\)
উত্তরঃ \(3, \ -\frac{1}{3}\)

\(Q.2.i.(i)\) \(|x+1|\lt{2}\) এবং \(|x-2|\lt{3}\)
উত্তরঃ \(-1\lt{x}\lt{1}\)

\(Q.2.i.(j)\) \(|x-1|\lt{2}\) এবং \(|x-2|\lt{1}\)
উত্তরঃ \(1\lt{x}\lt{3}\)

\(Q.2.i.(k)\) \(|x-1|\le{2}\) এবং \(x\in{\mathbb{N}}\)
উত্তরঃ \(x=1, \ 2, 3\)

\(Q.2.i.(l)\) \(|2x-7|\gt{5}\)
উত্তরঃ \(x\gt{6}\) বা, \(x\lt{1}\)
যঃ ২০১৭ ।

\(Q.2.i.(m)\) \(|x+1|\lt{3}\)
উত্তরঃ \(-4\lt{x}\lt{2}\)

\(Q.2.i.(n)\) \(|2x+1|\lt{3}\)
উত্তরঃ \(-2\lt{x}\lt{1}\)
যঃ ২০১২,২০০৯;সিঃ২০০৭ ।

\(Q.2.i.(o)\) \(|2x-5|\lt{3}\)
উত্তরঃ \(1\lt{x}\lt{4}\)
ঢাঃ ২০১৫,২০১৩,২০০৭; কুঃ২০০৮; রাঃ২০০৬ ।

\(Q.2.i.(p)\) \(|2x-5|\lt{1}\)
উত্তরঃ \(2\lt{x}\lt{3}\)
চঃ২০০১ ।

\(Q.2.i.(q)\) \(1\lt{|x|}\lt{5}\)
উত্তরঃ \(-5\lt{x}\lt{-1}\) অথবা \(1\lt{x}\lt{5}\)

সমাধান কর এবং সমাধান সেট সংখ্যারেখায় দেখাওঃ
\(Q.2.ii.(a)\) \(|x|\le{4}\)
উত্তরঃ \(-4\le{x}\le{4}\)
সংখ্যারেখাঃ
realNumber

সমাধান কর এবং সমাধান সেট সংখ্যারেখায় দেখাওঃ
\(Q.2.ii.(b)\) \(|3-x|\le{2}\)
উত্তরঃ \(1\le{x}\le{5}\)
সংখ্যারেখাঃ
realNumber

\(Q.2.ii.(c)\) \(|2x-5|\lt{1}\)
উত্তরঃ \(2\lt{x}\lt{3}\)
সংখ্যারেখাঃ
realNumber

\(Q.2.ii.(d)\) \(|x-5|\gt{2}\)
উত্তরঃ \(x\lt{3}\) বা, \(x\gt{7}\)
সংখ্যারেখাঃ
realNumber

\(Q.2.ii.(e)\) \(|2x+3|\lt{7}\)
উত্তরঃ \(-5\lt{x}\lt{2}\)
সংখ্যারেখাঃ
realNumber
চঃ ২০১২ ।

\(Q.2.ii.(f)\) \(|2x+5|\lt{1}\)
উত্তরঃ \(-3\lt{x}\lt{-2}\)
সংখ্যারেখাঃ
realNumber
যঃ ২০০৫ ।

\(Q.2.ii.(g)\) \(|x-2|\le{1}\) এবং \(|x-3|\le{1}\)
উত্তরঃ \(2\le{x}\le{3}\)
সংখ্যারেখাঃ
realNumber
যঃ ২০১৫ ।

\(Q.2.ii.(h)\) \(|x-5|\lt{3}\) এবং \(x\lt{5}\)
উত্তরঃ \(2\lt{x}\lt{5}\)
সংখ্যারেখাঃ
realNumber

\(Q.2.ii.(i)\) \(|3x-4|\lt{2}\)
উত্তরঃ \(\frac{2}{3}\lt{x}\lt{2}\)
সংখ্যারেখাঃ
realNumber
রাঃ ২০০৮; দিঃ ২০১৩; রুয়েটঃ ২০১১-২০১২ ।

\(Q.2.ii.(j)\) \(\frac{1}{|5x-1|}\gt{\frac{1}{9}}\) এবং \(x\ne{\frac{1}{5}}\)
উত্তরঃ \(S=\left\{x\in{\mathbb{R}}: -\frac{8}{5}\lt{x}\lt{2} \text{ এবং} \ x\ne{\frac{1}{5}}\right\}\)
সংখ্যারেখাঃ
realNumber
চঃ ২০১৫ ।

\(Q.2.ii.(k)\) \(|2x-5|\lt{3}\)
উত্তরঃ \(1\lt{x}\lt{4}\)
সংখ্যারেখাঃ
realNumber
ঢাঃ ২০১৫,২০১৩,২০০৭; কুঃ ২০০৮; রাঃ ২০০৬; যঃ ২০০৬; চঃ ২০১৪, ২০০৫; মাঃ ২০১২, ২০১০ ।

\(Q.2.ii.(l)\) \(|3x+2|\lt{7}\)
উত্তরঃ \(-3\lt{x}\lt{\frac{5}{3}}\)
সংখ্যারেখাঃ
realNumber
সিঃঃ ২০১২,২০০৭; রাঃ ২০১২; চঃ ২০১০; বঃ ২০১০ ।

\(Q.2.ii.(m)\) \(|2x-1|\lt{\frac{1}{3}}\)
উত্তরঃ \(\frac{1}{3}\lt{x}\lt{\frac{2}{3}}\)
সংখ্যারেখাঃ
realNumber
কুঃ ২০১৭ ।

\(Q.2.ii.(n)\) \(\frac{1}{|5x-3|}\ge{2}\) এবং \(x\ne{\frac{3}{5}}\)
উত্তরঃ \(S=\left\{x\in{\mathbb{R}}: \frac{1}{2}\le{x}\le{\frac{7}{10}} \text{ এবং} \ x\ne{\frac{3}{5}}\right\}\)
সংখ্যারেখাঃ
realNumber
ঢাঃ ২০১৯ ।

\(Q.2.ii.(o)\) \(|2-8x|\le{6}\)
উত্তরঃ \(S=\left\{x\in{\mathbb{R}}: -\frac{1}{2}\le{x}\le{1}\right\}\)
সংখ্যারেখাঃ
realNumber
রাঃ ২০১৪ ।

\(Q.2.ii.(p)\) \(|2x-3|\lt{1}\)
উত্তরঃ \(S=\left\{x\in{\mathbb{R}}: 1\lt{x}\lt{2}\right\}\)
সংখ্যারেখাঃ
realNumber
চঃ ২০০১ ।

\(Q.2.ii.(q)\) \(|2x+4|\lt{6}\)
উত্তরঃ \(S=\left\{x\in{\mathbb{R}}: -5\lt{x}\lt{1}\right\}\)
সংখ্যারেখাঃ
realNumber
যঃ ২০০২ ।

\(Q.2.ii.(r)\) \(\frac{1}{|3x-4|}\gt{2}\) এবং \(x\ne{\frac{4}{3}}\)
উত্তরঃ \(S=\left\{x\in{\mathbb{R}}: \frac{7}{6}\lt{x}\lt{\frac{3}{2}} \text{ এবং} \ x\ne{\frac{4}{3}}\right\}\)
সংখ্যারেখাঃ
realNumber
বঃ ২০১৯ ।

\(Q.2.ii.(s)\) \(\frac{1}{|5x+1|}\gt{\frac{1}{9}}\) এবং \(x\ne{-\frac{1}{5}}\)
উত্তরঃ \(S=\left\{x\in{\mathbb{R}}: -2\lt{x}\lt{\frac{8}{5}} \text{ এবং} \ x\ne{-\frac{1}{5}}\right\}\)
সংখ্যারেখাঃ
realNumber
যঃ ২০১৯ ।

\(Q.2.ii.(t)\) \(\frac{1}{|5x+2|}\ge{5}\)
উত্তরঃ \(S=\left\{x\in{\mathbb{R}}: -\frac{11}{25}\le{x}\le{-\frac{9}{25}} \text{ এবং} \ x\ne{-\frac{2}{5}}\right\}\)
সংখ্যারেখাঃ
realNumber
বুয়েটঃ ২০১৬-২০১৭ ।

\(Q.2.ii.(u)\) \(\left|5-\frac{2}{3x}\right|\lt{1}\)
উত্তরঃ \(S=\left\{x\in{\mathbb{R}}: \frac{1}{9}\lt{x}\lt{\frac{1}{6}} \text{ এবং} \ x\ne{0}\right\}\)
সংখ্যারেখাঃ
realNumber
ঢাবিঃ ২০১৫-২০১৬।

\(Q.2.ii.(v)\) \(2\le{\frac{1}{|x-1|}}\) এখানে, \(x\ne{1}\)
উত্তরঃ \(S=\left\{x\in{\mathbb{R}}: \frac{1}{2}\le{x}\le{\frac{3}{2}} \text{ এবং} \ x\ne{1}\right\}\)
সংখ্যারেখাঃ
realNumber
ঢাবিঃ ২০১৫-২০১৬।

\(Q.2.ii.(w)\) \(\left|\frac{1}{2x-3}\right|\gt{\frac{1}{10}}\) এখানে, \(x\ne{\frac{3}{2}}\)
উত্তরঃ \(S=\left\{x\in{\mathbb{R}}: -\frac{7}{2}\lt{x}\lt{\frac{13}{2}} \text{ এবং} \ x\ne{\frac{3}{2}}\right\}\)
সংখ্যারেখাঃ
realNumber
ঢাবিঃ ২০১৫-২০১৬।

\(Q.2.ii.(x)\) \(|4x-3|\gt{1}\)
উত্তরঃ \(S=\left\{x\in{\mathbb{R}}: x\lt{\frac{1}{2}} \text{ অথবা} \ x\gt{1}\right\}\)
সংখ্যারেখাঃ
realNumber

\(Q.2.ii.(y)\) \(|2x+3|\gt{9}\)
উত্তরঃ \(S=\left\{x\in{\mathbb{R}}: x\lt{-6} \text{ অথবা} \ x\gt{3}\right\}\)
সংখ্যারেখাঃ
realNumber
চঃ ২০০৩।

\(Q.2.ii.(z)\) \(|2x-7|\gt{5}\)
উত্তরঃ \(S=\left\{x\in{\mathbb{R}}: x\lt{1} \text{ অথবা} \ x\gt{6}\right\}\)
সংখ্যারেখাঃ
realNumber
চঃ ২০০৩।

\(Q.2.ii.(aa)\) \(|x|\lt{4}\)
উত্তরঃ \(S=\left\{x\in{\mathbb{R}}: -4\lt{x}\lt{4}\right\}\)
সংখ্যারেখাঃ
realNumber
রাঃ ২০০১।

\(Q.2.ii.(ab)\) \(|x-5|\gt{4}\)
উত্তরঃ \(S=\left\{x\in{\mathbb{R}}: x\lt{1} \text{ অথবা} \ x\gt{9}\right\}\)
সংখ্যারেখাঃ
realNumber

\(Q.2.ii.(ac)\) \(\frac{1}{|x-1|}\ge{2}\) এখানে, \(x\ne{1}\)
উত্তরঃ \(S=\left\{x\in{\mathbb{R}}: \frac{1}{2}\le{x}\le{\frac{3}{2}} \text{ এবং} \ x\ne{1}\right\}\)
সংখ্যারেখাঃ
realNumber
ঢাবিঃ ২০১৫-২০১৬।

\(Q.2.ii.(ad)\) \(\frac{x+1}{x+2}\gt{\frac{x+3}{x+4}}\)
উত্তরঃ \(S=\left\{x\in{\mathbb{R}}: -4\lt{x}\lt{-2}\right\}\)
সংখ্যারেখাঃ
realNumber

\(Q.2.ii.(ae)\) \(|7-3x|\le{5}\)
উত্তরঃ \(S=\left\{x\in{\mathbb{R}}: \frac{2}{3}\le{x}\le{4}\right\}\)
সংখ্যারেখাঃ
realNumber

\(Q.2.ii.(af)\) \(|5-2x|\ge{4}\)
উত্তরঃ \(S=\left\{x\in{\mathbb{R}}: x\le{\frac{1}{2}} \text{ অথবা} \ x\ge{\frac{9}{2}}\right\}\)
সংখ্যারেখাঃ
realNumber

\(Q.2.ii.(ag)\) \(x^2-5x+4\gt{0}\)
উত্তরঃ \(S=\left\{x\in{\mathbb{R}}: x\lt{1} \text{ অথবা} \ x\gt{4}\right\}\)
সংখ্যারেখাঃ
realNumber

\(Q.2.ii.(ah)\) \(x^2-7x+6\lt{0}\)
উত্তরঃ \(S=\left\{x\in{\mathbb{R}}: 1\lt{x}\lt{6}\right\}\)
সংখ্যারেখাঃ
realNumber

\(Q.2.ii.(ai)\) \(x^2-6x+5\lt{0}\)
উত্তরঃ \(S=\left\{x\in{\mathbb{R}}: 1\lt{x}\lt{5}\right\}\)
সংখ্যারেখাঃ
realNumber

\(Q.2.ii.(aj)\) \(|x-2|\lt{5}\)
উত্তরঃ \(S=\left\{x\in{\mathbb{R}}: -3\lt{x}\lt{7}\right\}\)
সংখ্যারেখাঃ
realNumber

\(Q.2.ii.(ak)\) \(|x-2|=|2x-7|\)
উত্তরঃ \(S=\left\{x\in{\mathbb{R}}: 3, \ 5\right\}\)
সংখ্যারেখাঃ
realNumber

\(Q.2.ii.(al)\) \(|3x-5|\lt{|2x-1|}\)
উত্তরঃ \(S=\left\{x\in{\mathbb{R}}: \frac{6}{5}\lt{x}\lt{4}\right\}\)
সংখ্যারেখাঃ
realNumber

\(Q.2.ii.(am)\) \(|2x+3|\lt{|3x-1|}\)
উত্তরঃ \(S=\left\{x\in{\mathbb{R}}: x\lt{-\frac{2}{5}} \text{ অথবা} \ x\gt{4}\right\}\)
সংখ্যারেখাঃ
realNumber

\(Q.2.ii.(an)\) \(|x^2-1|\lt{3}\)
উত্তরঃ \(S=\left\{x\in{\mathbb{R}}: -2\lt{x}\lt{2}\right\}\)
সংখ্যারেখাঃ
realNumber

\(Q.2.ii.(ao)\) \(x^2-5x+6\gt{0}\)
উত্তরঃ \(S=\left\{x\in{\mathbb{R}}: x\lt{2} \text{ অথবা} \ x\gt{3}\right\}\)
সংখ্যারেখাঃ
realNumber

\(Q.2.ii.(ap)\) \(x^2-4x+3\ge{0}\)
উত্তরঃ \(S=\left\{x\in{\mathbb{R}}: x\le{1} \text{ অথবা} \ x\ge{3}\right\}\)
সংখ্যারেখাঃ
realNumber
বুটেক্সঃ ২০০৭-২০০৮ ।

\(Q.2.ii.(aq)\) \(x^2-11x+24\lt{0}\)
উত্তরঃ \(S=\left\{x\in{\mathbb{R}}: 3\lt{x}\lt{8}\right\}\)
সংখ্যারেখাঃ
realNumber

\(Q.2.ii.(ar)\) \(2x^2+3x-5\le{0}\)
উত্তরঃ \(S=\left\{x\in{\mathbb{R}}: -\frac{5}{2}\le{x}\le{1}\right\}\)
সংখ্যারেখাঃ
realNumber

\(Q.2.ii.(as)\) \((x+2)(4x-3)\ge{0}\)
উত্তরঃ \(S=\left\{x\in{\mathbb{R}}: x\le{-2} \text{ অথবা} \ x\ge{\frac{3}{4}}\right\}\)
সংখ্যারেখাঃ
realNumber
রুয়েটঃ ২০০৯-২০১০ ।

\(Q.2.ii.(at)\) \(x(x-1)(x+2)\gt{0}\)
উত্তরঃ \(S=\left\{x\in{\mathbb{R}}: -2\lt{x}\lt{0} \text{ অথবা} \ x\gt{1}\right\}\)
সংখ্যারেখাঃ
realNumber

\(Q.2.ii.(au)\) \(x(x-1)(x-2)\ge{0}\)
উত্তরঃ \(S=\left\{x\in{\mathbb{R}}: 0\le{x}\le{1} \text{ অথবা} \ x\ge{2}\right\}\)
সংখ্যারেখাঃ
realNumber

\(Q.2.ii.(av)\) \((2x+1)(x-1)(x-3)\le{0}\)
উত্তরঃ \(S=\left\{x\in{\mathbb{R}}: x\le{-\frac{1}{2}} \text{ অথবা} \ 1\le{x}\le{3}\right\}\)
সংখ্যারেখাঃ
realNumber
সকলঃ ২০১৮ ।

\(Q.2.ii.(aw)\) \(\frac{x(x-4)}{x-5}\lt{0}\)
উত্তরঃ \(S=\left\{x\in{\mathbb{R}}: x\lt{0} \text{ অথবা} \ 4\lt{x}\lt{5}\right\}\)
সংখ্যারেখাঃ
realNumber

\(Q.2.ii.(ax)\) \(\frac{x+2}{x+1}\gt{\frac{x-3}{x-4}}\)
উত্তরঃ \(S=\left\{x\in{\mathbb{R}}: -1\lt{x}\lt{4}\right\}\)
সংখ্যারেখাঃ
realNumber
সিঃ ২০১৯ ।

\(Q.2.ii.(ay)\) \(\frac{2x+3}{x-3}\lt{\frac{x+3}{x-1}}\)
উত্তরঃ \(S=\left\{x\in{\mathbb{R}}: 1\lt{x}\lt{3}\right\}\)
সংখ্যারেখাঃ
realNumber
চঃ ২০১৭ ।

\(Q.2.ii.(az)\) \(\frac{3x+4}{5x+3}\lt{\frac{x+2}{2x+3}}\)
উত্তরঃ \(S=\left\{x\in{\mathbb{R}}: -\frac{3}{2}\lt{x}\lt{-\frac{3}{5}}\right\}\)
সংখ্যারেখাঃ
realNumber
চঃ ২০১৭ ।

\(Q.2.ii.(ba)\) \(\frac{(2x-3)(x-2)^2}{x+1}\gt{0}\)
উত্তরঃ \(S=\left\{x\in{\mathbb{R}}: x\lt{-1} \text{ অথবা} \ x\gt{\frac{3}{2}}, \ x\ne{2}\right\}\)
সংখ্যারেখাঃ
realNumber
চঃ ২০১৭ ।

\(Q.2.ii.(bb)\) \(\frac{1}{x+1}\gt{\frac{2}{x-1}}\)
উত্তরঃ \(S=\left\{x\in{\mathbb{R}}: x\lt{-3}\right\}\)
সংখ্যারেখাঃ
realNumber
দিঃ ২০১৬ ।

\(Q.2.ii.(bc)\) \(|x^2-1|\le{3}\)
উত্তরঃ \(S=\left\{x\in{\mathbb{R}}: -2\le{x}\le{2}\right\}\)
সংখ্যারেখাঃ
realNumber

\(Q.2.ii.(bd)\) \(|x|\le{5}\)
উত্তরঃ \(S=\left\{x\in{\mathbb{R}}: -5\le{x}\le{5}\right\}\)
সংখ্যারেখাঃ
realNumber

\(Q.2.ii.(be)\) \(2\lt{|x|}\lt{3}\)
উত্তরঃ \(S=\left\{x\in{\mathbb{R}}: -3\lt{x}\lt{-2} \text{ অথবা} \ 2\lt{x}\lt{3}\right\}\)
সংখ্যারেখাঃ
realNumber

\(Q.2.ii.(bf)\) \(|x-3|\lt{2}\)
উত্তরঃ \(S=\left\{x\in{\mathbb{R}}: 1\lt{x}\lt{5}\right\}\)
সংখ্যারেখাঃ
realNumber

\(Q.2.ii.(bg)\) \(\frac{x}{x^2-1}\lt{\frac{2x}{x+1}}\)
উত্তরঃ \(S=\left\{x\in{\mathbb{R}}: x\lt{-1} \text{ অথবা} \ x\gt{\frac{3}{2}}\right\}\)
সংখ্যারেখাঃ
realNumber

\(Q.2.ii.(bh)\) \(\frac{2}{3x-2}\lt{\frac{3}{4x-3}}\)
উত্তরঃ \(S=\left\{x\in{\mathbb{R}}: 0\lt{x}\lt{\frac{2}{3}} \text{ অথবা} \ x\gt{\frac{3}{4}}\right\}\)
সংখ্যারেখাঃ
realNumber

\(Q.2.ii.(bi)\) \(\frac{2x+1}{x-1}+\frac{3x-2}{x-3}\gt{5}\)
উত্তরঃ \(S=\left\{x\in{\mathbb{R}}: 1\lt{x}\lt{\frac{8}{5}} \text{ অথবা} \ x\gt{3}\right\}\)
সংখ্যারেখাঃ
realNumber

\(Q.2.ii.(bj)\) \(|x^2-2x|\lt{x}\)
উত্তরঃ \(S=\left\{x\in{\mathbb{R}}: 1\lt{x}\lt{3}\right\}\)
সংখ্যারেখাঃ
realNumber

\(Q.2.ii.(bk)\) \(\left|\frac{x+2}{x-1}\right|\gt{2}\)
উত্তরঃ \(S=\left\{x\in{\mathbb{R}}: x\lt{-5}, \ -2\lt{x}\lt{-1} \text{ অথবা} \ x\ge{1}\right\}\)
সংখ্যারেখাঃ
realNumber

\(Q.2.ii.(bl)\) \(\left|\frac{2x-1}{x-2}\right|\le{1}\)
উত্তরঃ \(S=\left\{x\in{\mathbb{R}}: -1\le{x}\le{1}\right\}\)
সংখ্যারেখাঃ
realNumber

সংখ্যারেখার সাহায্যে সমাধান করঃ
\(Q.2.iii.(a)\) \(\left|\frac{2x}{x-2}\right|\le{1}\)
উত্তরঃ \(S=\left\{x\in{\mathbb{R}}: -2\le{x}\le{\frac{2}{3}}\right\}\)

সংখ্যারেখার সাহায্যে সমাধান করঃ
\(Q.2.iii.(b)\) \(|x+1|\le{|x-1|}\)
উত্তরঃ \(S=\left\{x\in{\mathbb{R}}: x\le{0}\right\}\)

\(Q.2.iii.(c)\) \(\frac{x^2(x-1)}{x+1}\gt{0}\)
উত্তরঃ \(S=\left\{x\in{\mathbb{R}}: x\lt{-1} \text{ অথবা} \ x\gt{1}\right\}\)

\(Q.2.iii.(d)\) \(\frac{3x+4}{5x+3}\le{\frac{x+2}{2x+3}}\)
উত্তরঃ \(S=\left\{x\in{\mathbb{R}}: -\frac{3}{2}\le{x}\le{-\frac{3}{5}}\right\}\)

\(Q.2.iii.(e)\) \(\frac{2x+3}{x-3}\lt{\frac{x+3}{x-1}}\)
উত্তরঃ \(S=\left\{x\in{\mathbb{R}}: 1\lt{x}\lt{3}\right\}\)

\(Q.2.iii.(f)\) \(|x-5|\gt{4}\)
উত্তরঃ \(S=\left\{x\in{\mathbb{R}}: x\lt{1} \text{ অথবা} \ x\gt{9}\right\}\)

\(Q.2.iii.(g)\) \(|x|\le{4}\)
উত্তরঃ \(S=\left\{x\in{\mathbb{R}}: -4\le{x}\le{4}\right\}\)

নিচের অসমতাগুলির সমাধান সেটের লেখচিত্র অঙ্কন করঃ
\(Q.2.iv.(a)\) \(2x+3y\lt{6}\)

নিচের অসমতাগুলির সমাধান সেটের লেখচিত্র অঙ্কন করঃ
\(Q.2.iv.(b)\) \(2x-y\gt{6}\)

\(Q.2.iv.(c)\) \(3x-2y\le{12}\)

\(Q.2.iv.(d)\) \(x-y\ge{-10}\)

\(Q.2.iv.(e)\) \(y\ge{2x}\)

\(Q.2.iv.(f)\) \(y\le{x}\)

\(Q.2.iv.(g)\) \(x\gt{3}\)

\(Q.2.iv.(h)\) \(y\le{-2}\)

Read Board Question2
Q.3-এর বর্ণনামূলক প্রশ্নসমূহ
\(Q.3.(i)\) \(f(x)=x-1\) হলে, \(-2\lt{2-f(x)}\lt{8}\) অসমতাকে পরমমান চিহ্নের সাহায্যে প্রকাশ কর। যেখানে, \(x\in{\mathbb{R}}\)।
উত্তরঃ \(|x|\lt{5}\)
ঢাঃ ২০১৭ ।

\(Q.3.(ii)\) \(f(x)=3x+1\) হলে, \(2|f(x-2)|\le{1}\) এর সমাধান সেট সংখ্যারেখায় দেখাও।
উত্তরঃ \(\left\{x\in{\mathbb{R}}:\frac{3}{2}\le{x}\le{\frac{11}{6}}\right\}\)
সংখ্যারেখাঃ
realNumber
ঢাঃ,দিঃ,যঃ,সিঃ ২০১৮ ।

\(Q.3.(iii)\) \(f(x)=x-1\) হলে, \(|3f(x)-1|\lt{2}\) অসমতাকে সমাধান কর এবং সমাধান সেট সংখ্যারেখায় দেখাও। যেখানে, \(x\in{\mathbb{R}}\)।
উত্তরঃ \(S=\left\{x\in{\mathbb{R}}: \frac{2}{3}\lt{x}\lt{2}\right\}\)
সংখ্যারেখাঃ
realNumber
ঢাঃ ২০১৭ ।

\(Q.3.(iv)\) \(p=x-5\) হলে, \(\frac{1}{|p|}\ge{3}\) অসমতাটির সমাধান সেট নির্ণয় করে সংখ্যারেখায় দেখাও। যেখানে, \(x\in{\mathbb{R}}\) এবং \(x\ne{5}\)।
উত্তরঃ \(S=\left\{x\in{\mathbb{R}}: \frac{14}{3}\le{x}\le{\frac{16}{3}} \text{ এবং} \ x\ne{5}\right\}\)
সংখ্যারেখাঃ
realNumber
রাঃ ২০১৭ ।

\(Q.3.(v)\) \(|x-1|\lt{\frac{1}{10}}\) হলে, দেখাও যে, \(|x^2-1|\lt{\frac{21}{100}}\)।
ঢাঃ ২০১৭,২০১৪,২০১২,২০১০,২০০৬; চঃ২০১৫; সিঃ ২০১৫,২০০৯; রাঃ ২০১০; দীঃ ২০১৫,২০১২; যঃ২০০৮; চঃ ২০১৬,২০১১,২০০৮; বঃ ২০১৩,২০০৮; মাঃ ২০১৪,২০১১; রুয়েটঃ ২০১২-২০১৩ ।

\(Q.3.(vi)\) \(f(x)=ax+by+c\) এবং \(|f(x)-1|\lt{\frac{1}{11}}\) হলে, প্রমাণ কর যে, \(|\{f(x)\}^2-1|\lt{\frac{23}{121}}\)। যেখানে, \(a=1, \ b=c=0\)
কুঃ ২০১৭ ।

\(Q.3.(vii)\) \(f(x)=|x-3|\) এবং \(f(x)\lt{\frac{1}{5}}\) হলে, দেখাও যে, \(f(x^2-6)\lt{\frac{31}{25}}\)।
সিঃ ২০১৭ ।

\(Q.3.(viii)\) \(f(x)=|x-3|\) এবং \(f(x)\lt{\frac{1}{7}}\) হলে, প্রমাণ কর যে, \(|x^2-9|\lt{\frac{43}{49}}\)।
বঃ ২০১৭ ।

\(Q.3.(ix)\) \(|x-3|\lt{\frac{1}{5}}\) হয় তবে দেখাও যে, \(|x^2-8|\lt{\frac{56}{25}}\)।
চঃ ২০১৯ ।

দেখাও যে, নিম্নলিখিত সংখ্যাগুলি অমূলদ।
\(Q.3.x.(a)\) \(\sqrt{3}-\sqrt{5}\)

\(Q.3.(xi)\) \(x, \ y\) বাস্তব সংখ্যা এবং \(x\sqrt{2}-y\sqrt{3}=0\) হলে, দেখাও যে, \(x=y=0\)।

\(Q.3.(xii)\) দেখাও যে, দুইটি ভিন্ন মূলদ সংখ্যার মধ্যে অন্য মূলদ সংখ্যা আছে।

\(Q.3.(xiii)\) \(a\lt{b}\) এবং \(k\) ধনাত্মক মূলদ সংখ্যা হলে, প্রমাণ কর যে, \(a\lt{\frac{a+bk}{1+k}}\lt{b}\)।

নিম্নলিখিত সংখ্যাগুলিকে ভগ্নাংশ আকারে প্রকাশ করে দেখাও যে, সংখ্যাগুলি মূলদ।
\(Q.3.xiv.(a)\) \(0.66666....\)

নিম্নলিখিত সংখ্যাগুলিকে ভগ্নাংশ আকারে প্রকাশ করে দেখাও যে, সংখ্যাগুলি মূলদ।
\(Q.3.xiv.(b)\) \(0.56666....\)

\(Q.3.xiv.(c)\) \(9.727272...\)

\(Q.3.xiv.(d)\) \(4.272727...\)

\(Q.3.xiv.(e)\) \(0.6333...\)

\(Q.3.xiv.(f)\) \(3.242424.....\)

\(Q.3.xiv.(g)\) \(0.494949....\)

\(Q.3.xiv.(h)\) \(4.5\)

\(Q.3.xiv.(i)\) \(10.008\)

\(Q.3.xiv.(j)\) \(1.3333....\)

\(Q.3.xiv.(k)\) \(3.875875875....\)

\(Q.3.xiv.(l)\) \(0.08323232....\)

\(Q.3.xiv.(m)\) \(5.808080....\)

\(Q.3.(xv)\) \(|x-1|\lt{2}\) হলে, দেখাও যে, \(|x^2-1|\lt{8}\)।

\(Q.3.(xvi)\) \(|x-1|\lt{\frac{1}{2}}\) হলে, দেখাও যে, \(|x^2-1|\lt{\frac{5}{4}}\)।

\(Q.3.(xvii)\) \(f(x)=|bx-c|, \ b=1, \ c=2\) এবং \(f(x)\lt{\frac{1}{4}}\) হলে, দেখাও যে, \(f(x^2-2)\lt{\frac{17}{16}}\)।
রাঃ ২০১৯।

\(Q.3.(xviii)\) \(|x-1|\lt{\frac{1}{2}}\) হলে, দেখাও যে, \(|x^3-1|\lt{\frac{19}{8}}\)।

\(Q.3.(xix)\) \(|x-1|\lt{3}\) হলে, দেখাও যে, \(|x^3-1|\lt{63}\)।

\(Q.3.(xx)\) দেখাও যে, যেকোনো বিজোড় সংখ্যার বর্গ একটি বিজোড় সংখ্যা।

\(Q.3.(xxi)\) দেখাও যে, \(\sqrt{p}\) একটি অমূলদ সংখ্যা, যেখানে \(p\) একটি মৌলিক সংখ্যা।

\(Q.3.(xxii)\) \(x\in{\mathbb{R}}\) এর সীমা নির্ণয় করঃ যেখানে, \(x^2+6x-27\gt{0}\) এবং \(3x-x^2+4\gt{0}\)
উত্তরঃ \(S=\left\{x\in{\mathbb{R}}: 3\lt{x}\lt{4}\right\}\)

নিচের প্রত্যেক অসমতাযুগলের সমাধান সেটের লেখচিত্র অঙ্কন করঃ
\(Q.3.xxiii.(a)\) \(x+y-4\le{0}\) এবং \(2x-y-3\ge{0}\)

নিচের প্রত্যেক অসমতাযুগলের সমাধান সেটের লেখচিত্র অঙ্কন করঃ
\(Q.3.xxiii.(b)\) \(x+y-3\gt{0}\) এবং \(2x-y-5\gt{0}\)

\(Q.3.xxiii.(c)\) \(3x-3y\gt{5}\) এবং \(x+3y\le{9}\)

\(Q.3.xxiii.(d)\) \(5x-3y-9\gt{0}\) এবং \(3x-2y\ge{5}\)

\(Q.3.xxiii.(e)\) \(2x-3y-1\ge{0}\) এবং \(2x+3y-7\le{0}\)

\(Q.3.xxiii.(f)\) \(x+2y-4\gt{0}\) এবং \(2x-y-3\gt{0}\)

\(Q.3.xxiii.(g)\) \(2x-2y\gt{5}\) এবং \(x+2y\le{8}\)

\(Q.3.xxiii.(h)\) \(x\ge{3}\) এবং \(y\le{2}\)

\(Q.3.xxiii.(i)\) \(y\le{x}\) এবং \(x+y\le{1}\)

\(x\) এর কোন মানের জন্য নিচের রাশিগুলির মান বাস্তব হবে?
\(Q.3.xxiv.(a)\) \(\frac{x+2}{|x+1|}\)
উত্তরঃ \(\mathbb{R}-\{-1\}\)

\(x\) এর কোন মানের জন্য নিচের রাশিগুলির মান বাস্তব হবে?
\(Q.3.xxiv.(b)\) \(\frac{x-1}{|x|}\)
উত্তরঃ \(\mathbb{R}-\{0\}\)

\(Q.3.xxv\) সম্পূর্ণতা ধর্ম বলতে কি বুঝ? প্রমাণ কর যে মূলদ সংখ্যার সেটে সম্পূর্ণতা ধর্ম খাটে না।

\(Q.3.xxvi\) \(A=\left\{x: x=3n, n\in{\mathbb{N}}\right\}\) হলে, দেখাও যে, \(A\) গুণ প্রক্রিয়ার জন্য আবদ্ধ।

\(Q.3.xxvii\) \(\frac{10}{11}\lt{x}\lt{\frac{12}{11}}\) অসমতাকে পরম মান চিহ্নের সাহায্যে প্রকাশ কর এবং তার সাহায্যে দেখাও যে, \(|x^2-1|\lt{\frac{23}{121}}\)

\(Q.3.(xxviii)\) \(|x-4|\lt{\frac{1}{7}}\) হলে, দেখাও যে, \(|x^2-15|\lt{\frac{106}{49}}\)।

\(Q.3.(xxix)\) \(f(x)=|x-4|\) এবং \(f(x)\lt{\frac{1}{7}}\) হলে, দেখাও যে, \(f(x^2-11)\lt{\frac{106}{49}}\)।

\(Q.3.(xxx)\) \(f(x)=|x-4|\) এবং \(f(x)\lt{\frac{1}{7}}\) হলে, দেখাও যে, \(f(x)\times{f(x+15)}\lt{\frac{106}{49}}\)।

\(Q.3.(xxxi)\) \(25x^2-150x+224\lt{0}\) হলে, দেখাও যে, \(|x^2-9|\lt{\frac{31}{25}}\)।

\(Q.3.(xxxii)\) \(f(x)=x+1\) হলে, \(|2f(x)-1|\lt{3}\) অসমতাকে সমাধান কর এবং সমাধান সেট সংখ্যারেখায় দেখাও। যেখানে, \(x\in{\mathbb{R}}\)।
উত্তরঃ \(S=\left\{x\in{\mathbb{R}}: -2\lt{x}\lt{1}\right\}\)
সংখ্যারেখাঃ
realNumber
সিঃ ২০০৭, ২০০৪ ।

\(Q.3.(xxxiii)\) \(f(x)=3x+1\) হলে, \(\frac{1}{|f(x)|}\ge{5}, \ x\ne{-\frac{1}{3}}\) অসমতাকে সমাধান কর এবং সমাধান সেট সংখ্যারেখায় দেখাও।
উত্তরঃ \(S=\left\{x\in{\mathbb{R}}: -\frac{2}{5}\le{x}\le{-\frac{4}{15}}, \ x\ne{-\frac{1}{3}}\right\}\)
সংখ্যারেখাঃ
realNumber
দিঃ ২০১৯; চঃ২০১৩,২০০১; যঃ ২০০৮; সিঃ ২০০৬; রুয়েটঃ ২০১০-২০১১ ।

\(Q.3.(xxxiv)\) \(f(x)=|5x-3|\) হলে, \(\frac{1}{f(x)}\ge{2}, \ x\ne{\frac{3}{5}}\) অসমতাকে সমাধান কর এবং সমাধান সেট সংখ্যারেখায় দেখাও।
উত্তরঃ \(S=\left\{x\in{\mathbb{R}}: \frac{1}{2}\le{x}\le{\frac{7}{10}}, \ x\ne{\frac{3}{5}}\right\}\)
সংখ্যারেখাঃ
realNumber
ঢাঃ ২০১৯ ।

Read Board Question3
Q.4-এর বর্ণনামূলক প্রশ্নসমূহ
বাস্তব সংখ্যার সেট \(\mathbb{R}\) এর উপসেট \(A\) হলে, নিম্নলিখিত সেটগুলির সুপ্রিমাম ও ইনফিমাম বিদ্যমান থাকলে নির্ণয় করঃ
\(Q.4.i.(a)\) \(A=\{1, \ 2, \ 3, \ 4, \ 5\}\)
উত্তরঃ সুপ্রিমাম \(Sup(A)=5;\) ইনফিমাম \(Inf(A)=1\)

বাস্তব সংখ্যার সেট \(\mathbb{R}\) এর উপসেট \(A\) হলে, নিম্নলিখিত সেটগুলির সুপ্রিমাম ও ইনফিমাম বিদ্যমান থাকলে নির্ণয় করঃ
\(Q.4.i.(b)\) \(A=\left\{1-\frac{1}{3^n}: n\in{\mathbb{N}}\right\}\)
উত্তরঃ সুপ্রিমাম \(Sup(A)=1;\) ইনফিমাম \(Inf(A)=\frac{2}{3}\)

\(Q.4.i.(c)\) \(A=\left\{\frac{1}{5n}: n\in{\mathbb{Z}}, \ n\ne{0}\right\}\)
উত্তরঃ সুপ্রিমাম \(Sup(A)=\frac{1}{5};\) ইনফিমাম \(Inf(A)=-\frac{1}{5}\)

\(Q.4.i.(d)\) \(A=\left\{n\in{\mathbb{R}}: -5\lt{x}\lt{3}\right\}\)
উত্তরঃ সুপ্রিমাম \(Sup(A)=3;\) ইনফিমাম \(Inf(A)=-5\)

\(Q.4.i.(e)\) \(A=\left\{n\in{\mathbb{R}}: x=2^n, \ n\in{\mathbb{N}}\right\}\)
উত্তরঃ সুপ্রিমাম নেই; ইনফিমাম \(Inf(A)=2\)

\(Q.4.i.(f)\) \(A=\left\{x:(x-1)(x-2)\le{0} , \ x\in{\mathbb{R}}\right\}\)
উত্তরঃ ইনফিমাম \(Inf(A)=1;\) সুপ্রিমাম \(Sup(A)=2\)

\(Q.4.i.(g)\) \(A=\left\{x\in{\mathbb{R}}:|x-3|\le{4}\right\}\)
উত্তরঃ সুপ্রিমাম \(Sup(A)=7;\) ইনফিমাম \(Inf(A)=-1\)

\(Q.4.i.(h)\) \(A=\left\{x:5x^2-16x+3\lt{0} , \ x\in{\mathbb{R}}\right\}\)
উত্তরঃ ইনফিমাম \(Inf(A)=\frac{1}{5};\) সুপ্রিমাম \(Sup(A)=3\)
কুয়েটঃ ২০৪-২০০৫; বুটেক্সঃ ২০১২-২০১৩ ।

\(Q.4.i.(i)\) \(A=\left\{a:a\in{\mathbb{Z}} \text{ এবং} |g(a)|\lt{4}\right\},\) যেখানে, \(g(x)=2x-1, \ x\in{\mathbb{R}}\)।
উত্তরঃ সুপ্রিমাম \(Sup(A)=2;\) ইনফিমাম \(Inf(A)=-1\)
কুঃ ২০১৯ ।

\(Q.4.i.(j)\) \(A=\left\{x:x\in{\mathbb{R}}, \ -9\lt{5x+1}\lt{16}\right\}\)
উত্তরঃ সুপ্রিমাম \(Sup(A)=3;\) ইনফিমাম \(Inf(A)=-2\)
যঃ ২০১৯ ।

\(Q.4.i.(k)\) \(A=\{0, \ 1, \ 2, \ 3, \ 4, \ 5\}\)
উত্তরঃ সুপ্রিমাম \(Sup(A)=5;\) ইনফিমাম \(Inf(A)=1\)
টেক্সটাইলঃ ২০০৯-২০১০ ।

\(Q.4.i.(l)\) \(A=\left\{x:x^2-5x+6\lt{0} , \ x\in{\mathbb{R}}\right\}\)
উত্তরঃ সুপ্রিমাম \(Sup(A)=3;\) ইনফিমাম \(Inf(A)=2\)
চঃ ২০০০; কুয়েটঃ ২০০৪-২০০৫ ।

\(Q.4.i.(m)\) \(A=\left\{\frac{1}{n}: n\in{\mathbb{N}}\right\}\)
উত্তরঃ সুপ্রিমাম \(Sup(A)=1;\) ইনফিমাম \(Inf(A)=0\)

\(Q.4.i.(n)\) \(A=\left\{n-1: n\in{\mathbb{N}}\right\}\)
উত্তরঃ সুপ্রিমাম \(\left(Sup(A)\right)\)নেই; ইনফিমাম \(Inf(A)=0\)

\(Q.4.i.(o)\) \(A=\left\{n: n\in{\mathbb{Z}}\right\}\)
উত্তরঃ বৃহত্তম নিম্নসীমা \(Inf(A)\) এবং ক্ষুদ্রত্তম ঊর্ধসীমা \(\left(Sup(A)\right)\) নেই।

\(Q.4.i.(p)\) দেখাও যে, \(A=\left\{x: x\ge{0}\right\}\) হলে,
\((i)\) \(A\) এর নিম্নসীমা \(0\) ।
\((ii)\) \(A\) এর একটি ঊর্ধবসীমা \(2\) ।
\((iii)\) \(A\) এর একটি নিম্নসীমা \(-1\) নয়।
\((iv)\) কোনো ঋনাত্মক সংখ্যাই \(A\) এর একটি ঊর্ধবসীমা নয়।

\(Q.4.i.(q)\) \(A=\left\{-\frac{n+1}{n}: n\in{\mathbb{N}}\right\}.\)
উত্তরঃ সুপ্রিমাম \(Sup(A)=-1\) এবং ইনফিমাম \(Inf(A)=-2\)

\(Q.4.i.(r)\) \(A=\left\{\frac{3n+2}{2n+1}: n\in{\mathbb{N}}\right\}.\)
উত্তরঃ সুপ্রিমাম \(Sup(A)=\frac{5}{3}\) এবং ইনফিমাম \(Inf(A)=\frac{3}{2}\)

\(Q.4.i.(s)\) \(A=\left\{x\in{\mathbb{R}}: 2\le{x}\lt{8}\right\}.\)
উত্তরঃ সুপ্রিমাম \(Sup(A)=8\) এবং ইনফিমাম \(Inf(A)=2\)

\(Q.4.i.(t)\) \(S=\left\{x\in{\mathbb{R}}: 2x^2-7x+3\le{0}\right\}.\)
উত্তরঃ সুপ্রিমাম \(Sup(S)=3\) এবং ইনফিমাম \(Inf(S)=\frac{1}{2}\)

\(Q.4.(ii)\) বাস্তব সংখ্যায় বিপরীত এর অস্তিত্বের ব্যাখ্যা কর।
রাঃ ২০১৭ ।

\(Q.4.(iii)\) সকল বাস্তব সংখ্যা \(a\) এর জন্য প্রমাণ করঃ \(\sqrt{a^2}=|a|\)

\(Q.4.(iv)\) যদি \(a, \ b\in{\mathbb{R}}\) হয় তবে দেখাও যে, \((-a)(-b)=ab\) এবং \((ab)^{-1}=(b)^{-1}(a)^{-1}, \ (a\ne{0}, \ b\ne{0})\)
কুঃ ২০১২,২০০৭; সিঃ ২০১৬,২০১১ ।

\(Q.4.(v)\) যদি \(a, \ b\in{\mathbb{R}}\) এবং \(ab=0\) হয় , তবে দেখাও যে, \(a=0\) অথবা \(b=0\)।
সিঃ ২০১৩ ।

\(Q.4.(vi)\) যদি \(a, \ b, \ c\in{\mathbb{R}}\) এবং \(a+b=a+c\) হয় , তবে দেখাও যে, \(b=c\)
ঢাঃ ২০১২,২০০৭; যঃ ২০১১; সিঃ ২০০৯; রাঃ ২০১১; বঃ ২০০৭ ।

\(Q.4.(vii)\) যদি \(a, \ b, \ c\in{\mathbb{R}}, \ ac=bc\) এবং \(c\ne{0}\) হয় , তবে দেখাও যে, \(a=b\)
কুঃ ২০১৬,২০১৪,২০০৯; চঃ ২০১০,২০০৬; দিঃ ২০১৩,২০১০; বঃ ২০১৩ ।

\(Q.4.(viii)\) যদি \(a, \ b\in{\mathbb{R}}\) হয় , তবে দেখাও যে, \(a(-b)=(-a)b=-(ab)\)
বঃ ২০১১ ।

\(Q.4.(ix)\) যদি \(a\gt{b}\) হয় , তবে দেখাও যে, \(a+c\gt{b+c}\) যেখানে, \(a, \ b, \ c\) বাস্তব সংখ্যা।
রাঃ ২০০৮; চঃ ২০১২ ।

\(Q.4.(x)\) যদি \(a, \ b, \ c\in{\mathbb{R}}\) এবং \(a\le{b}\) হয় , তবে দেখাও যে, \(ac\le{bc}\) যখন \(c\gt{0}\) এবং \(ac\ge{bc}\) যখন \(c\lt{0}\)।

\(Q.4.(xi)\) যদি \(a\in{\mathbb{R}}\) হয় , তবে দেখাও যে, \(-1.a=-a\)।

\(Q.4.(xii)\) যদি \(a\in{\mathbb{R}}\) হয় , তবে দেখাও যে, \(-(-a)=a\)।

\(Q.4.(xiii)\) যদি \(a, \ b\in{\mathbb{R}}\) হয় , তবে দেখাও যে, \(-(a+b)=-a-b\)।

\(Q.4.(xiv)\) যদি \(a, \ b, \ c\in{\mathbb{R}}\) এবং \(a\lt{b}\) হয় , তবে দেখাও যে, \(a+c\lt{b+c}\)।
রাঃ ২০০৮; চঃ ২০১২ ।

\(Q.4.(xv)\) যদি \(a, \ b, \ c\in{\mathbb{R}}\) এবং \(a\lt{b}, \ b\lt{c}\) হয় , তবে দেখাও যে, \(a\lt{c}\)।
রাঃ ২০০৮; চঃ ২০১২ ।

\(Q.4.(xvi)\) \(g(x)=2x-1, \ x\in{\mathbb{R}}\) এবং \(A=\left\{a: a\in{\mathbb{Z}} \text{ এবং} |g(a)|\lt{4}\right\}\) হলে, \(A\) সেটটির সুপ্রিমাম ও ইনফিমাম বের কর।
উত্তরঃ সুপ্রিমাম \(Sup(A)=2;\) ইনফিমাম \(Inf(A)=-1\)
কুঃ ২০১৯ ।

\(Q.4.(xvii)\) \(f(x)=5x+1\) হলে, \(S=\left\{x:x\in{\mathbb{R}}, \ -9\lt{f(x)}\lt{16} \right\}\) এর সুপ্রিমাম নির্ণয় কর।
উত্তরঃ সুপ্রিমাম \(Sup(A)=3\)
যঃ ২০১৯ ।

\(Q.4.(xviii)\) \(S\subset{\mathbb{R}}\) এর ক্ষেত্রে \(S=\left\{\frac{1}{n}: n\in{\mathbb{N}}\right\}\) এর বৃহত্তম নিম্নসীমা নির্ণয় কর।
উত্তরঃ ইনফিমাম \(Inf(A)=0\)
বঃ ২০১৯ ।

\(Q.4.(xix)\) যথাযথ কারণ উল্লেখ করে \(\left||3-5|-|7-12|\right|\) এর মান নির্ণয় কর।
উত্তরঃ \(3\)
কুঃ ২০০৬ ।

\(Q.4.(xx)\) প্রমাণ কর যে, \(|a-c|\le{|a-b|+|b-c|}\) যেখানে, \(a, \ b, \ c\in{\mathbb{R}}.\)

Read Board Question4
Q.5-এর সৃজনশীল প্রশ্নসমূহ
\(Q.5.(i)\) \(f(x)=x-1\)
\((a)\) \(|x-1|\le{\frac{1}{2}}\) অসমতাটি পরম মান চিহ্ন ব্যতীত প্রকাশ কর।
সিঃ ২০১৭ ।
\((b)\) \(\frac{1}{|3f(x)-2|}\gt{2}\) অসমতাটির সমাধান সেট সংখ্যারেখায় দেখাও।
\((c)\) \(5|f(x-2)|\lt{1}\) হলে, দেখাও যে, \(25|f(x-2)\times{f(x+2)}|\lt{21}\)
উত্তরঃ \((a) \ \frac{1}{2}\lt{x}\lt{\frac{3}{2}}\)
\((b) \ S=\left\{x\in{\mathbb{R}}: \frac{3}{2}\lt{x}\lt{\frac{11}{6}} \text{ এবং} \ x\ne{\frac{5}{3}}\right\}\)
realNumber

\(Q.5.(ii)\) \(f(x)=3x-x^2+4\) এবং
\(g(x)=x^2+6x-27\)
\((a)\) \(|x-5|=|2x-3|\) এর সমাধান সেট নির্ণয় কর।
\((b)\) \(f(x)\gt{0}\) অসমতাটি পরম মান চিহ্নের সাহয্যে প্রকাশ কর।
\((c)\) \(x\in{\mathbb{R}}\) এর সীমা নির্ণয় কর; যেখানে, \(f(x)\gt{0}\) এবং \(g(x)\gt{0}\) ।
উত্তরঃ \((a) \ S=\left\{-2, \ \frac{8}{3}\right\}\)
\((b) \ |2x-3|\lt{5}\)
\((c) \ 3\lt{x}\lt{4}\)

\(Q.5.(iii)\) \(f(x)=5x-1\)
\(g(x)=(x+1)^2\) এবং
\(h(x)=7x-3\)
\((a)\) মান নির্ণয় করঃ \(|-1-8|+|3-1|\) ।
\((b)\) সংখ্যারেখার সাহায্যে সমাধান করঃ \(|f(x)|+|h(x)|\le{3}\)।
\((c)\) সমাধান সেট নির্ণয় করঃ \(f(x)\lt{g(x)}\lt{h(x)}\) ।
উত্তরঃ \((a) \ 11\)
\((b) \ S=\left\{x\in{\mathbb{R}}: \frac{1}{2}\le{x}\le{\frac{7}{12}}\right\}\)
\((c) \ 2\lt{x}\lt{4}\)
realNumber

\(Q.5.(iv)\) \((1) \ |5-2x|\ge{4}\)
\((2) \ x+y-4\le{0}\) এবং \(2x-y-3\ge{0}\)
\((a)\) প্রমাণ কর যে, \(|a-c|\le{|a-b|+|b-c|}\) যেখানে, \(a, \ b, \ c\in{\mathbb{R}}.\)
\((b)\) \((1)\) এ উল্লেখিত অসমতাটির সমাধান সেট সংখ্যারেখায় দেখাও।
\((c)\) \((2)\) এ উল্লেখিত অসমতাযুগলের সমাধান সেটের লেখচিত্র অঙ্কন কর।
উত্তরঃ \((b) \ S=\left\{x\in{\mathbb{R}}: x\le{\frac{1}{2}} \text{ অথবা} \ x\ge{\frac{9}{2}}\right\}\)
realNumber

\(Q.5.(v)\) \((1) \ |x-1|\lt{\frac{1}{2}}\)
\((2)\) সকল \(a, \ b\in{\mathbb{R}}\) এর জন্য, \(|a+b|\le{|a|+|b|}\)
\((a)\) বাস্তব সংখ্যার সেট \(\mathbb{R}\) এর উপসেট \(S=\left\{x:5x^2-16x+3\lt{0}\right\}\) এর বৃহত্তম নিম্নসীমা \(\left(Inf(S)\right)\) নির্ণয় কর।
\((b)\) \((1)\) এর সাহায্যে দেখাও যে, \(|x^3-1|\lt{\frac{19}{8}}.\)
\((c)\) \((2)\) এ উল্লেখিত বাস্তব সংখ্যার পরমমানের ধর্মটি প্রমাণ কর।
উত্তরঃ \((a) \ \left(Inf(S)\right)=\frac{1}{5}\)

\(Q.5.(vi)\) \(f(x)=2+5x, \ g(x)=x-1\)
\((a)\) \(5x^2-19x-4\lt{0}\) এর সমাধান নির্ণয় কর।
\((b)\) \(\frac{1}{|g(x+2)|}\ge{3}\) অসমতাটির সমাধান সেট সংখ্যারেখায় দেখাও।
\((c)\) সংখ্যারেখার সাহায্যে \(|f(x)|\le{|g(x)|}\) এর সমাধান নির্ণয় কর।
উত্তরঃ \((a) \ S=\left\{x:-\frac{1}{5}\lt{x}\lt{4} , \ x\in{\mathbb{R}}\right\}\)
\((b) \ S=\left\{x\in{\mathbb{R}}: -\frac{4}{3}\le{x}\lt{-1} \text{ অথবা} \ -1\lt{x}\le{-\frac{2}{3}}\right\}\)
realNumber
\((c) \ -\frac{3}{4}\le{x}\le{-\frac{1}{6}}\)

\(Q.5.(vii)\) \(f(x)=x+1, \ g(y)=y-1\)
\((a)\) \(|g(y)|\le{\frac{1}{2}}\) অসমতাটি পরম মান চিহ্ন ব্যতীত প্রকাশ কর।
\((b)\) \(|g(x)|\lt{\frac{1}{9}}\) হলে দেখাও যে, \(|x^2-1|\lt{\frac{19}{81}}\)
\((c)\) \(g(x)+f(y-3)\gt{0}\) ও \(2f(x)+g(y-6)\gt{0}\) অসমতাযুগলের সমাধান সেটের লেখচিত্র অঙ্কন কর।
উত্তরঃ \((a) \ \frac{1}{2}\le{y}\le{\frac{3}{2}}\)

\(Q.5.(viii)\) \(f(x)=3x-4, \ g(x)=5x+6\)
\((a)\) যদি \(a\lt{b}\) হয় তবে দেখাও যে, \(a+c\lt{b+c};\) যেখানে, \(a, \ b, \ c\in{\mathbb{R}}.\)
\((b)\) \(\frac{1}{|f(x)|}\ge{5}\) অসমতাটির সমাধান সেট সংখ্যারেখায় দেখাও।
\((c)\) \(\frac{(x-1)f(x)}{g(x)}\lt{0}\) অসমতাটির সমাধান নির্ণয় কর।
উত্তরঃ \((b) \ \frac{19}{15}\le{x}\lt{\frac{4}{3}}\) অথবা, \(\frac{4}{3}\lt{x}\le{\frac{7}{5}}\)
realNumber
\((c) \ x\lt{-\frac{6}{5}}\) অথবা, \(1\lt{x}\lt{\frac{4}{3}}\)

\(Q.5.(ix)\) \(f(x)=x-2\) এবং \(a=7\)
\((a)\) \(|f(y)|\le{\frac{1}{a}}\) অসমতাটি পরম মান চিহ্ন ব্যতীত প্রকাশ কর।
\((b)\) \(|f(x)|\lt{\frac{1}{5}}\) হলে, \(|f(x)\times{f(x+4)}|\) এর মান নির্ণয় কর।
\((c)\) প্রমাণ কর যে, \(\sqrt{a}\) একটি অমূলদ সংখ্যা।
উত্তরঃ \((a) \ \frac{13}{7}\le{y}\le{\frac{15}{7}}\)
\((b) \ |f(x)\times{f(x+4)}|\lt{\frac{31}{25}}\)

\(Q.5.(x)\) \(f(x)=x^2+2x+2, \ g(x)=x-5\)
\((a)\) \(-1\lt{x+2}\le{5}\) অসমতাটির সমাধান সেট সংখ্যারেখায় দেখাও।
\((b)\) \(f(x)\lt{10}\) অসমতাকে পরম মান চিহ্নের সাহয্যে প্রকাশ কর।
\((c)\) \(|g(x)|-2x\gt{4}\) অসমতাটির সমাধান সেট নির্ণয় কর।
উত্তরঃ \((a) \ S=\left\{x\in{\mathbb{R}}: -3\lt{x}\le{3}\right\}\)
realNumber
\((b) \ |x+1|\lt{3}\)
\((c) \ S=\left\{x\in{\mathbb{R}}: x\lt{\frac{1}{3}}\right\}\)

\(Q.5.(xi)\) \(L=a+b, \ \frac{M}{a}=\frac{N}{b}=1, \ a, \ b\in{\mathbb{R}}\) এবং \(f(x)=3x-5\)
\((a)\) \([-4, 4]\) সেটটিকে অসমতা আকারে প্রকাশ করে সুপ্রিমাম \(Sup\) নির্ণয় কর।
\((b)\) \(\frac{1}{|f(x)|}\gt{2}\) অসমতাটির সমাধান সেট সংখ্যারেখায় দেখাও; যেখানে, \(f(x)\ne{0}.\)
\((c)\) প্রমাণ কর যে, \(|L|\le{|M|+|N|.}\)
উত্তরঃ \((a) \ 4\)
\((b) \ S=\left\{x\in{\mathbb{R}}: \frac{3}{2}\le{x}\le{\frac{11}{6}}\right\}\)
realNumber

\(Q.5.(xii)\) \(f(x)=x^2-9\)
\((a)\) \(S=\left\{x\in{\mathbb{R}}: -3\lt{x-2}\le{5}\right\}\) কে ব্যবধি আকারে প্রকাশ কর।
\((b)\) \(f(x)\lt{5x-13}\) অসমতাকে পরম মান চিহ্নের সাহয্যে প্রকাশ কর।
\((c)\) \(|f(x)|\le{5(x-1)}\) অসমতাটির সমাধান সেট সংখ্যারেখার সাহয্যে প্রকাশ কর।
উত্তরঃ \((a) \ ]-1, 7]\)
\((b) \ |2x-5|\lt{7}\)
\((c) \ S=\left\{x\in{\mathbb{R}}: 3\le{x}\le{7}\right\}\)
realNumber

\(Q.5.(xiii)\) \(a\in{\mathbb{R}}, \ b\in{\mathbb{Q^{\prime}}}\) এবং \(\frac{M}{a}=\frac{N}{b}=1.\)
\((a)\) \(x\) এর বাস্তব মানের জন্য \(1\le{|x-3|}\le{8}\) অসমতাটির সমাধান নির্ণয় কর।
\((b)\) প্রমাণ কর যে, \(|M-N|\ge{||a|-|b||.}\)
\((c)\) স্বীকার্যের সাহায্যে প্রমাণ কর যে, \(M+N\in{\mathbb{Q^{\prime}}}.\)
উত্তরঃ \((a) \ S=\left\{x\in{\mathbb{R}}: -5\le{x}\le{2}\right\}\cup{\left\{x\in{\mathbb{R}}: 4\le{x}\le{11}\right\}}\)

\(Q.5.(xiv)\) \(a\in{\mathbb{R}}, \ b\in{\mathbb{Q^{\prime}}}\) এবং \(P=x-1.\)
\((a)\) \(x\) এর বাস্তব মানের জন্য \(0\lt{|x-2|}\lt{5}\) অসমতাটির সমাধান নির্ণয় কর।
\((b)\) স্বীকার্যের সাহায্যে প্রমাণ কর যে, \(ab\in{\mathbb{Q^{\prime}}}.\)
\((c)\) সংখ্যারেখার সাহায্যে \(|P+2|\le{|P|}\) এর সমাধান নির্ণয় কর।
উত্তরঃ \((a) \ S=\left\{x\in{\mathbb{R}}: -3\le{x}\le{2}\right\}\cup{\left\{x\in{\mathbb{R}}: 2\le{x}\le{7}\right\}}\)
\((c) \ S=\left\{x\in{\mathbb{R}}: x\le{0}\right\}\)

\(Q.5.(xv)\)
realNumber
এবং \(f(x)=|x-1|-\frac{1}{7}.\)
\((a)\) \(f(x)\) এর সর্বনিম্ন মান নির্ণয় কর।
\((b)\) সংখ্যারেখায় নির্দেশিত অংশটির সমাধান সেটকে পরমমান চিহ্নের সাহায্যে প্রকাশ কর।
\((c)\) \(f(x)\lt{0}\) হলে প্রমাণ কর যে, \(|x^2-1|\lt{\frac{15}{49}}\)
উত্তরঃ \((a) \ -\frac{1}{7}\)
\((b) \ |3x+2|\lt{7}\)

\(Q.5.(xvi)\)
realNumber
\((a)\) প্রমাণ কর যে, \(|x+a|+|x-a|\ge{|2x|}.\)
\((b)\) \(PQRS\) আয়তক্ষেত্রটির ক্ষেত্রফল \(8\) বর্গ এককের অধীক না হওয়ার শর্তকে পরমমান চিহ্নের সাহায্যে প্রকাশ কর।
\((c)\) \(AB\le{AC}\le{BC}\) হলে, \(ABC\) ত্রিভুজটি অঙ্কনের শর্তে \(x\) এর মান সংখ্যারেখার সাহায্যে নির্ণয় কর।
উত্তরঃ \((b) \ |3y-1|\le{4}\)
\((c) \ 0\lt{x}\lt{3}\)

\(Q.5.(xvii)\) \(f(x)=x-1, \ g(x)=x+1, \ h(x)=x^2\)
\((a)\) দেখাও যে, \(f(x)g(x)+h(x)+1\lt{0}\) এর কোন বাস্তব সমাধান নেই।
\((b)\) \(f(x)+g(y)-3\gt{0}\) ও \(2f(x)-g(y)-2\gt{0}\) অসমতাযুগলের সমাধান সেটের লেখচিত্র অঙ্কন কর।
\((c)\) সংখ্যারেখার সাহায্যে \(\frac{h(x)f(x)}{g(x)}\gt{0}\) এর সমাধান নির্ণয় কর।
উত্তরঃ \((c) \ x\lt{-1} \text{ অথবা} \ x\gt{1}\)

\(Q.5.(xviii)\) \(x=a+5, \ a\in{\mathbb{R}}\)
\((a)\) \(S=\left\{x\in{\mathbb{R}}: x^2-7x-6\gt{0}\right\}\) কে ব্যবধি আকারে প্রকাশ কর।
\((b)\) \(|a|\lt{\frac{1}{13}}\) হলে প্রমাণ কর যে, \(|a(a+10)|\lt{\frac{131}{169}}\)
\((c)\) \(\frac{a+5}{a^2+10a+26}\lt{\frac{1}{a+6}}\) হলে, উদ্দীপকের আলোকে \(x\) এর মান নির্ণয় করে সমাধান সেট সংখ্যারেখায় দেখাও।
উত্তরঃ \((a) \ x\in{]-\infty, \frac{7-\sqrt{73}}{2}[}\cup{]\frac{7+\sqrt{73}}{2}, \infty[}\)
\((c) \ \left\{x\in{\mathbb{R}}: -1\lt{x}\lt{1}\right\}\)

\(Q.5.(xix)\) \(a=5x-3, \ a\in{\mathbb{R}}\)
\((a)\) \(S=\left\{x\in{\mathbb{R}}: -1\lt{x+2}\le{1}\right\}\) সেটটিকে ব্যবধি আকারে প্রকাশ কর।
\((b)\) \(|\sqrt{a}-5|\lt{2}\) এর সমাধান সেট নির্ণয় কর।
\((c)\) \(x=1\) হলে প্রমাণ কর যে, \(\sqrt{a}\) একটি অমূলদ সংখ্যা।
উত্তরঃ \((a) \ x\in(-3, \ -1]\)
\((b) \ S=\left\{x\in{\mathbb{R}}:\frac{12}{5}\lt{x}\lt{\frac{52}{5}}\right\}\)

\(Q.5.(xx)\) \(f(x)=x\)
\((a)\) \(a, \ b\in{\mathbb{R}}\) হলে প্রমাণ কর যে, \(|ab|=|a||b|.\)
\((b)\) \(f(x)\gt{\frac{2}{f(x)}}\) এর সমাধান নির্ণয় কর।
\((c)\) \(|f(x)-1|\lt{3}\) হলে দেখাও যে, \(|x^3-1|\lt{63}\)
উত্তরঃ \((b) \ x\lt{-\sqrt{2}}\) অথবা, \(x\gt{\sqrt{2}}\)

\(Q.5.(xxi)\) \(f(x)=x-1;\) যেখানে, \(x\in{\mathbb{R}}\)
\((a)\) \(-2\lt{2-f(x)}\lt{8}\) অসমতাকে পরম মান চিহ্নের সাহয্যে প্রকাশ কর।
\((b)\) \(|f(x)|\lt{\frac{1}{10}}\) হলে দেখাও যে, \(f(x).f(x+2)\lt{\frac{21}{100}}\)
\((c)\) \(|3f(x)-1|\lt{2}\) অসমতাকে সমাধান কর এবং সমাধান সেট সংখ্যারেখায় দেখাও।
উত্তরঃ \((a) \ |x|\lt{5}\)
\((c) \ S=\left\{x\in{\mathbb{R}}: \frac{2}{3}\lt{x}\lt{2}\right\}\)
ঢাঃ ২০১৭ ।
realNumber

\(Q.5.(xxii)\) \(f(x)=ax+by+c, \ g(x)=lx+my+n.\)
\((a)\) \(|2x-1|\lt{\frac{1}{3}}\) এর সমাধান সেট সংখ্যারেখায় দেখাও।
\((b)\) \(a=1, \ b=c=0, \ |f(x)-1|\lt{\frac{1}{11}}\) হলে প্রমাণ কর যে, \(|\left\{f(x)\right\}^2-1|\lt{\frac{23}{121}}\)
\((c)\) \(a=1, \ b=0, \ c=-1\) হলে, \(|3f(x)-1|\lt{2}\) অসমতাকে সমাধান কর এবং সমাধান সেট সংখ্যারেখায় দেখাও।
উত্তরঃ \((a) \ S=\left\{x\in{\mathbb{R}}: \frac{1}{3}\lt{x}\lt{\frac{2}{3}}\right\}\)
realNumber
\((c) \ S=\left\{x\in{\mathbb{R}}: \frac{2}{3}\lt{x}\lt{2}\right\}\)
কুঃ ২০১৭ ।
realNumber

\(Q.5.(xxiii)\) দৃশ্যকল্প-১: \(L=\left\{x\in{\mathbb{R}}: 2x^2+5x\lt{0}\right\}.\)
দৃশ্যকল্প-২: \(f(x)=x^2-x.\)
\((a)\) সমাধান করঃ \(|2x-7|\gt{5}.\)
\((b)\) \(L\) এর সমাধান সেটের অসমতাটিকে পরম মান চিহ্নের সাহয্যে প্রকাশ কর।
\((c)\) সংখ্যারেখার সাহায্যে \(f(x)\le{0}\) এর সমাধান নির্ণয় কর।
উত্তরঃ \((a) \ S=\left\{x\in{\mathbb{R}}: x\lt{1} \text{ অথবা} \ x\gt{6}\right\}\)
\((b) \ \left|x+\frac{5}{4}\right|\lt{\frac{5}{4}}\)
\((c) \ S=\left\{x\in{\mathbb{R}}: 0\le{x}\le{1}\right\}\)
যঃ ২০১৭ ।

\(Q.5.(xxiv)\) \(f(x)=x\) এবং \(a, \ b\in{\mathbb{R}}.\)
\((a)\) \(|f(x)|\lt{a}\) হলে প্রমাণ কর যে, \(-a\lt{f(x)}\lt{a}\) যেখানে, \(a\gt{0}\)
রাঃ ২০০১ ।
\((b)\) প্রমাণ কর যে, \(\sqrt{\left\{f(a)\right\}^2}=|f(a)|\)
সিঃ ২০০৩ ।
\((c)\) প্রমাণ কর যে, \(|f(a)-f(b)|\le{|f(a)|+|f(b)|}\)
চঃ ২০০৯; যঃ২০১০; দিঃ ২০১৪; রুয়েটঃ ২০১২-২০১৩ ।

\(Q.5.(xxv)\) \(P=|x-2|, \ f(x)=x\) এবং \(g(x)=3x+2.\)
\((a)\) \(-6\le{\frac{5x}{2}+1}\le{0}\) অসমতাটিকে পরম মান চিহ্নের সাহয্যে প্রকাশ কর।
\((b)\) \(P\lt{\frac{1}{5}}\) হলে দেখাও যে, \(|x^2-4|\lt{\frac{21}{25}}\)
\((c)\) সমাধান কর এবং সমাধান সেট সংখ্যারেখায় দেখাওঃ \(f(x)g(x)\le{1}.\)
উত্তরঃ \((a) \ |5x+8|\le{6}\)
\((c) \ -1\le{x}\le{\frac{1}{3}}\)
realNumber

\(Q.5.(xxvi)\) \((1) \ \frac{x}{x^2+1}\lt{\frac{1}{x+1}}.\)
\((2) \ |x-1|\lt{\frac{1}{10}}\)
\((a)\) \(4\lt{x}\lt{10}\) অসমতাটিকে পরম মান চিহ্নের সাহয্যে প্রকাশ কর।
\((b)\) \((1)\) নং সমাধান কর এবং সমাধান সেট সংখ্যারেখায় দেখাও।
\((c)\) \((2)\) নং এর আলোকে দেখাও যে, \(|x^2-1|\le{\frac{21}{100}}.\)
উত্তরঃ \((a) |x-7|\lt{3}\)
\((b) \ \left\{x\in{\mathbb{R}}: -1\lt{x}\lt{1}\right\}\)
realNumber

\(Q.5.(xxvii)\) \(f(x)=x-1, \ g(x)=2x+3\) দুইটি ফাংশন।
\((a)\) \(a, \ b\in{\mathbb{R}}\) হলে প্রমাণ কর যে, \(||a|-|b||\le{|a-b|}.\)
\((b)\) \(|f(a)-4|\lt{\frac{1}{2}}\) হলে দেখাও যে, \(|8a^3+29|\lt{1360}.\)
\((c)\) \(|g(x)|\lt{|f(x)|}\) অসমতাটিকে সমাধান কর এবং সমাধান সেট সংখ্যারেখায় দেখাও।
উত্তরঃ \((c) \ (b) \ \left\{x\in{\mathbb{R}}: -4\lt{x}\lt{-\frac{2}{3}}\right\}\)
realNumber

\(Q.5.(xxviii)\) \(f(x)=\frac{1}{1-4x}\) এবং \(g(x)=x-1\)
\((a)\) \(-6\le{x}\le{-1}\) অসমতাটিকে পরম মান চিহ্নের সাহয্যে প্রকাশ কর।
\((b)\) \(\frac{1}{|f(x)|}\ge{3}, \ x\ne{\frac{1}{4}}\) অসমতাটিকে সমাধান কর এবং সমাধান সেট সংখ্যারেখায় দেখাও।
\((c)\) \(\frac{g(x)}{f(x)}\gt{0} \) এর সমাধান কর।
উত্তরঃ \((a) \ |2x+7|\le{5}\)
\((b) \ S=\left\{x\in{\mathbb{R}}: x\le{-\frac{1}{2}} \text{ অথবা} \ x\ge{1}\right\}\)
realNumber
\((c) \ S=\left\{x\in{\mathbb{R}}: \frac{1}{4}\lt{x}\lt{1}\right\}\)

\(Q.5.(xxix)\) \(T=\left\{x\in{\mathbb{R}}: 7+6x-x^2\lt{0}\right\}\) এবং \(S=\left\{x\in{\mathbb{R}}: 5x^2-16x+3\lt{0}\right\}\)
\((a)\) \(|x|\gt{x} \) এর সমাধান সেট নির্ণয় কর।
\((b)\) \(T\) এর সুপ্রিমাম \(\left(Sup(T)\right)\) ও ইনফিমাম \(\left(Inf(T)\right)\) নির্ণয় কর।
\((c)\) \(S\) এর সমাধান সেটের অসমতাটিকে পরম মান চিহ্নের সাহয্যে প্রকাশ কর।
উত্তরঃ \((a) \ S=\left\{x\in{\mathbb{R}}: x\lt{0}\right\}\)
\((b)\) সুপ্রিমাম \(\left(Sup(T)\right)\) ও ইনফিমাম \(\left(Inf(T)\right)\) নেই।
\((c) \ |5x-8|\lt{7}\)

\(Q.5.(xxx)\) \(A=\left\{\frac{1}{n+1}: n\in{\mathbb{N}}\right\}\) ও \(B=\left\{\frac{n}{n+1}: n\in{\mathbb{N}}\right\}\) এবং \(f(x)=x-1\) একটি ফাংশন।
\((a)\) \(a, \ b\in{\mathbb{R}}\) হলে প্রমাণ কর যে, \(|a+b|\le{|a|+|b|}.\)
\((b)\) \(A\cup{B}\) এর সুপ্রিমাম \(Sup \) ও ইনফিমাম \(Inf\) নির্ণয় কর।
\((c)\) \(|f(x)|\lt{\frac{1}{2}}\) এবং \(|x^3-1|\lt{p}\) হলে, \(p\) এর মান কত হবে?
উত্তরঃ \((b) \ A\cup{B}\) সেটের সুপ্রিমাম \(1\) এবং ইনফিমাম \(0\)
\((c) \ p=\frac{19}{8}\)

\(Q.5.(xxxi)\) \(f(x)=\frac{x+1}{x-2}\) এবং \(g(x)=x-\frac{1}{3}\)
\((a)\) \(||2-6|-10+|7-3||\) এর মান নির্ণয় কর।
\((b)\) \(xf(x)\gt{0}\) এর সমাধান কর এবং সমাধান সেট সংখ্যারেখায় দেখাও।
\((c)\) \(\frac{1}{|g(x)|}\ge{3}, \ x\ne{\frac{1}{3}}\) অসমতাকে পরম মান চিহ্ন ব্যতিরেকে প্রকাশ কর এবং সংখ্যারেখায় দেখাও।
উত্তরঃ \((a) \ 2\)
\((b) \ S=\left\{x\in{\mathbb{R}}: -1\lt{x}\lt{0}\right\}\cup\left\{x\in{\mathbb{R}}: x\gt{2}\right\}\)
realNumber
\((c) \ S=\left\{x\in{\mathbb{R}}: 0\le{x}\le{\frac{2}{3}}\right\}\)
realNumber

\(Q.5.(xxxii)\) \(f(x)=x\) একটি ফাংশন, যেখানে \(x\in{\mathbb{R}}.\)
\((a)\) যদি \(a\lt{b}\) হয় তবে দেখাও যে, \(a+c\lt{b+c}\) যেখানে \(a, \ b, \ c\in{\mathbb{R}}\)
\((b)\) উদ্দীপকের আলোকে \(a, \ b\in{\mathbb{R}}\) হলে প্রমাণ কর যে, \(|f(a)+f(b)|\le{|a|+|b|}.\)
\((c)\) \(|f(2x)-1|\lt{\frac{1}{9}}\) হলে উদ্দীপকের আলোকে দেখাও যে, \(|4f(x^2)-1|\lt{\frac{19}{81}}\)

\(Q.5.(xxxiii)\) \(f(x)=3x-4, \ g(x)=5x+6\)
\((a)\) \(|-5-7|-|-2+9|+|-3|\) এর মান নির্ণয় কর।
\((b)\) \(\frac{1}{|f(x)|}\ge{5}\) অসমতাটির সমাধান সেট সংখ্যারেখায় দেখাও।
\((c)\) \(\frac{(x-1)f(x)}{g(x)}\lt{0} \) অসমতাটির সমাধান নির্ণয় কর।
উত্তরঃ \((a) \ 8\)
\((b) \ S=\left\{x\in{\mathbb{R}}: \frac{19}{15}\le{x}\lt{\frac{4}{3}} \text{ অথবা} \ \frac{4}{3}\lt{x}\le{\frac{7}{5}}\right\}\)
realNumber
\((c) \ S=\left\{x\in{\mathbb{R}}: x\lt{-\frac{6}{5}} \text{ অথবা} \ 1\lt{x}\lt{\frac{4}{3}}\right\}\)

\(Q.5.(xxxiv)\) \(f(x)=|x+1|\) এবং \(g(x)=|x-1|\)
\((a)\) দেখাও যে, \(f(x)+g(x)\ge{|2x|}.\)
\((b)\) \(f(x)+g(x)\le{3}\) হলে সমাধান সেট নির্ণয় কর।
\((c)\) সংখ্যারেখার সাহায্যে \(f(x)\le{g(x)}\) এর সমাধান নির্ণয় কর।
উত্তরঃ \((b) \ S=\left\{x\in{\mathbb{R}}: -\frac{3}{2}\le{x}\le{\frac{3}{2}}\right\}\)
\((c) \ S=\left\{x\in{\mathbb{R}}: x\le{0}\right\}\)

\(Q.5.(xxxv)\) \(f(x)=x\left(\frac{x-4}{x-5}\right)\) একটি ফাংশন এবং \(B=\left\{1+\frac{(-1)^n}{n}: n\in{\mathbb{N}}\right\}\) একটি সেট।
\((a)\) প্রমাণ কর যে, \(|a-c|\le{|a-b|+|b-c|};\) যেখানে \(a, \ b, \ c\in{\mathbb{R}}\)
\((b)\) \(f(x)\lt{0}\) অসমতাটির সমাধান কর এবং সমাধান সেট সংখ্যারেখায় দেখাও।
\((c)\) \(B\) সেটটির সুপ্রিমাম \(Sup \) ও ইনফিমাম \(Inf\) নির্ণয় কর।
উত্তরঃ \((b) \ S=\left\{x\in{\mathbb{R}}: x\lt{0} \text{ অথবা} \ 4\lt{x}\lt{5}\right\}\)
realNumber
\((c) \ \therefore B\) সেটের সুপ্রিমাম \(\frac{3}{2}\) এবং ইনফিমাম \(0\)

\(Q.5.(xxxvi)\) \(f(x)=3x-2, \ |x+y|\lt{\frac{7}{3}}.\)
\((a)\) যদি \(p, \ q, \ r\in{\mathbb{R}}, \ pq=rq\) এবং \(q\ne{0}\) হয় তবে দেখাও যে, \(p=r.\)
\((b)\) \(\frac{1}{|f(x-2)+3|}\gt{3}\) এর সমাধান সেট সংখ্যারেখায় দেখাও।
\((c)\) দেখাও যে, \(|f(2x)+f(2y)|\lt{10}.\)
উত্তরঃ \((b) \ S=\left\{x\in{\mathbb{R}}: \frac{14}{9}\lt{x}\lt{\frac{16}{9}}\right\}, \ x\ne{\frac{5}{3}}\)
realNumber

\(Q.5.(xxxvii)\) \(P=x+y-3, \ Q=2x-y-5\)
\((a)\) পরম মান চিহ্ন ব্যতিত প্রকাশ করঃ \(|x-2|\lt{5}.\)
\((b)\) \(y=-x\) হলে \(\frac{1}{|Q|}\gt{2}\) এর সমাধান নির্ণয় করে সংখ্যারেখায় উপস্থাপন কর। যখন \(x\ne{\frac{5}{3}}\)
\((c)\) \(P\gt{0}\) এবং \(Q\gt{0}\) অসমতাযুগলের সমাধান সেটের লেখচিত্র অঙ্কন কর।
উত্তরঃ \((a) \ -3\lt{x}\lt{7}\)
\((b) \ S=\left\{x\in{\mathbb{R}}: \frac{3}{2}\lt{x}\lt{\frac{11}{6}} \text{ এবং} \ x\ne{\frac{5}{3}}\right\}\)
realNumber

\(Q.5.(xxxviii)\) \(f(x)=2x+3\) এবং \(g(x)=\sqrt{x}\)
\((a)\) \(x\) এর মান কত হলে \(f=g^2\) হবে?
\((b)\) \(|f(x)|\lt{7}\) সমাধান করে সমাধান সেট সংখ্যারেখায় দেখাও।
\((c)\) প্রমাণ কর যে, \(g(3)\) একটি অমূলদ সংখ্যা।
উত্তরঃ \((a) \ x=-3\) হলে, \(f=g^2\) হবে।
\((b) \ S=\left\{x\in{\mathbb{R}}: -5\lt{x}\lt{2}\right\}\)
realNumber

\(Q.5.(xxxix)\) \(P(x)=x-4\) এবং \(Q(y)=y-3\) দুইটি ফাংশন।
\((a)\) \(a\in{\mathbb{R}}\) হলে দেখাও যে, \(\sqrt{a^2}=|a|.\)
\((b)\) \(\frac{P(x)}{x+2}+\frac{x+6}{Q(x)}\gt{0}\) অসমতাটির সমাধান কর।
\((c)\) \(\frac{Q(x)-P(x)}{|3P(x)-Q(x)|}\le{\frac{P(7)}{Q(5)}}\) অসমতাটির সমাধান কর এবং সমাধান সেট সংখ্যারেখায় দেখাও।
উত্তরঃ \((b) \ S=\left\{x\in{\mathbb{R}}: x\lt{-2} \text{ অথবা} \ x\gt{3}\right\}\)
\((c) \ S=\left\{x\in{\mathbb{R}}: x\ge{\frac{29}{6}} \text{ অথবা} \ x\le{\frac{25}{6}}\right\}\)
realNumber

\(Q.5.(xL)\) \(x=a+5, \ a\in{\mathbb{R}}\)
\((a)\) \(p, \ q\in{\mathbb{R}}\) হলে প্রমাণ কর যে, \(|pq|={|p||q|}.\)
\((b)\) \(|a|\lt{\frac{1}{13}}\) হলে দেখাও যে, \(|x^2-25|\lt{\frac{131}{169}}\)
\((c)\) \(\frac{a+5}{a^2+10a+26}\lt{\frac{1}{a+6}}\) হলে উদ্দীপকের আলোকে \(x\) এর মান নির্ণয় করে সমাধান সেট সংখ্যারেখায় দেখাও।
উত্তরঃ \((c) \ S=\left\{x\in{\mathbb{R}}: -1\lt{x}\lt{1}\right\}\)
realNumber

\(Q.5.(xLi)\) উদ্দীপক-১: \(\frac{x+2}{x+1}\gt{\frac{x-3}{x-4}}\)
উদ্দীপক-২: \(x, \ y\in{\mathbb{Q}}\) এবং \(x\sqrt{2}-y\sqrt{3}=0\)
\((a)\) \(|x+2|\lt{2}\) এবং \(x\in{\mathbb{Z}}\) এর সমাধান কর
\((b)\) উদ্দীপক-১ এর অসমতাটি সমাধান কর এবং সমাধান সেট সংখ্যারেখায় দেখাও।
\((c)\) উদ্দীপক-২: ব্যবহার করে দেখাও যে, \(x=y=0.\)
উত্তরঃ \((a) \ x=-3, \ -2, \ -1\)
\((b) \ S=\left\{x\in{\mathbb{R}}: -1\lt{x}\lt{4}\right\}\)
realNumber

\(Q.5.(xLii)\)
realNumber
\((a)\) \(a, \ b, \ c\in{\mathbb{R}}\) এবং \(a+b=a+c\) হলে প্রমাণ কর যে, \(b=c.\)
\((b)\) অন্তর্বৃত্ত অপেক্ষা বড় এবং বহির্বৃত্ত অপেক্ষা ছোট বৃত্তগুলোর ব্যাসার্ধকে পরমমান সংবলিত অসমতার সাহায্যে প্রকাশ কর।
\((c)\) মধ্যবর্তী বৃত্তগুলোর ক্ষেত্রফল \(A\) হলে প্রমাণ কর যে, \(|A^2-36\pi^2|\lt{28\pi^2}\)
উত্তরঃ \((b) \ |r-\sqrt{2}-1|\lt{\sqrt{2}-1}\)

\(Q.5.(xLiii)\) \(f(x)=x(x+1)\) এবং \(g(x)=x-2\) দুইটি ফাংশন।
\((a)\) \(|g(x)|\lt{5}\) এর সমাধান সেট নির্ণয় কর।
\((b)\) \(f(x)\gt{g(x)}\) হলে অসমতাটির সমাধান সেট নির্ণয় কর।
\((c)\) দেখাও যে, \(f(x)\lt{g(x)+2}\) এর কোনো বাস্তব সমাধান নেই।
উত্তরঃ \((a) \ S=\left\{x\in{\mathbb{R}}: -3\lt{x}\lt{7}\right\}\)
\((b) \ \mathbb{R}\)

\(Q.5.(xLiv)\) \(f(x)=x-1\) যেখানে \(x\in{\mathbb{R}}.\)
\((a)\) \(-2\lt{2-f(x)}\lt{8}\) অসমতাটিকে পরম মান চিহ্নের সাহয্যে প্রকাশ কর।
\((b)\) \(|f(x)|\lt{\frac{1}{10}}\) হলে, দেখাও যে, \(|f(x)\times{f(x+2)}|\lt{\frac{21}{100}}.\)
\((c)\) \(|3f(x)-1|\lt{2}\) অসমতাটি সমাধান কর এবং সমাধান সেট সংখ্যারেখায় দেখাও।
উত্তরঃ \((a) \ |x|\le{5}\)
\((c) \ S=\left\{x\in{\mathbb{R}}: \frac{2}{3}\lt{x}\lt{2}\right\}\)
realNumber
ঢাঃ ২০১৭ ।

\(Q.5.(xLv)\) দৃশ্যকল্প-১: \(L=\left\{x\in{\mathbb{R}}: 2x^2+5x\lt{0}\right\}.\)
দৃশ্যকল্প-২: \(f(x)=x^2-x.\)
\((a)\) সমাধান করঃ \(|2x-7|\gt{5}.\)
\((b)\) \(L\) এর সমাধান সেটের অসমতাটিকে পরম মান চিহ্নের সাহয্যে প্রকাশ কর।
\((c)\) সংখ্যারেখার সাহায্যে \(f(x)\le{0}\) এর সমাধান নির্ণয় কর।
উত্তরঃ \((a) \ S=\left\{x\in{\mathbb{R}}: x\lt{1} \text{ অথবা} \ x\gt{6}\right\}\)
\((b) \ \left|x+\frac{5}{4}\right|\lt{\frac{5}{4}}\)
\((c) \ S=\left\{x\in{\mathbb{R}}: 0\le{x}\le{1}\right\}\)
ঢাঃ ২০১৭ ।

\(Q.5.(xLvi)\) \((i) \ -6.5\le{x}\le{-3.5}\)
\((ii) \ |4-3x|\ge{5}\)
\((a)\) মূলদ ও অমূলদ সংখ্যার পার্থক্য লিখ।
\((b)\) \((i)\) কে পরম মান চিহ্নের সাহয্যে প্রকাশ কর।
\((c)\) \((ii)\) এর সমাধান কর এবং সমাধান সেট সংখ্যারেখায় দেখাও।
উত্তরঃ \((b) \ |2x+10|\le{3}\)
\((c) \ S=\left\{x\in{\mathbb{R}}: x\le{-\frac{1}{3}} \text{ অথবা} \ x\ge{3}\right\}\)
realNumber

Read Creative Question
ভর্তি পরীক্ষায় আসা প্রশ্নসমূহ
\(Q.6.(i)\) অসমতা \(x^2\le{x}\) এর সমাধান কি হবে?
উত্তরঃ \(0\le{x}\le{1}\)
বুটেক্সঃ ২০০৯-২০১০।

\(Q.6.(ii)\) \(A=\{0, \ 1, \ 2, \ 3, \ 4, \ 5\}\) হলে, এর ক্ষুদ্রতম ঊর্ধবসীমা কত?
উত্তরঃ \(5\)
বুটেক্সঃ ২০০৯-২০১০।

\(Q.6.(iii)\) \(5x-x^2-6\gt{0}\) হলে, \(x\) এর মান নির্ণয় কর।
উত্তরঃ \(2\lt{x}\lt{3}\)
বুটেক্সঃ ২০০৭-২০০৮।

\(Q.6.(iv)\) প্রমাণ কর যে, \(|a-b|\le{|a|+|b|}\) ।
বুয়েটঃ ২০১২-২০১৩।

\(Q.6.(v)\) মান নির্ণয় করঃ \(|3x-4|\lt{2}\) ।
উত্তরঃ \(\frac{2}{3}\lt{x}\lt{2}\)
বুয়েটঃ ২০১০-২০১১।

সমাধান করঃ
\(Q.6.(vi).(a)\) \(\frac{1}{|3x+1|}\ge{5}\) ।
উত্তরঃ \(-\frac{2}{5}\le{x}\le{-\frac{4}{15}}\) এবং \(x\ne{-\frac{1}{3}}\)
রুয়েটঃ ২০১০-২০১১।

সমাধান করঃ
\(Q.6.(vi).(b)\) \(\frac{1}{|3x-5|}\gt{2}\) ।
উত্তরঃ \(\frac{3}{2}\le{x}\le{\frac{11}{6}}\) এবং \(x\ne{\frac{5}{3}}\)
রুয়েটঃ ২০০৯-২০১০।

\(Q.6.(vi).(c)\) \(2\le{\frac{1}{|x-1|}}\) ।
উত্তরঃ \(\frac{1}{2}\le{x}\le{\frac{3}{2}}\) এবং \(x\ne{1}\)
রুয়েটঃ ২০০৪-২০০৫।

\(Q.6.(vii)\) বাস্তব সংখ্যার উপসেট \(S=\{x: 5x^2-16x+3\lt{0}\}\) এর বৃহত্তম নিম্নসীমা ও ক্ষুদ্রতম ঊর্ধবসীমা নির্ণয় কর।
উত্তরঃ বৃহত্তম নিম্নসীমা \(\left(Inf(S)\right)=\frac{1}{5}\) এবং ক্ষুদ্রতম ঊর্ধবসীমা \(\left(Sup(S)\right)=3\)
কুয়েটঃ ২০০৪-২০০৫।

Read Admission Question

Read More

Post List

Mathematics

Geometry 11 and 12 standard
Algebra 11 and 12 standard
Trigonometry 11 and 12 standard
Diff. Calculus 11 and 12 standard
Int. Calculus 11 and 12 standard
Geometry Honours course standard
Vector 11 and 12 standard
Vector Honours course standard
Algebra 9 and 10 standard
    Coming Soon !

Chemistry